You are on page 1of 164

PAPER – 3 : ADVANCED AUDITING AND PROFESSIONAL ETHICS

PART – I ACADEMIC UPDATE


(Legislative Amendments / Notifications / Circulars / Rules / Guidelines issued by
Regulating Authority)
Students are required to refer updated Chapters applicable for May 2021 Examination on
the below mentioned link:
Chapter. Chapter Name Link
No.
Chapter 5 The Company Audit https://resource.cdn.icai.org/63102bos51055-
cp5.pdf
Chapter 11 Audit of Non-Banking https://resource.cdn.icai.org/63103bos51055-
Financial Companies cp11.pdf
Chapter 13 Audit of Public Sector https://resource.cdn.icai.org/63104bos51055-
Undertakings cp13.pdf
Chapter 14 Liabilities of Auditors https://resource.cdn.icai.org/63105bos51055-
cp14.pdf

Note: Students are also advised to refer RTP of Paper 1 Financial Reporting (for AS, Ind AS
and other updates) and Paper 4 Part I -Corporate Laws (for academic updates relating to
Company Law).

PART – II : QUESTIONS AND ANSWERS

QUESTIONS
PART A: MULTIPLE CHOICE QUESTIONS
Integrated Case Scenario 1.
M/s. Suresh & Co., a partnership firm, has been appointed, for the 7 th consecutive year, as the
statutory auditor of Alkis Ltd., an unlisted public company, for financial year 2020-21.
Mr. Suresh is the engagement partner for the audit assignment of Alkis Ltd. The engagement
team, before starting the assignment, was made to read the policies and procedures designed
to achieve desired quality control, with respect to the type of assignment being underta ken.

© The Institute of Chartered Accountants of India


PAPER – 3 : ADVANCED AUDITING AND PROFESSIONAL ETHICS 53

Mr. Suresh, referred the engagement letter, signed with the management initially and was
considering whether there was a requirement to send a new engagement letter, in light of
following circumstances in the Company during F.Y. 2020-21:
• Two senior whole time directors of the Company have retired out of total five directors.
• 40% stake in the Company was held by promoters, which was reduced to 5%, by selling
shares to general public.
• One more factory unit was set up in Gorakhpur, this year.
• Management has requested to cover 90% of the transactions with respect to each revenue
line item, this time, instead of 80% of the transactions, as was set out in the audit plan,
considering the materiality and other factors.
The following data is presented from the audited financial statements of Alkis Ltd., for the
financial year 2019-2020:
(I) Paid up share capital - ` 8 crore;
(II) Turnover - ` 55 crore;
(III) Outstanding Borrowings - ` 14 crore;
(IV) Outstanding Public Deposits - ` 28 crore.
Mr. Suresh while preparing a report under section 143 of the Companies Act, 2013, made a
statement with respect to the remuneration paid by the Alkis Ltd. to one of its directors,
Mr. Mahesh, was in excess of the limit laid down under section 197 and also gave such other
details as prescribed.
Mr. Suresh, for additional reporting purpose, while auditing with respect to compliance with
CARO, 2016, observed the following, relevant to Para 3(vii) of CARO , 2016:
Statutory Dues Undisputed Date Payable Date Paid
Amount (` in lakh)
Income Tax Demand for 2 25th September, 2020 28th March, 2021
A.Y. 2018-19
GST 1.5 3rd October, 2020 4th April, 2021
Customs Duty 0.80 20th September, 2020 10th April, 2021
Provident Fund 0.45 12th October, 2020 Not Paid till date

Also, a representation was made to GST Department for waiving a penalty of ` 1 lakh for late
payment of GST demand.
The board of Alkis Ltd. declared interim dividend of ` 20 lakh on 20th May, 2021, to its 180
shareholders, out of surplus in the profit and loss account and such dividend amount was
deposited in a separate bank with a branch of SBI.

© The Institute of Chartered Accountants of India


54 FINAL (NEW) EXAMINATION: MAY, 2021

Dividend amounting to ` 1 lakh was not claimed by a shareholder, Mr. Rohit, till 19 th June, 2021,
and so the said amount of ` 1 lakh was transferred to Unpaid Dividend Account on 31 st July,
2020.
On the basis of the abovementioned facts, you are required to answer the followin g MCQs:
Question No.: (1-5)
1. Which of the following option is correct with respect to Alkis Ltd.?
(a) Alkis Ltd. needs to form an Audit Committee. Further, provisions relating to internal
audit as well as rotation of auditors are applicable to Alkis Ltd.
(b) Alkis Ltd. need not to form an Audit Committee. Further, provisions relating to internal
audit is not applicable to Alkis Ltd. However, the provisions with respect to rotation of
auditors are applicable to it.
(c) Alkis Ltd. need not to form an Audit Committee. Further, provisions relating to rotation
of auditors is not applicable to Alkis Ltd. However, the provisions with respect to
internal audit are applicable to it.
(d) Alkis Ltd. needs to form an Audit Committee. Provisions relating to internal audit is
applicable to Alkis Ltd. However, the provisions with respect to rotation of auditors
are not applicable to it.
2. Under which section of the auditor’s report, Mr. Suresh needs to report with respect to the
excess remuneration being paid to Mr. Mahesh?
(a) Other Matters Paragraph.
(b) Report on Other Legal and Regulatory Requirements.
(c) Basis for Qualified Opinion.
(d) Auditor’s Responsibilities for the Audit of the Financial Statements.
3. What total amount of statutory dues needs to be reported by Mr. Suresh as per Para 3 of
CARO?
(a) ` 2.75 lakh.
(b) ` 0.80 lakh.
(c) ` 2.80 lakh.
(d) ` 2.30 lakh.
4. How much amount of interest Alkis Ltd. would be liable to pay with respect to unpaid
dividend amount?
(a) ` 575.
(b) ` 1,216.
(c) ` 1,726.

© The Institute of Chartered Accountants of India


PAPER – 3 : ADVANCED AUDITING AND PROFESSIONAL ETHICS 55

(d) ` 1,151.
5. By what date, the amount of interim dividend should have been deposited in the scheduled
bank after being declared and also by what date, the unpaid or unclaimed dividend amount
should have been transferred to Unpaid Dividend Account?
(a) 25th May, 2021 and 24 th June, 2021, respectively.
(b) 25th May, 2021 and 26 th June, 2021, respectively.
(c) 30th May, 2021 and 19 th July, 2021, respectively.
(d) 27th May, 2021 and 26 th June, 2021, respectively.
Integrated Case Scenario 2.
Victor & Co; a reputed Chartered Accountants firm is appointed as a Statutory auditor of Copper
Man Creations Limited. The Company is into manufacturing of robotic products. The Company
has advanced in all its endeavours by supplying million Copper suits. The Company has started
the production of version 10 under its flagship and tags it as “Why to worry about a vehicle,
when you have steel man”. The main idea of the Company evolved after the promoter watched
the Marvel series Iron Man. The product has been promoted by Robert Downy Jr as its product
Brand Ambassador. The Company expects itself to manufacture these prototypes and expects
the old prototypes to be obsolete due to the demand for version 10. Each version of the product
has a separate department and promotes their sales under the single flagship of ‘Copper Man’
and thus, the managerial decision making is left to each version manager. You have assigned
the ‘Fixed Assets area’ to Mr. Mamma Mia and he came out to you with the following points.
You need to answer the questions raised by him and go through the notes prepared to reach a
reasonable conclusion over Property, Plant and Equipment FSLI (Financial Statement caption):
• The Company is holding the property in its name in Andaman & Nicobar while the land is
registered in another person’s name. The property is in dispute for the past 20 years. This
is the major plant for the Company and it is the critical success factor for the client. The
Company’s 80% of the revenue is derived from this factory. When enquired with
management, it would have to incur huge costs to relocate and the present advantageous
conditions of the plant are very critical for the product manufactured. The Company has
not conducted the physical verification of fixed assets since last 10 years but it has
conducted the verification at other locations every year. When enquired with management,
the Company explained it is highly impossible as the plant is 24*7 running and it couldn’t
be halted as the restart of operations will cost huge amounts and a month’s time to get the
Company back to current position.
• The audit team has come across a transaction where the Company is enjoying the property
rent free. The audit team is of the opinion that the provisions of Benami transactions
(Prohibition) Act, 1988 might apply in such scenario. This should be evaluated as part of
CARO reporting. No other procedures in this regard need to be performed.

© The Institute of Chartered Accountants of India


56 FINAL (NEW) EXAMINATION: MAY, 2021

• The Company follows the depreciation policy as per the Schedule II across all the factories
even when the factory at Andaman & Nicobar is the only factory that runs 24*7. The useful
life has been taken as it is mentioned in the Schedule II without modifications and the
Company’s future prospects are good, there are no impairment indications.
On the basis of the abovementioned facts, you are required to choose the most appropriate
answer for the following MCQs:
Question No.: (6-10)
6. The audit team has asked you about the Benami Transaction:
(a) There is no requirement for the auditor to report the transaction as there are no
proceedings initiated or pending against the Company under the Benami
Transactions (Prohibition) Act, 1988.
(b) As the auditor is not sure about the transaction and did not gather proper evidence,
he can ignore the transaction. The auditor needs to obtain the representation letter
and note the same as a follow up point for the next year audit.
(c) The auditor needs to obtain the additional evidence about the transaction. He needs
to assess the situation as to its impact over the financial statements along-with
consideration of SA 250. Thus, he should consider the seriousness of matter and
should assess the impact of the same over the report even though it is not required
to be reported as part of CARO.
(d) The auditor needs to report such matter as a part of CARO as it might turn into a
potential issue under the Benami Transactions (Prohibition) Act, 1988.
7. The audit team has asked you about the implications of dispute on the Property, Plant and
Equipment and whether any additional considerations/reporting are needed for the same:
(a) The dispute on account of Property, Plant and Equipment is a civil case and one or
the other Company may face such consequences. Thus, no additional audit
procedures are required. However, auditor may report this fact under CARO.
(b) The Property, Plant and Equipment is in dispute and the Company has to incur huge
costs to identify the ideal plant with same conditions. Thus, this might amount to
material uncertainty on the Company’s side to continue as a going concern. Thus, he
needs to report the same. However, he need not to report under CARO.
(c) The Property, Plant and Equipment is under dispute, the auditor needs to report it as
a key audit matter and request the Company to disclose it in notes to accounts in a
single line that the property is in dispute. However, he need not to report under CARO.
(d) The Company’s major line of business is from the factory, which is under dispute, the
audit team need to consider the status of the case and assess its implications over
the going concern assumption of the Company if it loses its case. It should also report
it as part of Sec 143(3) about the Company’s financial transactions or matters which

© The Institute of Chartered Accountants of India


PAPER – 3 : ADVANCED AUDITING AND PROFESSIONAL ETHICS 57

have an adverse impact on the functioning of the Company. It also needs to be


reported as per CARO.
8. The audit team has asked you about the impairment of assets of the Company.
(a) The Company has no impairment condition as the Company expects positive future
cash flows from the assets and thus no need to assess the impairment.
(b) The Company need to assess the impairment condition for the assets and need to
assess the fair value of the assets used to generate income from the older versions.
The auditee needs to take a decision based on the cash inflows of a Company as a
whole for assessing the existence of the impairment condition.
(c) There exists an impairment condition as the Company does not expect much business
from the older versions due to anticipation of the huge demand of the new product.
The Company need to assess the cash inflows at each version level.
(d) The Company need not assess impairment of assets as this is very common in
dynamic industries where the older versions become obsolete when the new one is
introduced by the Company.
9. The audit team is sceptical about the Depreciation policy followed by the Company for the
Andaman and Nicobar plant:
(a) As the Company is following the Schedule II, the depreciation policy and the useful
life is in line with the Companies Act, 2013. Hence the Company’s depreciation policy
is good to go.
(b) As the Company is operating the plant 24*7, it will be eligible for extra shift
depreciation as per Schedule II. For the assets where the condition of extra shift
depreciation does not exist, the Company will be eligible to claim 50% extra
depreciation as per schedule II.
(c) As the Schedule III is applicable for the whole Company, the policy including useful
life for the assets need to be same. There cannot be different useful lives for different
assets across different locations. Thus, the depreciation policy of the Company is
good to go.
(d) As the Company is operating the plant 24*7, it will be referred to as continuous
process plant. For the assets where the condition of extra shift depreciation does not
exist, the Company will not be eligible to claim 100% extra depreciation as per
Schedule II.
10. The audit team has raised a question over hiring an international brand ambassador for an
Indian product and raised concerns over the contract of the same:
(a) The auditor has no role to play in such scenario as the selection of brand ambassador
and the running the business lies with the management. The auditor needs to go
through the agreements entered, payments made etc.

© The Institute of Chartered Accountants of India


58 FINAL (NEW) EXAMINATION: MAY, 2021

(b) The auditor needs to inform the Central Government as this might constitute a serious
non-compliance of laws and regulations. The auditor should also assess the integrity
of the management about the appointment of the foreign brand ambassador.
(c) As per the SA 250, “Consideration of laws and regulations in an audit of Financials
Statements” the auditor needs to assess such matters as it is a legal violation to hire
an international brand ambassador ignoring the local people. The audit team need to
consider the same and report in its audit report about such implications.
(d) The auditor needs to qualify its audit report as the Company is against the “Vocal for
Local” policy. The auditor needs to highlight the same in its audit report as this may
lead to a serious brand deterioration of the Company.
Independent MCQs
11. Anant & Co. is the auditor of ST Insurance Company. The insurance company is also
involved in re-insurance business and necessary provision for re-insurance premium has
been made in the books of accounts. The insurance company is into a re -insurance
whereby their contract relates to one particular risk and is expressed in the re-insurance
policy. Each transaction is negotiated individually, and each party has a free choice i.e. for
the insurance company to offer and the re-insurer to accept. What kind of a re-insurance
business is the insurance company into?
(a) Facultative Re-insurance.
(b) Stop loss treaty re-insurance.
(c) Auto-fac re-insurance.
(d) Proportional treaty re-insurance.
12. Below is an extract from the list of supplier statements as at 31st March 2020 held by t he
Company and corresponding payables ledger balances at the same date along with some
commentary on the noted differences:
Supplier Statement balance Payables ledger balance
`'000 `'000
Cete Company 80 60
Lice Company 185 115
The difference in the balance of Cete Company is due to an invoice which is under dispute
due to defective goods which were returned on 30 th March 2020. Which of the following
audit procedures should be carried out to confirm the balance owing to Cete
Company?
(I) Review post year-end credit notes for evidence of acceptance of return.
(II) Inspect pre year-end goods returned note in respect of the items sent back to the
supplier.

© The Institute of Chartered Accountants of India


PAPER – 3 : ADVANCED AUDITING AND PROFESSIONAL ETHICS 59

(III) Inspect post year-end cash book for evidence that the amount has been settled.
(a) 1, 2 and 3.
(b) 1 and 3 only.
(c) 1 and 2 only.
(d) 2 and 3 only.
PART B : DESCRIPTIVE QUESTIONS
Standards on Auditing, Statements and Guidance Notes
13. (a) Coccyx Ltd. supplies navy uniforms across the country. The Company has 3
warehouses at different locations throughout the India and 5 warehouses at the
borders. The major stocks are generally supplied from the borders. Coccyx Ltd.
appointed M/s OPAQE & Co. to conduct its audit for the financial year 2020-21. Mr.
P, partner of M/s OPAQE & Co., attended all the physical inventory counting
conducted throughout the India but could not attend the same at borders due to some
unavoidable reason.
You are required to advise M/s OPAQE & Co.,
(I) How sufficient appropriate audit evidence regarding the existence and condition
of inventory may be obtained?
(II) How is an auditor supposed to deal when attendance at physical inventory
counting is impracticable?
(b) CA. Dev, a recently qualified practicing Chartered Accountant got his first internal
audit assignment of a large manufacturing concern Growth Limited. As an internal
auditor for Growth Limited, CA. Dev is required to verify whether there are adequate
records for identification and value of Plant and Machinery, tools and dies and
whether any of these items have become obsolescent and not in use. Draft a suitable
audit programme for the above.
(c) (I) In an initial audit engagement, the auditor will have to satisfy about the
sufficiency and appropriateness of ‘Opening Balances’ to ensure that they are
free from misstatements, which may materially affect the current financial
statements. Lay down the audit procedure, you will follow in cases (i) when the
financial statements are audited for the preceding period by another auditor; and
(ii) when financial statements are audited for the first time.
(II) If, after performing the procedure, you are not satisfied about the correctness of
‘Opening Balances’; what approach you will adopt in drafting your audit report
in two situations mentioned in (I) above?

© The Institute of Chartered Accountants of India


60 FINAL (NEW) EXAMINATION: MAY, 2021

The Company Audit & Audit Report


14. Star Ltd. is a power generating company which uses coal as raw material for its power
generating plant. The Company has been allotted coal blocks in the state of Jharkhand
and Odisha. During the FY 2020-21, a scam regarding allotment of coal blocks was
unveiled leading to a ban on the allotment of coal blocks to various companies including
Star Ltd. This happened in the month of December 2020 and as such entire power
generation process of Star Ltd, came to a halt in that month. As a result of such ban, and
the resultant stoppage of the production process, many key managerial personnel of the
company left the Company. There were delays in the of payment of wages and salaries
and the banks from whom the Company had taken funds for project financing also decided
not to extend further finance or to fund further working capital requirements of the
Company.
Further, when discussed with the management, the statutory auditor understood that the
Company had no action plan to mitigate such circumstances. Further, all such
circumstances were not reflected the financial statements of Star Ltd. What course of
action should the statutory auditor of the Company consider in such situation?
Audit Committee and Corporate Governance
15. Mr. Ibrahim was appointed as statutory auditor of New Limited and Old Limited. Both the
Companies were having their base in Chennai they had recently listed their shares on the
Stock Exchange. For the financial year 2020-21, Mr. Ibrahim had signed limited review
reports for each quarter, till the quarter ended 31 st December 2020 for both the companies.
Owing to his personal commitments and increased workload, he tendered his resignation
to M/s New Limited on 30 th January 2021 and asked the Company to appoint another
auditor to issue audit report for the remaining quarter and the FY 2020 -21 as a whole. But
the management of the Company did not accept the same.
Mr. Ibrahim continued to as act as auditor for M/s Old limited. During the 1 st week of March
2021, Mrs. W (wife of Mr. Ibrahim) had borrowed a sum of ` 6 lakh from the Company for
her personal use. Having come to know about this, Mr. Ibrahim immediately informed the
management that he had been disqualified to act as auditor and told them that he
won ’t issue audit report for last quarter. But management of the Company argued that it’s
the legal responsibility of Mr. Ibrahim to do the same.
Whether contention of management of New Limited and Old Limited is justified in asking
Mr. Ibrahim to issue audit report for the last quarter and the FY 2020-21 as a whole, despite
his resignation? Discuss.
Audit of Banks & Insurance Company
16. (a) You are auditing a small bank branch with staff strength of the manager, cashier and
three other staff Peter, Prem and Pran. Among allocation of work for other areas,
Peter who is a peon also opens all the mail and forwards it to the concerned person.
He does not have a signature book so as to check the signatures on important

© The Institute of Chartered Accountants of India


PAPER – 3 : ADVANCED AUDITING AND PROFESSIONAL ETHICS 61

communications. Prem has possession of all bank forms (e.g. Cheque books, demand
draft/pay order books, travelers’ cheques, foreign currency cards etc.). He maintains
a record meticulously which you have test checked also. However, no one among
staff regularly checks that. You are informed that being a small branch with shortage
of manpower, it is not possible to always check the work and records. Give your
comments.
(b) As at 31st March 2020 while auditing Universe Insurance Ltd, you observed that a
policy has been issued on 27 th March 2020 for fire risk favouring one of the leading
corporate houses in the country without the actual receipt of premium and it was
reflected as premium receivable. The Company maintained that it is a usual practice
in respect of big customers and the money was collected on 7th April, 2020. You
further noticed that there was a fire accident in the premises of the insured on 31st
March 2020 and a claim was lodged for the same. The insurance company also made
a provision for claim. Please respond.
Audit under Fiscal Laws
17. (a) Arihant Pvt Ltd is engaged in the business of providing corporate/professional training
programs. It has an annual turnover of INR 74 crore. The Company is subject to tax
audit for which the work has been started by the tax auditor. For the financial year
ending 31 March 2021, the Company applied for GST registration for 5 new locations
for which registration certificates have not yet been received by the Company.
However, the registration number is available on the portal of relevant authority which
can be verified by checking the details of the Company. In this case what should be
the audit procedures to verify this registration number?
(b) Mr. Rohan, made an outward supply of ` 4.00 lakh to M/s. Park Enterprises on 30 th
April, 2020 on a credit period of 15 days. However, M/s Park Enterprises made the
payment to Mr. Rohan after 45 days along with interest for 30 days delayed payment
@ 12%. As such, Mr. Rohan received total payment of ` 4,04,000/- along with
interest. However, while filing Form GSTR-3B/ Form GSTR-1, Mr. Rohan declared
his outward supplies at ` 4.00 lakh. Even while filing Form GSTR-9, Mr. Rohan did
not discharge his tax liability. As a GST auditor in Form GSTR-9C, what action is
recommended by the auditors. Comment.
Internal Audit, Management Audit and Operational Audit
18. M/s Raka & Co., Chartered Accountants have been approached by Abhinandan Ltd., a
company engaged in iron and steel manufacturing industry. The Company has been facing
following operational issues:
(a) Penal interest for delayed payments to the overseas vendors despite having enough
cash flows; and
(b) Despite having regular production and enough inventory, delays in shipping the final
goods to the customers leading to its deteriorating vendor rating.

© The Institute of Chartered Accountants of India


62 FINAL (NEW) EXAMINATION: MAY, 2021

As a partner of M/s Raka & Co., through detailed discussion with the Senior Manager of
Abhinandan Ltd., you have concluded that all these delays are because of long decision -
making cycles in the Company. As a consultant to the Company, would you recommend
Management Audit or Operational Audit?
Professional Ethics
19. Comment on the following with reference to the Chartered Accountants Act, 1949 and
schedules thereto:
(a) C.A. Ajitnath is Special Executive Magistrate. He also took over as the Executive
Chairman of Software Company on 1.4.2020. He is also a leading income tax
practitioner and consultant for derivative products. He resides in Chennai near to the
ION commodity stock exchange and does trading in commodity derivatives. Every
day, he invests nearly 40% of his time to settle the commodity transactions. He has
not taken any permission for becoming Special Executive Magistrate. However, he
has got special permission of Council of ICAI for becoming Executive Chairman. Is
C.A. Ajitnath liable for professional misconduct?
(b) CA. Sambhav, the auditor of Mahvir Pvt. Ltd. has delegated following works to his
articles and staff:
❖ Raising of bills and issuing acknowledgements for money receipts.
❖ Initiating and stamping of vouchers and of schedules prepared for the purpose
of audit.
❖ Issuing acknowledgements for records produced.
❖ Signing financial statements of the company.
Is this correct as per the Professional Ethics and ICAI’s guidelines and
pronouncements?
20. Write a short note on the following:
(a) Technical, Ethical and Professional Standards as per Statement on Peer Review.
(b) Important issues to be kept in mind by the investigator while preparing his report.
(c) Direction by Tribunal in case auditor acted in a fraudulent manner.
(d) Differences between Division II (Ind- AS- Other than NBFCs) and Division III (Ind-
AS- NBFCs) of Schedule III.
(e) Technology based /Digital Forensics Techniques.

© The Institute of Chartered Accountants of India


PAPER – 3 : ADVANCED AUDITING AND PROFESSIONAL ETHICS 63

SUGGESTED ANSWERS

PART A : ANSWERS TO MULTIPLE QUESTIONS


1. (c)
2. (b)
3. (b)
4. (d)
5. (b)
6. (c)
7. (d)
8. (c)
9. (d)
10. (a)
11. (a)
12. (c)
PART B
13. (a) (I) Special Consideration with Regard to Inventory: As per SA 501 “Audit
Evidence- Specific Considerations for Selected Items”, when inventory is
material to the financial statements, the auditor shall obtain sufficient
appropriate audit evidence regarding the existence and condition of inventory
by:
(1) Attendance at physical inventory counting, unless impracticable, to:
(i) Evaluate management’s instructions and procedures for recording
and controlling the results of the entity’s physical inventory counting;
(ii) Observe the performance of management’s count procedures;
(iii) Inspect the inventory; and
(iv) Perform test counts; and
(2) Performing audit procedures over the entity’s final inventory records to
determine whether they accurately reflect actual inventory count results.
(II) Attendance at Physical Inventory Counting Not Practicable: In some cases,
attendance at physical inventory counting may be impracticable. This may be
due to factors such as the nature and location of the inventory, for example,
where inventory is held in a location that may pose threats to the safety of the

© The Institute of Chartered Accountants of India


64 FINAL (NEW) EXAMINATION: MAY, 2021

auditor. The matter of general inconvenience to the auditor, however, is not


sufficient to support a decision by the auditor that attendance is impracticable.
Further, as explained in SA 200 “Overall Objectives of the Independent Auditor
and the Conduct of an Audit in Accordance with Standards on Auditing”, the
matter of difficulty, time, or cost involved is not in itself a valid basis for the
auditor to omit an audit procedure for which there is no alternative or to be
satisfied with audit evidence that is less than persuasive.
Further, where attendance is impracticable, alternative audit procedures, for
example, inspection of documentation of the subsequent sale of specific
inventory items acquired or purchased prior to the physical inventory counting,
may provide sufficient appropriate audit evidence about the existence and
condition of inventory.
In some cases, though, it may not be possible to obtain sufficient appropriate
audit evidence regarding the existence and condition of inventory by performing
alternative audit procedures. In such cases, SA 705 on Modifications to the
Opinion in the Independent Auditor’s Report, requires the auditor to modify the
opinion in the auditor’s report as a result of the scope limitation.
(b) The Internal Audit Programme in connection with Plant and Machinery and
Tools and dies may be on the following lines:
(i) Internal Control Aspects: The following may be incorporated in the audit
programme to check the internal control aspects-
(a) Maintaining separate register for hired assets, leased asset and jointly
owned assets.
(b) Maintaining register of fixed asset and reconciling to physical inspection of
fixed asset and to nominal ledger.
(c) All movements of assets are accurately recorded.
(d) Authorisation be obtained for –
(1) a declaring a fixed asset scrapped.
(2) selling a fixed asset.
(e) Check whether additions to fixed asset register are verified and checked
by authorised person.
(f) Proper recording of all additions and disposal.
(g) Examining procedure for the purchase of new fixed assets, including
written authority, work order, voucher and other relevant evidence.
(h) Regular review of adequate security arrangements.
(i) Periodic inspection of assets is done or not.

© The Institute of Chartered Accountants of India


PAPER – 3 : ADVANCED AUDITING AND PROFESSIONAL ETHICS 65

(j) Regular review of insurance cover requirements over fixed assets.


(ii) Assets Register: To review the registers and records of plant, machinery, etc.
showing clearly the date of purchase of assets, cost price, location, depreciation
charged, etc.
(iii) Cost Report and Journal Register: To review the cost relating to each plant
and machinery and to verify items which have been capitalised.
(iv) Code Register: To see that each item of plant and machinery has been given a
distinct code number to facilitate identification and verify the maintenance of
Code Register.
(v) Physical Verification: To see physical verification has been conducted at
frequent intervals.
(vi) Movement Register: To verify (a) whether Movement Register for movable
equipments and (b) log books in case of vehicles, etc. are being maintained
properly.
(vii) Assets Disposal Register: To review whether assets have been disposed off
after proper technical and financial advice and sales/disposal/retirement, etc. of
these assets are governed by authorisation, sales memos or other appropriate
documents.
(viii) Spare Parts Register: To examine the maintenance of a separate register of
tools, spare parts for each plant and machinery.
(ix) Review of Maintenance: To scrutinise the programme for an actual periodical
servicing and overhauling of machines and to examine the extent of utilisation
of maintenance department services.
(x) Review of Obsolescence: To scrutinise whether expert’s opinion have been
obtained from time to time to ensure purchase of technically most useful efficient
and advanced machinery after a thorough study.
(xi) Review of R&D: To review R&D activity and ascertain the extent of its relevance
to the operations of the organisation, maintenance of machinery efficiency and
prevention of early obsolescence.
(c) (I) (i) Financial Statements Audited by another Auditor – Audit Procedure:
If the prior period’s financial statements were audited by a predecessor
auditor, the auditor may be able to obtain sufficient appropriate audit
evidence regarding the opening balances by perusing the copies of the
audited financial statements including the other relevant documents
relating to the prior period financial statements such as supporting
schedules to the audited financial statements. Ordinarily, the current
auditor can place reliance on the closing balances contained in the financial
statements for the preceding period, except when during the performance

© The Institute of Chartered Accountants of India


66 FINAL (NEW) EXAMINATION: MAY, 2021

of audit procedures for the current period the possibility of misstatements


in opening balances is indicated.
(ii) Audit of Financial Statements for the First Time – Audit Procedure:
When the audit of financial statements is being conducted for the first time,
the auditor has to perform auditing procedures to obtain sufficient
appropriate audit evidence. Since opening balances represent effect of
transaction and events of the preceding period and accounting policies
applied in the preceding period, the auditor need to obtain evidence having
regard to nature of opening balances, materiality of the opening balances
and accounting policies. Since it will not be possible for auditor to perform
certain procedures, e.g., observing physical verification of inventories, etc.
the auditor may obtain confirmation, etc. and perform suitable procedures
in respect of fixed assets, investments, etc. The auditor can also obtain
management representation with regards to the opening balances.
(II) Drafting Audit Report: If the auditor is unable to obtain sufficient appropriate
audit evidence regarding the opening balances, the auditor shall express a
qualified opinion or a disclaimer of opinion, as appropriate. Further, If the auditor
concludes that the opening balances contain a misstatement that materially
affects the current period’s financial statements, and the effect of the
misstatement is not properly accounted for or not adequately presented or
disclosed, the auditor shall express a qualified opinion or an adverse opinion.
14. SA 570- “Going Concern” deals with the auditor’s responsibilities in the audit of financial
statements relating to going concern and the implications for the auditor’s report.
The auditor’s responsibilities are to obtain sufficient appropriate audit evidence regarding,
and conclude on, the appropriateness of management’s use of the going concern basis of
accounting in the preparation of the financial statements, and to conclude, ba sed on the
audit evidence obtained, whether a material uncertainty exists about the entity’s ability to
continue as a going concern.
When the use of going concern basis of accounting is inappropriate i.e., if the financial
statements have been prepared using the going concern basis of accounting but, in the
auditor’s judgment, management’s use of the going concern basis of accounting in the
preparation of the financial statements is inappropriate, the auditor shall express an
adverse opinion.
Also, when adequate disclosure of a material uncertainty is not made in the financial
statements the auditor shall express a qualified opinion or adverse opinion, as appropriate,
in accordance with SA 705 (Revised); and in the Basis for Qualified (Adverse) Opinion
section of the auditor’s report, state that a material uncertainty exists that may cast
significant doubt on the entity’s ability to continue as a going concern and that the financial
statements do not adequately disclose this matter.

© The Institute of Chartered Accountants of India


PAPER – 3 : ADVANCED AUDITING AND PROFESSIONAL ETHICS 67

In the present case, the following circumstances indicate the inability of Star Ltd. to
continue as a going concern:
• Ban on the allotment of coal blocks
• Halt in power generation
• Key Managerial Personnel leaving the Company.
• Banks decided not to extend further finance and not to fund the working capital
requirements of the Company.
• Non availability of sound action plan to mitigate such circumstances.
Therefore, considering the above factors it is clear that the going concern basis is
inappropriate for the Company. Further, such circumstances are not reflected in the
financial statements of the Company. As such, the statutory auditor of Star Ltd. should:
(1) Express an adverse opinion in accordance with SA 705 (Revised) and
(2) In the Basis of Opinion paragraph of the auditor’s report, the statutory auditor should
state that a material uncertainty exists that may cast significant doub t on the entity’s
ability to continue as a going concern and that the financial statements do not
adequately disclose this matter.
15. In the given scenario, Mr. Ibrahim was appointed as statutory auditor of two listed entities
i.e., New Limited and Old Limited. For the financial year 2020-21, Mr. Ibrahim had signed
limited review reports for first three quarter i.e., till the quarter ended 31 st December 2020
for both the companies. Owing to his personal commitments and increased workload, he
resigned from New Limited and asked the Company to appoint another auditor to issue
audit report for the remaining quarter and audit report for the FY 2020-21.
Further, Mr. Ibrahim immediately informed the management of Old Limited that he had
been disqualified to act as auditor and told them that he won’t issue audit report for last
quarter as Mrs. W (wife of Mr. Ibrahim) had borrowed a sum of ` 6 lakh from the Company
for her personal use.
As per SEBI LODR Regulations, if the auditor has signed the limited review/ audit report
for the first three quarters of a financial year, then the auditor shall, before such resignation,
issue the limited review/ audit report for the last quarter of such financial year as well as
the audit report for such financial year. This provision will not apply if the auditor is
disqualified due to Section 141 of the Companies Act, 2013.
Thus, in the given situation, in view of above conditions to be complied with upon
resignation of the statutory auditor of a listed entity/material subsidiary with respect to
limited review / audit report as per SEBI LODR Regulations, Mr. Ibrahim is required to
issue the audit report for the last quarter and audit report for the year 2020 -21 for New
Limited as he has issued audit report for the first three quarters whereas Mr. Ibrahim is not

© The Institute of Chartered Accountants of India


68 FINAL (NEW) EXAMINATION: MAY, 2021

required to issue the audit report for remaining quarter and audit report for the year 2020 -
21 as a whole for Old Limited as he is disqualified under section 141 of Companies Act.
Accordingly, contention of Management of New Limited is correct and tenable for issuing
the audit report for remaining quarter and audit report for financial year 2020 -21 however,
contention of management of Old Limited is not correct regarding the legal responsibility
of Mr. Ibrahim to issue audit report for remaining quarter and for the whole year.
16. (a) Banks are required to implement and maintain a system of internal controls for
mitigating risks, maintain good governance and to meet the regulatory requirements.
Given below are examples of internal controls that are violated in the given situation:
In the instant case, Peter who is a peon opens all the mail and forwards it to the
concerned person. Further, he does not have a signature book so as to check the
signatures on important communications is not in accordance with implementation
and maintenance of general internal control. As the mail should be opened by a
responsible officer. Signatures on all the letters and advices received from other
branches of the bank or its correspondence should be checked by an officer with the
signature book.
All bank forms (e.g. Cheque books, demand draft/pay order books, travelers’
cheques, foreign currency cards etc.) should be kept in the possession of an officer,
and another responsible officer should verify the issuance and stock of such
stationery. In the given case, Prem has possession of all bank forms (e.g. cheque
books, demand draft/pay order books, travelers’ cheques, foreign currency cards
etc.). He maintains a record meticulously which were also verified on test check basis.
Further, contention of bank that being a small branch with shortage of manpower they
are not able to check the work and records on regular basis, is not tenable as such
lapses in internal control pose risk of fraud.
The auditor should report the same in his report accordingly.
(b) No risk can be assumed by the insurer unless the premium is received. According to
section 64VB of the Insurance Act, 1938, no insurer should assume any risk in India
in respect of any insurance business on which premium is ordinarily payable in Ind ia
unless and until the premium payable is received or is guaranteed to be paid by such
person in such manner and within such time, as may be prescribed, or unless and
until deposit of such amount, as may be prescribed, is made in advance in the
prescribed manner. The premium receipt of insurance companies carrying on general
insurance business normally arise out of three sources, viz., premium received from
direct business, premium received from reinsurance business and the share of co -
insurance premium.
In view of the above, the insurance company is not liable to pay the claim and hence
no provision for claim is required.

© The Institute of Chartered Accountants of India


PAPER – 3 : ADVANCED AUDITING AND PROFESSIONAL ETHICS 69

17. (a) Clause (4) (Details as to Indirect Tax Registration) of Part A of Form No. 3CD
generally requires the auditor to ensure whether the assessee is liable to pay indirect
tax like excise duty, service tax, sales tax, goods and service tax, custom duty, etc.
If yes, please furnish the registration number or GST number or any other
identification number allotted for the same. Thus, the auditor is primarily required to
furnish the details of registration numbers as provided to him by the assessee. The
reporting is required to be done in the manner or format specified by the e-filing utility
in this context.
In the given situation, Arihant Pvt Ltd is engaged in the business of providing
corporate/professional training programs. The Company is subject to tax audit. For
the financial year ending 31 March 2021, the Company applied for GST registration
for 5 new locations for which registration certificates have not yet been received by
the Company. However, the registration number is available on the portal of relevant
authority.
In the instant case, the tax auditor of Arihant Private Limited should verify the
registration number for the locations for which registration certificates have not been
received from online portal of the relevant authority.
The auditor should also ensure that the details furnished while checking the
registration number pertains to the company only. If the company has filed any returns
for these locations, the auditor should enquire for the same from the management
and should check those returns to verify the correctness of the registration numbers.
In addition, the auditor should also obtain specific representation in respect of this
point from the management.
(b) In terms of section 15(2)(d) while computing value of taxable supply the transaction
value shall include interest or late fee or penalty for delayed payme nt of any
consideration for any supply. Since, Mr. Rohan did not pay tax on interest component,
he made violation of valuation provisions. Mr. John was having option to discharge
such liability at the time of filing of Form GSTR-9, which he did not avail. Therefore,
the GST auditor may recommend him to discharge such liability at the time of making
reconciliation statement in Form GSTR-9C.
18. A comparison between the Management Audit & the Operational Audit is as follows:
Management audit is concerned with the “Quality of managing”, whereas operational audit
focuses on the “Quality of operations”.
Management audit is the “Audit of management” while the operational audit is the “Audit
for the management”. The focus of Management Audit is on “Quality of Decision Making”
rather than the effectiveness or efficiency of operations.
The basic difference between the two audits, then, is not in method, but in the level of
appraisal. In a management audit, the auditor is to make his tests to the level of top
management, its formulation of objectives, plans and policies and its decision making. It is

© The Institute of Chartered Accountants of India


70 FINAL (NEW) EXAMINATION: MAY, 2021

not that he just verifies the operations of control and procedures and fulfilment of plans in
conformity with the prescribed policies.
Since, the delays in payments and consequent penal interest payments and the delays in
shipping and the consequent deteriorating vendor ratings are happening because of the
delays in decision-making process of the management of Abhinandan Ltd. Therefore, it
appears that this is not just an internal control or operational issue but an issue of
management process.
Therefore, Management Audit would be recommended in this case.
19. (a) Engaging into a Business: As per Clause (11) of Part I of First Schedule of
Chartered Accountants Act, 1949, a Chartered Accountant in practice is deemed to
be guilty of professional misconduct if he engages in any business or occupation
other than the profession of Chartered Accountant unless permitted by the Council
so to engage.
However, the Council has granted general permission to the members to engage in
certain specific occupation. In respect of all other occupations specific permission of
the Institute is necessary.
In this case, C.A. Ajitnath is Special Executive Magistrate, engaged in the occupation
of trading in commodity derivatives and also took over as the Executive Chairman on
01.04.2020.
In this context, it may be noted that the Special Executive Magistrate which is
generally permitted for Members of the Institute in practice, further specific
permission is required for holding the position of Executive Chairman and getting
engaged in the occupation of trading in commodity derivatives.
In the given situation, C.A. Ajitnath is acting as Special Executive Magistrate which
is generally permitted for Members of the Institute in practice. Further, He is engaged
in the occupation of trading in commodity derivatives which is not covered under the
general permission. He also took over as the Executive Chairman for which specific
permission is required. CA. Ajitnath got the permission for the same from the Council
of ICAI.
Conclusion: Hence, CA. Ajitnath is not guilty for acting as Special Executive
Magistrate as it is covered under the general permission. He is also not guilty for
holding the position of Executive Chairman after getting specific permission of the
Institute.
However, he is guilty of professional misconduct under Clause (11) of Part I of First
Schedule of Chartered Accountants Act, 1949 for getting engaged in the occupation
of trading in commodity derivatives which is not covered under the general
permission.

© The Institute of Chartered Accountants of India


PAPER – 3 : ADVANCED AUDITING AND PROFESSIONAL ETHICS 71

(b) As per Clause (12) of Part I of the First Schedule of the Chartered Accountants Act,
1949, a Chartered Accountant in practice is deemed to be guilty of professional
misconduct if he allows a person not being a member of the institute in practice or a
member not being his partner to sign on his behalf or on behalf of his firm, any balance
sheet, profit and loss account, report or financial statements.
The Council has clarified that the power to sign routine documents on which a
professional opinion or authentication is not required to be expressed may be
delegated in the following instances and such delegation will not attract provisions of
this clause:
(i) Issue of audit queries during the course of audit.
(ii) Asking for information or issue of questionnaire.
(iii) Letter forwarding draft observations/financial statements.
(iv) Initiating and stamping of vouchers and of schedules prepared for the purpose
of audit.
(v) Acknowledging and carrying on routine correspondence with clients.
(vi) Issue of memorandum of cash verification and other physical verification or
recording the results thereof in the books of the clients.
(vii) Issuing acknowledgements for records produced. Raising of bills and issuing
acknowledgements for money receipts.
(ix) Attending to routine matters in tax practice, subject to provisions of Section 288
of Income Tax Act.
(x) Any other matter incidental to the office administration and routine work involved
in practice of accountancy.
In the instant case, CA. Sambhav, the auditor of Mahvir Pvt. Ltd. has delegated
certain task to his articles and staff such as raising of bills and issuing
acknowledgements for money receipts, initiating and stamping of vouchers and of
schedules prepared for the purpose of audit and issuing acknowledgements for
records produced and signing financial statements of the company.
Therefore, CA. Sambhav is correct in allowing first three tasks i.e., raising of bills and
issuing acknowledgements for money receipts, initiating and stamping of vouchers
and of schedules prepared for the purpose of audit.
However, if the person signing the financial statements on his behalf is not a member
of the institute in practice or a member not being his partner to sign on his behalf or
on behalf of his firm, CA. Sambhav is not right in delegating signing of financial
statements to his staff.
Conclusion: In view of this, CA. Sambhav would be guilty of professional misconduct
for allowing the person signing the financial statements on his behalf to his articles

© The Institute of Chartered Accountants of India


72 FINAL (NEW) EXAMINATION: MAY, 2021

and staff under Clause 12 of Part 1 of First Schedule of the Chartered Accountants
Act, 1949.
20. (a) As per the Statement on Peer Review, Technical, Professional and Ethical
Standards means:
(i) Accounting Standards issued by ICAI that are applicable for entities other than
companies under the Companies Act, 2013;
(ii) Accounting Standards prescribed under section 133 of the Companies Act; 2013
by the Central Government based on the recommendation of ICAI and in
consultation with the National Financial Reporting Authority (NFRA) and notified
as Accounting Standards Rules 2006, as amended from to time;
(iii) Indian Accounting Standards prescribed under section 133 of the Companies
Act 2013 by the Central Government based on the recommendation of ICAI and
in consultation with NFRA and notified as Companies (Indian Accounting
Standards) Rules, 2015, as amended from time to time;
(iv) Standards :

Standards issued by the Institute of Chartered Accountants of India


including-
(a) Engagement standards
(b) Statements
(c) Guidance notes
(d) Standards on Internal Audit.
(e) Guidelines/ Notifications / Directions / Announcements /
Pronouncements / Professional Standards issued from time to time by
the Council or any of its Committees.

(v) Framework for the preparation and presentation of financial statements, Preface
to the Standards on Quality Control, Auditing, Review, Other Assurance and
Related Services and Framework for Assurance engagements;
(vi) Provisions of the relevant statutes and / or rules or regulations which are
applicable in the context of the specific engagements being reviewed including
instructions, guidelines, notifications, directions issued by regulatory bodies as
covered in the scope of assurance engagements.
(b) The important issues to be kept in mind by the investigator while preparing his
report are as follows:
(i) The report should not contain anything which is not relevant either to highlight
the nature of the investigation or the final outcome thereof.

© The Institute of Chartered Accountants of India


PAPER – 3 : ADVANCED AUDITING AND PROFESSIONAL ETHICS 73

(ii) Every word or expression used should be properly considered so that the
possibility of arriving at a different meaning or interpretation other than the one
intended by the investigator can be minimized.
(iii) Relevant facts and conclusions should be properly linked with evidence.
(iv) Bases and assumptions made should be explicitly stated. Reasonableness of
the bases and assumptions made should be well examined and care should be
taken to see that none of the bases and assumptions can be considered to be
in conflict with the objective of the investigation. For example, in an investigation
into over-stocking of raw materials, inventories and spares etc. it should not be
assumed that the ordering levels indicated on bin cards provide fair guidance
about acquisition of further materials. Also, since investigation is a fact -finding
assignment, assumptions should be made only when it is unavoidably
necessary.
(v) The report should clearly spell out the nature and objective of the assignment
accepted its scope and limitations, if any.
(vi) The report should be made in paragraph form with headings for the paragraphs.
Any detailed data and figures supporting any finding may be given in Annexures.
(vii) The report should also state restrictions or limitations, if any, imposed on the
instructions given by the client. Preferably the reasons for placing such
restrictions and their impact on the final result should also be stated.
(viii) The opinion of the investigator should appear in the final paragraph of the report.
(c) Direction by Tribunal in case auditor acted in a fraudulent manner: As per sub-
section (5) of the section 140, the Tribunal either suo motu or on an application made
to it by the Central Government or by any person concerned, if it is satisfied that the
auditor of a company has, whether directly or indirectly, acted in a fraudulent manner
or abetted or colluded in any fraud by, or in relation to, the company or its directors
or officers, it may, by order, direct the company to change its auditors.
However, if the application is made by the Central Government and the Tribunal is
satisfied that any change of the auditor is required, it shall within fifteen days of receipt
of such application, make an order that he shall not function as an auditor and the
Central Government may appoint another auditor in his place.
It may be noted that an auditor, whether individual or firm, against whom final order
has been passed by the Tribunal under this section shall not be eligible to be
appointed as an auditor of any company for a period of five years from the date of
passing of the order and the auditor shall also be liable for action under section 447.
It is hereby clarified that the case of a firm, the liability shall be of the firm and that of
every partner or partners who acted in a fraudulent manner or abetted or colluded in
any fraud by, or in relation to, the company or its director or officers.

© The Institute of Chartered Accountants of India


74 FINAL (NEW) EXAMINATION: MAY, 2021

(d) Differences between Division II (Ind- AS- Other than NBFCs) and Division III
(Ind- AS- NBFCs) of Schedule III: The presentation requirements under Division III
for NBFCs are similar to Division II (Non NBFC) to a large extent except for the
following:
(i) NBFCs have been allowed to present the items of the balance sheet in order of
their liquidity which is not allowed to companies required to follow Division II.
Additionally, NBFCs are required to classify items of the balance sheet into
financial and non-financial whereas other companies are required to classify the
items into current and non-current.
(ii) An NBFC is required to separately disclose by way of a note any item of ‘other
income’ or ‘other expenditure’ which exceeds 1 per cent of the total income.
Division II, on the other hand, requires disclosure for any item of income or
expenditure which exceeds 1 per cent of the revenue from operations or ` 10
lakh, whichever is higher.
(iii) NBFCs are required to separately disclose under ‘receivables’, the debts due
from any Limited Liability Partnership (LLP) in which its director is a partner or
member.
(iv) NBFCs are also required to disclose items comprising ‘revenue from operations’
and ‘other comprehensive income’ on the face of the Statement of profit and
loss instead of showing those only as part of the notes.
(v) Separate disclosure of trade receivable which have significant increase in credit
risk & credit impaired.
(vi) The conditions or restrictions for distribution attached to statutory reserves have
to be separately disclose in the notes as stipulated by the relevant statute.
(e) Technology based/Digital Forensics Techniques: Every transaction leaves a
digital footprint in today's computer-driven society. Close scrutiny of relevant emails,
accounting records, phone logs and target company hard drives is a requisite facet
of any modern forensic audit. Before taking steps such as obtaining data from email
etc. the forensic auditor should take appropriate legal advice so that it doesn’t amount
to invasion of privacy. Digital investigations can become quite complex and require
support from trained digital investigators. However, many open-source digital
forensics tools are now available to assist in this phase of the investigation .
Cross Drive Analysis EnCase
Live Analysis MD5
Deleted Files Tracking Log Files
Stochastic Forensics PC System Log
Steganography Free Log Tools

© The Institute of Chartered Accountants of India


PAPER – 3 : ADVANCED AUDITING AND PROFESSIONAL ETHICS
PART – I ACADEMIC UPDATE
(Legislative Amendments / Notifications / Circulars / Rules / Guidelines issued by
Regulating Authority)

Chapter 5: Company Audit


1. Definition of "Government company" [Section 2(45)]: In this section, the following
explanation has been inserted in the definition. Explanation.- For the purposes of this clause,
the "paid up share capital" shall be construed as "total voting power", where shares with
differential voting rights have been issued.] [insertion on page no. 5.4 of Study Material as
per notification dated 2-3-2020 by way of Exemptions to Government Companies under section
462 with effect from 3-3-2020 ]
2. “The auditor of the company shall, in his report under section 143, make a statement as
to whether the remuneration paid by the company to its directors is in accordance with the
provisions of this section, whether remuneration paid to any director is in excess of the limit
laid down under this section and give such other details as may be prescribed” (as per section
197(16) of the Companies Act, 2013)
As per Advisory issued by ICAI, the aforesaid reporting requirement for auditors of public
companies needs to be covered in auditor’s report under the Section “Report on Other Legal
and Regulatory Requirements”. Accordingly, auditors of public companies are advised to
comply with the aforesaid reporting requirements in their auditor’s reports. (in) (insertion in
point no 4 on page no 5.32 of Study Material.)
3. In section 132—
(i) after sub-section (1), the following sub-section shall be inserted, namely:—
“(1A) The National Financial Reporting Authority shall perform its functions through
such divisions as may be prescribed.”;
[Amendment to be incorporated on Pg 5.47 of SM]
(ii) Every auditor referred to in Rule 3 shall file a return with the NFRA on or before
30th November every year in Form NFRA-2”. (Insertion on Page no 5.48 of SM)
(iii) The Central Government has amended the National Financial Reporting Authority
Rules, 2018, by the National Financial Reporting Authority (Amendment) Rules, 2019.
In the National Financial Reporting Authority Rules, 2018, after clause (c) of sub-rule
(1) of rule 3, the following explanation shall be inserted, namely:-

© The Institute of Chartered Accountants of India


52 FINAL (NEW) EXAMINATION: NOVEMBER, 2020

“Explanation.- For the purpose of this clause, “banking company” includes


‘corresponding new bank’ as defined in clause (d) of section 2 of the Banking
Companies (Acquisition and Transfer of Undertakings) Act, 1970 (5 of 1970) and
clause (b) of section 2 of the Banking Companies (Acquisition and Transfer of
Undertakings) Act, 1980 (40 of 1980) and ‘subsidiary bank’ as defined in clause (k)
of section 2 of the State Bank of India (Subsidiary Bank) Act, 1959 (38 of 1959).”.
(insertion on page no 5.48 of SM.)
(iv) Punishment in case of Non-Compliance : (Insertion on Page no 5.50 of SM) If a
company or any officer of a company or an auditor or any other person contravenes
any of the provisions of NFRA Rules, the company and every officer of the company
who is in default or the auditor or such other person shall be punishable as per the
provisions of section 450 of the Act.
Further, as per Section 132(4) of the Companies Act, 2013 as amended by the
Companies Amendment Act, 2019 , National Financial Reporting Authority, where
professional or other misconduct is proved, have the power to make order for:
(A) imposing penalty of—
I. not less than one lakh rupees, but which may extend to five times of the
fees received, in case of individuals; and
II. not less than five lakh rupees, but which may extend to ten times of the
fees received, in case of firms;
(B) debarring the member or the firm from:
I. being appointed as an auditor or internal auditor or undertaking any audit
in respect of financial statements or internal audit of the functions and
activities of any company or body corporate; or
II. performing any valuation as provided under section 247, for a minimum
period of six months or such higher period not exceeding ten years as may
be determined by the National Financial Reporting Authority.

Chapter 7: Audit Committee and Corporate Governance


As per Stock and Exchange Board of India circular no. CIR/CFD/CMD1/114/2019 dated
18th October, 2019 on resignation of statutory auditors from listed entities and their
material subsidiaries:
1. Listed companies are required to make timely disclosures to investors in the securities
market for enabling them to take informed investment decisions.
2. Under Sub-clause (2) of Clause A in Part C of Schedule II under Regulation 18(3) of the
SEBI (Listing Obligations and Disclosure Requirements) Regulations, 2015 ("SEBI LODR
Regulations"), the Audit Committee of a listed entity, inter alia, has to make

© The Institute of Chartered Accountants of India


PAPER – 3 : ADVANCED AUDITING AND PROFESSIONAL ETHICS 53

recommendations for the appointment, remuneration and terms of appointment of auditors


of a listed entity. Under Sub-clause (7), the Audit Committee is also responsible for
reviewing and monitoring the independence and performance of auditors and the
effectiveness of the audit process.
3. Further, Sub-clause (7A) inserted under Clause A in Part A of Schedule Ill under
Regulation 30(2) of SEBI LODR Regulations requires detailed reasons to be disclosed by
the listed entities to the stock exchanges in case of resignation of the auditor of a listed
entity as soon as possible but not later than twenty-four hours of receipt of such reasons
from the auditor.
4. Regulation 36(5) of the SEBI LODR Regulations lays down certain disclosures to be made
part of the notice to the shareholders for an AGM, where the statutory auditors are
proposed to be appointed/re-appointed, including their terms of appointment.
5. Resignation of an auditor of a listed entity / its material subsidiary before completion of the
audit of the financial results for the year due to reasons such as pre-occupation may
seriously hamper investor confidence and deny them access to reliable information for
taking timely investment decisions.
6. In light of the above, the conditions to be complied with upon resignation of the statutory
auditor of a listed entity/material subsidiary w.r.t. limited review / audit report as per SEBI
LODR Regulations, are as under:
A. All listed entities/material subsidiaries shall ensure compliance with the
following conditions while appointing/re-appointing an auditor:
(i) If the auditor resigns within 45 days from the end of a quarter of a financial year,
then the auditor shall, before such resignation, issue the limited review/ audit
report for such quarter.
(ii) If the auditor resigns after 45 days from the end of a quarter of a financial year,
then the auditor shall, before such resignation, issue the limited review/ audit
report for such quarter as well as the next quarter.
(iii) Notwithstanding the above, if the auditor has signed the limited review/ audit
report for the first three quarters of a financial year, then the auditor shall, before
such resignation, issue the limited review/ audit report for the last quarter of such
financial year as well as the audit report for such financial year.
B. Other conditions relating to resignation shall include:
(i) Reporting of concerns with respect to the listed entity/its material subsidiary to
the Audit Committee:
a. In case of any concern with the management of the listed entity/material
subsidiary such as non-availability of information I non-cooperation by the
management which may hamper the audit process, the auditor shall
approach the Chairman of the Audit Committee of the listed entity and the

© The Institute of Chartered Accountants of India


54 FINAL (NEW) EXAMINATION: NOVEMBER, 2020

Audit Committee shall receive such concern directly and immediately


without specifically waiting for the quarterly Audit Committee meetings.
b. In case the auditor proposes to resign, all concerns with respect to the
proposed resignation, along with relevant documents shall be brought to
the notice of the Audit Committee. In cases where the proposed resignation
is due to non-receipt of information I explanation from the company, the
auditor shall inform the Audit Committee of the details of information I
explanation sought and not provided by the management, as applicable.
c. On receipt of such information from the auditor relating to the proposal to
resign as mentioned above, the Audit Committee I board of directors, as
the case may be, shall deliberate on the matter and communicate its views
to the management and the auditor.
(ii) Disclaimer in case of non-receipt of information: In case the listed entity/ its
material subsidiary does not provide information required by the auditor, to that
extent, the auditor shall provide an appropriate disclaimer in the audit report,
which may be in accordance with the Standards of Auditing as specified by ICAI/
NFRA.
The listed entity/ material subsidiary shall ensure that the conditions as mentioned in
6(A) and 6(B) above are included in the terms of appointment of the statutory auditor
at the time of appointing/re-appointing the auditor. In case the auditor has already
been appointed, the terms of appointment shall be suitably modified to give effect to
6(A) and 6(B) above.
The practicing company secretary shall certify compliance by a listed entity with 6(A)
and 6(B) above in the annual secretarial compliance report issued in terms of SEBI
Circular no. CIR/CFD/CMD1/27/2019 dated February 08, 2019.
C. Obligations of the listed entity and its material subsidiary:
(i) Format of information to be obtained from the statutory auditor upon resignation:
Upon resignation, the listed entity/ its material subsidiary shall obtain information
from the Auditor in the format as specified in Annexure A to this Circular. The
listed entity shall ensure disclosure of the same under Sub-clause (7A) of Clause
A in Part A of Schedule Ill under Regulation 30(2) of SEBI LODR Regulations.
(ii) Co-operation by listed entity and its material subsidiary: During the period from
when the auditor proposes to resign till the auditor submits the report for such
quarter I financial year as specified above, the listed entity and its material
subsidiaries shall continue to provide all such documents/information as may be
necessary for the audit I limited review.
(iii) Disclosure of Audit Committee's views to the Stock Exchanges: Upon
resignation of the auditor, the Audit Committee shall deliberate upon all the
concerns raised by the auditor with respect to its resignation as soon as

© The Institute of Chartered Accountants of India


PAPER – 3 : ADVANCED AUDITING AND PROFESSIONAL ETHICS 55

possible, but not later than the date of the next Audit Committee meeting and
communicate its views to the management. The listed entity shall ensure the
disclosure of the Audit Committee's views to the stock exchanges as soon a s
possible but not later than twenty-four hours after the date of such Audit
Committee meeting.
7. In case an entity is not mandated to have an Audit Committee, then the board of directors
of the entity shall ensure compliance of this circular.
8. The Stock Exchanges are advised to bring the provisions of this circular to the notice of all
listed entities and their material subsidiaries and also disseminate it on their websites.
9. In case the auditor is rendered disqualified due to operation of any condition mentioned in
Section 141 of the Companies Act, 2013, then the provisions of this Circular shall not apply.
10. The Circular is issued in exercise of the powers conferred under Section 11 (1) of the
Securities and Exchange Board of India Act, 1992 read with regulations 18(3), 30(2) and
36(5) of the SEBI LODR Regulations and shall be in addition to the provisions of
Companies Act, 2013. For more details visit: https://www.sebi.gov.in/legal/circulars/oct-
2019/resignation-of-statutory-auditors-from-listed-entities-and-their-material-subsidiaries_44703.html

Chapter 18: Professional Ethics

Joining/Association with “Networks” by Members in Practice


It is hereby clarified that associations with “Network” as a medium of referral of professional
work is permissible only if the Network is registered with the Institute, comprising only of
Chartered Accountants/ Chartered Accountant Firms, and governed by the Institute’ s Network
Guidelines, which may be accessed at https://resource.cdn.icai.org/24427announ14280.pdf
Members‟ attention is also drawn towards following provisions of Chartered Accountants Act,
1949 (hereinafter referred to as the “Act”) :
Clause (2) of Part I of First Schedule to the Act
A Chartered Accountant in practice shall be deemed to be guilty of professional misconduct, if
he pays or allows or agrees to pay or allow, directly or indirectly, any share, commission or
brokerage in the Fees or profits of his professional business, to any person other than a member
of the Institute or a partner or a retired partner or the legal representative of a deceased partner,
or a member of any other professional body or with such other persons having such
qualifications as may be prescribed, for the purpose of rendering such professional services
from time to time in or outside India.
Explanation — In this item, “partner” includes a person residing outside India with whom a
chartered accountant in practice has entered into partnership which is not in contravention of
item (4) of this Part;

© The Institute of Chartered Accountants of India


56 FINAL (NEW) EXAMINATION: NOVEMBER, 2020

Clause (3) of Part I of First Schedule to the Act


A Chartered Accountant in practice shall be deemed to be guilty of professional misconduct, if
he accepts or agrees to accept any part of the profits of the professional work of a person who
is not a member of the Institute:
Provided that nothing herein contained shall be construed as prohibiting a member from entering
into profit sharing or other similar arrangements, including receiving any share commission or
brokerage in the fees, with a member of such professional body or other person having
qualifications, as is referred to in item (2) of this Part;
Clause (5) of Part I of First Schedule to the Act
A Chartered Accountant in practice shall be deemed to be guilty of professional misconduct, if
he secures, either through the services of a person who is not an employee of such Chartered
Accountant or who is not his partner or by means which are not open to a chartered accountant,
any professional business.
Provided that nothing herein contained shall be construed as prohibiting any arrangement
permitted in terms of items (2), (3) and (4) of this Part;
Clause (6) of Part I of First Schedule to the Act
A Chartered Accountant in practice shall be deemed to be guilty of professional misconduct, if
he solicits clients or professional work either directly or indirectly by circular, advertisement,
personal communication or interview or by any other means:
Provided that nothing herein contained shall be construed as preventing or prohibiting —
(i) any chartered accountant from applying or requesting for or inviting or securing
professional work from another chartered accountant in practice; or
(ii) a member from responding to tenders or enquiries issued by various users of professional
services or organisations from time to time and securing professional work as a
consequence;
In view of the above provisions, it is not permissible for members in practice to join Networks
(by whatever name called) other than the Networks registered with the Institute.
Members may note that joining such Networks as mentioned above may result in noncompliance
of the above stated provisions of the Act resulting in disciplinary proceedings in accordance with
the provisions of the Act.
A - Meaning of Network & Network Firm –
Network - A larger structure (a) That is aimed at co-operation; and (b) That is clearly aimed at
profit or cost sharing or shares common ownership, control or management, common quality
control policies and procedures, common business strategy, the use of a common brand name,
or a significant part of professional resources.
Network Firm – “Network Firm” means a firm or Entity that belongs to a Network.

© The Institute of Chartered Accountants of India


PAPER – 3 : ADVANCED AUDITING AND PROFESSIONAL ETHICS 57

B - Concept of Network
1. To enhance their ability to provide professional services, firms frequently form larger
structures with other firms and entities. Whether these larger structures create a network
depends on the particular facts and circumstances and does not depend on whether the
firms and entities are legally separate and distinct. For example, a larger structure may be
aimed only at facilitating the referral of work, which in itself does not meet the criteria
necessary to constitute a network. Alternatively, a larger structure might be such that it is
aimed at co-operation and the firms share a common brand name, a common system of
quality control, or significant professional resources and consequently is deemed to be a
network.
2. The judgment as to whether the larger structure is a network shall be made in light of
whether a reasonable and informed third party would be likely to conclude, weighing all the
specific facts and circumstances, that the entities are associated in such a way that a
network exists. This judgment shall be applied consistently throughout the network.
3. Where the larger structure is aimed at co-operation and it is clearly aimed at profit or cost
sharing among the entities within the structure, it is deemed to be a network. However, the
sharing of immaterial costs does not in itself create a network. In addition, if the sharing of
costs is limited only to those costs related to the development of audit methodologies,
manuals, or training courses, this would not in itself create a network.
Further, an association between a firm and an otherwise unrelated entity to jointly provide
a service or develop a product does not in itself create a network.
4. Where the larger structure is aimed at cooperation and the entities within the structure
share common ownership, control or management, it is deemed to be a network. This could
be achieved by contract or other means.
5. Where the larger structure is aimed at co-operation and the entities within the structure
share common quality control policies and procedures, it is deemed to be a network. For
this purpose, common quality control policies and procedures are those designed,
implemented and monitored across the larger structure.
6. Where the larger structure is aimed at co-operation and the entities within the structure
share a common business strategy, it is deemed to be a network. Sharing a common
business strategy involves an agreement by the entities to achieve common strategic
objectives. An entity is not deemed to be a network firm merely because it co -operates
with another entity solely to respond jointly to a request for a proposal for the pro vision.
7. Where the larger structure is aimed at co-operation and the entities within the structure
share the use of a common brand name, it is deemed to be a network. A common brand
name includes common initials or a common name. A firm is deemed to be using a common
brand name if it includes, for example, the common brand name as part of, or along with,
its firm name, when a partner of the firm signs an audit report.

© The Institute of Chartered Accountants of India


58 FINAL (NEW) EXAMINATION: NOVEMBER, 2020

8. Even though a firm does not belong to a network and does not use a common brand name
as part of its firm name, it may give the appearance that it belongs to a network if it makes
reference in its stationery or promotional materials to being a member of an association of
firms.
Accordingly, if care is not taken in how a firm describes such memberships, a perception
may be created that the firm belongs to a network.
9. Where the larger structure is aimed at co-operation and the entities within the structure
share a significant part of professional resources, it is deemed to be a network.
Professional resources include:
• Common systems that enable firms to exchange information such as client data,
billing and time records;
• Partners and staff;
• Technical departments that consult on technical or industry specific issues,
transactions or events for assurance engagements;
• Audit methodology or audit manuals; and
• Training courses and facilities.
10. The determination of whether the professional resources shared are significant, and
therefore the firms are network firms, shall be made based on the relevant facts and
circumstances. Where the shared resources are limited to common audit methodology or
audit manuals, with no exchange of personnel or client or market information, it is unlikely
that the shared resources would be significant. The same applies to a common training
endeavor. Where, however, the shared resources involve the exchange of people or
information, such as where staff are drawn from a shared pool, or a common technical
department is created within the larger structure to provide participating firms with technical
advice that the firms are required to follow, a reasonable and informed third party is more
likely to conclude that the shared resources are significant.
C- Forms of the Network: The different forms of Network can be as under:-
1. A network can be constituted as a mutual entity which will act as a facilitator for the
constituents of the Network. In such a case the Network itself will not carry out any
professional practice.
2. A network can be constituted as a partnership firm subject to the condition that the total
number of partners does not exceed twenty.
3. A network can be constituted as a Limited Liability Partnership subject to the provision of
the Chartered Accountant Act and Rules and such other laws as may be applicabl e.
4. A network can be constituted as company subject to the guidelines prescribed by Institute
for corporate form of practice and formation of management consultancy services
company.

© The Institute of Chartered Accountants of India


PAPER – 3 : ADVANCED AUDITING AND PROFESSIONAL ETHICS 59

5. Network Firms shall consist of sole Practitioner/proprietor, partnership or any such entity
of professional accountants as may be permitted by the Act
6. A firm is allowed to join only one network.
7. Firms having common partners shall join only one Network.
D - Approval of Name of Network amongst firms registered with Institute:
1. The Network may have distinct name which should be approved by the Institute. To
distinguish a “Network” from a “firm” of Chartered Accountants, the words “& Affiliates”
shall be used after the name of the network and the words “& Co.” / “& Associates” shall
not be used. The prescribed format of application for approval of Name for Network is at
Form ‘A’ (enclosed). Illustrative examples of names of Network: -
(a) If the Network is a Mutual Entity or Partnership Firm: AB & Affiliates
(b) If the Network is a LLP: AB Affiliates LLP
(c) If the Network is a Limited Company: AB Affiliates P. Ltd/Limited
2. Provisions of Regulation 190 of the Chartered Accountants Regulations, 1988 shall be
applicable to the name of Network. However, even if a name is approved and subsequently
it is found that the same is undesirable then, the said name may be withdrawn at any time
by the Institute. The Institute shall reject any undesirable name and the provisions in
respect of names of companies as prescribed in the Companies Act, 1956 shall be
applicable in spirit.
3. The Institute shall approve or reject the name of the Network and intimate the same to the
Network at its address mentioned in Form ‘A’ within a period which shall not be later than
30 days from the date of receipt of the said Form.
4. Mere approval of the name of the Network shall not entitle the Network to carry on practice
in its own name.
E - Registration of Network with entities in India
1. After the name of a Network is approved as per provision under Guideline 5, the Institute
same shall reserve such name for a period of three (3) months from the date of approval.
2. The Network shall get itself registered with the Institute by applying in Form B within the
period of 3 months, failing which the name assigned shall stand cancelled on the expiry of
the said period.
3. Registration of Network with Institute is mandatory.
4. If different Indian firms are networked with a common Multinational Accounting Firm, they
shall be considered as a part of network.
F - Listing of Network with entities outside India
1. The duly authorized representative(s) of the Indian Member firm (s)/Member constituting
the Network with entities outside India shall file a declaration with the Ins titute in Form ‘D’

© The Institute of Chartered Accountants of India


60 FINAL (NEW) EXAMINATION: NOVEMBER, 2020

for Listing of such Network within 30 days from the date of entering into the Network
arrangement.
2. Proprietary/individual members, partnership firms as well as members in LLP or any such
other entity of members as may be permitted by the Act, shall be permitted to join such
network with entities outside India provided that the proprietary/individual members,
partnership firms as well as members in LLP or any such other entity of members are
allowed to join only one network and firms having common partners shall join only one
such network.
G - Change in constitution of registered Network: In case of change in the constitution of
registered Network on account of any entry into or exit from the Network, the network shall
communicate the same to the Institute by filing Form ‘C’ within a period of thirty (30) days from
the date of change in the constitution.
H - Ethical Compliance: Once the relationship of network arises, it will be necessary for such
a network to comply with all applicable ethical requirements prescribed by the Institute from time
to time in general and the following requirements in particular: -
1. If one firm of the network is the statutory auditor of an entity then the associate [including
the networked firm(s)] or the said firm directly/indirectly shall not accept the internal audit
or book-keeping or such other professional assignments which are prohibited for the
statutory auditor firm.
2. The guidelines of ceiling on Non-audit fees is applicable in relation to a Network as follows:
i) For a Network firm who is doing statutory audit (including its associate concern and/or
firm(s) having common partnership), it shall be the same as mentioned in the said
notification; and ii) For other firms of the same Network collectively, it shall be 3 times of
the fee payable for carrying out the statutory audit of the same undertaking/company.
3. In those cases where rotation of firms is prescribed by any regulatory authority, no member
firm of the network can accept appointment as an auditor in place of any member firm of
the network which is retiring.
4. The Network may advertise the Network to the extent permitted by the Advertisement
Guidelines issued by Institute. The firms constituting the network are permitted to use the
words “Network Firms” on their professional stationery.
5. The constituent member firms of a Network and the Network shall comply with all the
Ethical Standards prescribed by the Council from time to time.
I - Consent of Client: The effect of registration of network with Institute will be deemed to be a
public notice of the network and therefore consent of client will be deemed to be obtained.
J - Framework of Internal Byelaws of Network: To streamline the networking, a network shall
formulate operational bye-laws. Bye-laws may contain the following clauses on which the
affiliates of the network may enter into a written agreement among themselves:

© The Institute of Chartered Accountants of India


PAPER – 3 : ADVANCED AUDITING AND PROFESSIONAL ETHICS 61

(i) Appointment of a Managing Committee, from among the managing partners of the member
firms of the network and the terms and conditions under which it should function. The
minimum and maximum number of members of the Managing Committee shall also be
agreed upon.
(ii) Administration of the network
(iii) Contribution of membership fees to meet the cost of the administration of the network.
(iv) Identifying a partner of any of the member firms of the network to be responsible for the
assignment (engagement partner)
(v) Dispute settlement procedures through arbitration and conciliation
(vi) Development of training materials for members of the network
(vii) Issue of News-letters for staff and clients
(viii) Development of softwares for different types of assignments
(ix) Development and maintenance of data bases relevant for different types of assignments
(x) Library
(xi) Appointment of a technical director to whom references can be made
(xii) Determining the methodology for drawing resources from each member firm
(xiii) Determining compensation to member firms for resources to be drawn from them
(xiv) Peer review of the member firms These clauses are illustrative.
K - Repeal and Saving: The erstwhile “Rules/Guidelines of Network” issued by the Institute
stands repealed from the date of commencement of these Guidelines.
Provided that notwithstanding such repeal, anything done or any action taken or purported to
have been done or taken in respect of the erstwhile Rules/Guidelines prior to the date of
applicability of these Guidelines shall be deemed to have been done or take n under the
corresponding provisions of these Guidelines.
Note: Students are also advised to refer RTP of Paper 1 Financial Reporting (for AS, Ind AS
and other updates) and Paper 4 Part A -Corporate Laws (for academic updates relating to
Company Law).

© The Institute of Chartered Accountants of India


62 FINAL (NEW) EXAMINATION: NOVEMBER, 2020

PART – II : QUESTIONS AND ANSWERS

QUESTIONS
PART A: MULTIPLE CHOICE QUESTIONS
Integrated Case Scenario 1.
YS & Associates, a firm of Chartered Accountants, having CA. Y and CA. S as partners, is based
at Mumbai. YS & Associates get their website developed as www.ysassociates.com from KPY
Ltd. The colour of their website was very bright and attractive to run on a “push” technology.
Names of the partners of the firm and the major clients were also displayed on the web -site
without any disclosure obligation from any regulator.
CA. Y, accepted his appointment as tax auditor of a firm under Section 44AB, of the Income-tax
Act, and commenced the tax audit within two days of appointment since the client was in a hurry
to file Return of Income before the due date. After commencing the audit, CA. Y realised his
mistake of accepting this tax audit without sending any communication to the previous tax
auditor. In order to rectify his mistake, before signing the tax audit report, he sent a registered
post to the previous auditor and obtained the postal acknowledgement.
CA. S, provides management consultancy and other services to his clients. During 2019, looking
to the growing needs of his clients to invest in the stock markets, he also advised them on
Portfolio Management Services whereby he managed portfolios of some of his clie nts. Looking
at his expertise in financial management, Mr. Tarak, a student of Chartered Accountancy course,
is very much impressed with his knowledge. He approached CA. S to take guidance on some
topics of financial management subject related to his course. CA. S, on request, decided to
spare some time and started providing classes to Mr. Tarak along with some other aspirants for
3 days in a week and for 1 hours in a day. However, he has not taken any specific permission
for such private tutorship from the Council.
YS & Associates is appointed to conduct statutory audit of XYZ Ltd. XYZ Ltd is required to
appoint internal auditor as per statutory provisions given in the Companies Act, 2013 and
appointed CA. IA as its internal auditor. YS & Associates asked Mr. IA to provide direct
assistance to him regarding evaluating significant accounting estimates by the management and
assessing the risk of material misstatements. He also seeks his direct assistance in assembling
the information necessary to resolve exceptions in confirmation responses with respect to
external confirmation requests and evaluation of the results of external confirmation procedures.
XYZ Ltd is seeking advice of YS & Associates to appoint CA. IA for conducting GST Audit.

© The Institute of Chartered Accountants of India


PAPER – 3 : ADVANCED AUDITING AND PROFESSIONAL ETHICS 63

Question No.: (1-5)


1. YS & Associates sought direct assistance from CA. IA, internal auditor as stated in the
above scenario. Advise as to whether he is permitted to do so in accordance with relevant
Standards on Auditing.
(a) YS & Associates cannot ask CA. IA for direct assistance regarding evaluating
significant accounting estimates and assessing the risk of material misstatements.
However, CA. IA may assist YS & Associates in assembling information necessary to
resolve exceptions in confirmation responses as per SA 610.
(b) CA. IA cannot assist YS & Associates in assembling information necessary to resolve
exceptions in confirmation responses. However, YS & Associates can ask Mr. IA for
direct assistance regarding evaluating significant accounting estimates and
assessing the risk of material misstatements as per SA 610.
(c) YS & Associates cannot ask CA. IA for direct assistance regarding evaluating
significant accounting estimates and assessing the risk of material misstatements and
in assembling the information necessary to resolve exceptions in confirmation
responses as per SA 610.
(d) YS & Associates can ask CA. IA for direct assistance regarding evaluating significant
accounting estimates and assessing the risk of material misstatements and in
assembling the information necessary to resolve exceptions in confirmation
responses as per SA 610.
2. Whether CA S is guilty of professional misconduct in providing private tutorship to
Mr. Tarak along with some other aspirants for 3 days in a week and for 1 hours in a day in
the absence of specific approval.
(a) CA. S is not guilty of professional misconduct as he is teaching within prescribed
hours i.e. not exceeding 25 hours a month as per Regulation 192A
(b) CA. S is not guilty of professional misconduct as he is teaching within prescribed
hours i.e. not exceeding 25 hours a month as per Regulation 190A
(c) CA. S is guilty of professional misconduct as he has not obtained specific permission
for the same.
(d) CA. S is not guilty of professional misconduct as he is teaching within prescribed
hours i.e. not exceeding 25 hours a week as per Regulation 190A
3. Before signing the tax audit report, CA. Y sent a registered post to the previous auditor
and obtained the postal acknowledgement. Will CA. Y be held guilty of professional
misconduct under the Chartered Accountants Act, 1949?
(a) As per Clause (8) of Part I of First Schedule to the Chartered Accountants Act, 1949
CA. Y will not be held guilty of professional misconduct as he communicated with the
previous tax auditor before signing the audit report.

© The Institute of Chartered Accountants of India


64 FINAL (NEW) EXAMINATION: NOVEMBER, 2020

(b) As per Clause (8) of Part I of First Schedule to the Chartered Accountants Act, 1949,
CA. Y will not be held guilty of professional misconduct since the requirement for
communicating with the previous auditor being a chartered accountant in practice
would apply to statutory audit only.
(c) As per Clause (8) of Part I of First Schedule to the Chartered Accountants Act, 1949,
CA. Y will be held guilty of professional misconduct since he has accepted the tax
audit, without first communicating with the previous auditor in writing.
(d) As per Clause (8) of Part I of Second Schedule to the Chartered Accountants Act,
1949, CA. Y will be held guilty of professional misconduct since he has accepted the
tax audit, without first communicating with the previous auditor in writing.
4. In view of YS & Associates, whether CA. IA is eligible to undertake Goods and Service Tax
(GST) Audit of XYZ Ltd simultaneously?
(a) CA. IA is internal auditor of XYZ Ltd and therefore, is eligible to undertake Goods and
Service Tax (GST) Audit of XYZ Ltd simultaneously
(b) CA. IA is internal auditor of XYZ Ltd and therefore, not eligible to undertake Goods
and Service Tax (GST) Audit of XYZ Ltd simultaneously
(c) Being Internal Auditor CA. IA is appropriate person to carry out Goods and Service
Tax (GST) Audit of XYZ Ltd
(d) None of the above
5. Whether, website designed for www.ysassociates.com is in compliance with the guidelines
given in Clause (6) of Part I of First Schedule to the Chartered Accountants Act, 1949:
(a) Yes, website can have names of partners and major clients along with its fess.
(b) Yes, as the websites can be designed on a “push” technology.
(c) Yes, as there is no restriction on the colours used in the website.
(d) No, as names of the partners of the firm and the major clients were displayed without
any disclosure obligation from any Regulator.
Integrated Case Scenario 2.
M/s ANS & Associates have been appointed as statutory auditors of Delco Ltd., listed company
(referred to as ‘Company’) for the Financial Year 2019-20. Delco Ltd. deals in the manufacture
of shoes.
During the course of the audit, the auditor found that the company has acquired two subsidiaries
namely Sole Ltd. dealing in the manufacture of shoe soles and Soccer Ltd. dealing in the
manufacture of kid shoes. The auditors of Sole Ltd. and Soccer Ltd. are M/s XYZ & Associates.
Delco Ltd. has prepared the consolidated financial statements under Indian Accounting
Standards and consolidated the financial statements of subsidiary Sole Ltd. during the current
financial year. However, the company has not consolidated the financial statements of Soccer

© The Institute of Chartered Accountants of India


PAPER – 3 : ADVANCED AUDITING AND PROFESSIONAL ETHICS 65

Ltd. which was also acquired during the current financial year as the company has not yet been
able to ascertain the fair values of certain material assets and liabilities of Soccer Ltd. as on the
acquisition date. This acquisition is accounted for as an investment in th e books of Delco Ltd.
Had the company consolidated the financial statements of both the subsidiary, there would have
been material impact on important elements of the financial statements.
At the time of finalization of the Audit Report of the company for the year ended March 31, 2020,
the auditors are considering their reporting responsibility for non-consolidation of the financial
statements of Soccer Limited.
The auditors also asked the management to disclose their reason for non -consolidation of
financial of Soccer Ltd. in the notes to accounts.
Also the financial statements of Delco Ltd. of the current financial year include the corresponding
figures (without consolidation) of the previous financial year i.e. FY 2018 -19.
Further, the auditors are also considering the implications on their responsibilities and the
management’s responsibilities with respect to the financial statements and in the audit of such
financial statements.
Question No.: (6-10)
6. With respect to the non-consolidation of financial statements of Soccer Ltd. with the
financial statements of Delco Ltd., how should the auditor deal with the same in their audit
report?
(a) The auditor should give a disclaimer of opinion.
(b) The auditor should give an adverse opinion if the impact is material and pervasive in
his audit report.
(c) The auditor should mention this fact in the emphasis of matter paragraph pervasive
in his audit report.
(d) The auditor should mention this fact in other matter paragraph pervasive in his audit
report.
7. What are the auditors reporting responsibility with respect to the corresponding figures of
the financial year 2018-19 in the current year financial statements, for the same?
(a) The auditor’s opinion should refer to each period for which the financial statements
are presented.
(b) The auditors need to report on the current year financials only be it comparative or
corresponding figures.
(c) The auditor’s opinion shall not refer to the corresponding figures except if the previous
period audit report is other than an unqualified opinion or the auditor has sufficient
evidence that a material misstatement exist in the financial statement of prior period
which was not addressed earlier.

© The Institute of Chartered Accountants of India


66 FINAL (NEW) EXAMINATION: NOVEMBER, 2020

(d) The auditor has no reporting responsibility for the financial statements of any year
other than the current financial year for which they have been appointed.
8. What is the reporting responsibility of the auditor in case, the prior period financial
statements are not audited?
(a) The auditors need to report such matter in the Key Audit Matters paragraph in his
report.
(b) The auditors need to report such matter in the other matter paragraph in his report.
(c) The auditor will be responsible for obtaining sufficient appropriate audit evidence that
opening balance so not contain any material misstatement.
(d) Both b & c.
9. Preparation of the financial statements in accordance with the applicable financial reporting
framework is the responsibility of the management of Delco Ltd. Which of the following is
correct in regard to the disclosure of such management responsibility: -
(a) This is implied responsibility of management and is presumed in an audit of financial
statements and therefore need not be specifically mentioned anywhere.
(b) The management may undertake to accept such responsibility through an
engagement letter itself.
(c) The auditor report should describe the management responsibility in a section with
heading “responsibility of management for financial statements”.
(d) The auditor’s report should refer to the responsibility of auditors and not that of the
management as the same is obvious.
10. If the auditors of Delco Ltd. decides to give a qualified/ adverse opinion in the current
financial year with respect to the non-consolidation of financials of Soccer Ltd., which of
the following is true with regard to the use of EOM paragraph for some other matter: -
(a) The auditor cannot add EOM paragraph in his report on any matter as a qualified/
adverse opinion is given by the auditor.
(b) EOM paragraph on a matter can be added if auditors opinion is neither qualified/ nor
adverse in respect to that particular matter and the matter is fundamental to the user’s
understanding of financial statements.
(c) EOM paragraph on any matter can be added in the auditor’s report even if the report
is qualified/ adverse with respect to that particular matter.
(d) EOM paragraph indicates that the auditor’s opinion is modified in respect to the matter
emphasized.

© The Institute of Chartered Accountants of India


PAPER – 3 : ADVANCED AUDITING AND PROFESSIONAL ETHICS 67

Independent MCQs
11. NIC Chartered Accountants was appointed as statutory auditors by PNG Ltd. for the audit
of their financial statements. During the course of audit the auditors noticed a fraud of
` 101 lac committed by an officer of the Company. The officer sanctioned and made the
payment to fake vendors for purchase of fixed assets, however, the assets were not
entered in the Fixed Assets Register. The auditor reported the fraud in his audit report to
the shareholders of the Company presented in the Annual General Meeting, but did not
mention the name of the parties involved. The Board of Directors of the Company asked
ICAI to take necessary action against the auditor as he did not comply with his duty to
report fraud as per Section 143(12) of the Companies Act, 2013. What is the duty of the
auditor as per the Companies Act, 2013 in reporting the fraud committed by officers /
employees of the Company?
(a) As per the Companies Act, 2013, since the amount of fraud is more than ` 100 lac;
the auditor should have reported the matter within 2 days of his knowledge to the
Board of Directors/ Audit committee of the Company seeking their reply or
observations within 45 days. After completion of 45 days, the auditor should forward
his report to the Central Government along with the reply, if any, received from Board/
Audit Committee.
(b) As per the Companies Act, 2013, during the course of audit if the auditor has reason
to believe that a fraud has been committed by the officers or employees of the
Company, the auditor shall report the matter to the Central Government immediately.
(c) The auditor’s duty is restricted to reporting the fraud to shareholders and he is not
required to report the matter to the Board of Directors/ Audit Committee/ Central
Government.
(d) The auditor can submit his report on fraud to shareholders but is required to mention
the name of the parties involved in fraud, as per Section 143(12) of the Companies
Act, 2013.
12. Before concluding the audit, there was a difference of opinion between the audit committee
and the auditors as to which among the following are the areas which the auditor should
take into account to determine “Key Audit Matter” as per SA 701:
(I) The effect on audit of significant transactions that took place in the FY.
(II) Areas of high risk as assessed and reported by management’s expert.
(III) Significant auditor judgement relating to areas in the financials that involved
significant management judgement.
As per SA 701- Communicating Key audit matters in the Independent auditor’s Report,
which among the above-mentioned areas should CA & Co. take into account to determine
“Key Audit Matter”?
(a) (I) & (III)

© The Institute of Chartered Accountants of India


68 FINAL (NEW) EXAMINATION: NOVEMBER, 2020

(b) (II) only


(c) (I) & (II)
(d) (I), (II) & (III)
PART B : DESCRIPTIVE QUESTIONS
Standards on Auditing, Statements and Guidance Notes
13. (a) MEA Limited is a listed company having its operation across India. MEA Limited
appointed Mr. X, Mr. Y and Mr. Z, as its joint auditors for the year 2019-20. After
making sure that all of them are qualified to be appointed as statutory auditor, MEA
Limited issued engagement letter to all of them. But Mr. X was not clear on some
points, so he requested MEA Limited to slightly change the terms of his engagement.
This change will not impact the ultimate opinion on the financial statement. The
engagement letter contains the details on objective and scope of audit,
responsibilities of auditor and identification of framework applicable. It also contains
the reference to expected form and content of report from all three joint auditors. In
your opinion what was the discrepancy in the Audit engagement letter issued by MEA
Limited?
(b) ENN Limited is availing the services of APP Private Limited for its payroll operations.
Payroll cost accounts for 65% of total cost for ENN Limited. APP Limited has provided
the type 2 report as specified under SA 402 for its description, design and operating
effectiveness of control.
APP Private Limited has also outsourced a material part of payroll operation M/s SMP
& Associates in such a way that M/s SMP & Associates is sub-service organization
to ENN Limited. The Type 2 report which was provided by APP Private Limited was
based on carve-out method as specified under SA 402.
CA Raman while reviewing the unmodified audit report drafted by his assistant found
that, a reference has been made to the work done by the service auditor. CA Raman
hence asked his assistant to remove such reference and modify report accordingly.
Comment whether CA Raman is correct in removing the reference of the work done
by service auditor?
The Company Audit & Audit Report
14. (a) Petro Ltd. is engaged in generation of electricity for captive consumption through
Captive Generating Plant. The Company also maintains cost records in its books of
account as required under Cost Records and Audit Rules. Mr. Xylo, friend of Managing
Director of the Company, suggested name of his brother, who is a Cost Accountant in
Practice, for the purpose of cost audit. However, the statutory auditor of the company,
is of the view that the Company is not legally required to conduct cost audit. Now, the
Managing Director is in dilemma about the requirement of cost audit. Being an expert
in cost records and audit rules, you are required to guide in this regard.

© The Institute of Chartered Accountants of India


PAPER – 3 : ADVANCED AUDITING AND PROFESSIONAL ETHICS 69

(b) AKB Associates, a renowned audit firm in the field of CA practice for past two
decades. The firm was appointed to conduct statutory audit of Rica Ltd. an unlisted
company, which is engaged in the business of paper manufacturing. It decided to
commence the audit for the recently concluded financial year. Once after making
significant progress in the audit, the auditors made the following observations:
Observation 1: The management had disclosed in the financials that, during the
year, one of the warehouses of the Company was affected due to a major flood. As a
result of the same, the Company had incurred some losses. But the management was
of the view that it was not material.
Observation 2: Due to flood, few records maintained by the Company with respect
to a particular transaction was completely destroyed and there was no duplicate
record maintained by the Company. However, those details were not pervasive, but
material.
You are required to advise, whether AKB Associates should report Observation 1 and
2 in its audit report? If so, under which heading should it be reported?
Audit Committee and Corporate Governance
15. M/s FCA & Associates, Chartered Accountants is one of the leading auditing firms in
Guwahati. The firm received an assignment to examine the compliance conditions [as
stated in SEBI (LODR) Regulations] of corporate governance by ABC Ltd., a listed entity
with no outstanding SR equity shares. The firm had made the following observations:
Observation No. 1: Mr. Fine, one of the Director of the Company, also the Chairman of
the Stakeholder Relationship Committee, was acting as the audit committee Chairman in
4 other listed companies as well & 1 private company, simultaneously.
Observation No. 2: The Nomination & Remuneration Committee consisted of 6 members,
which regularly met biannually.
Observation No. 3: The Risk Management Committee consisted of 9 directors, out of
which, the number of independent directors is the majority, but it was less than two thirds
of the total strength.
Which among the above three observations made by the auditor of ABC Ltd. should be
reported by M/s. FCA & Associates?
Audit of Banks & Insurance Company
16. (a) ABN Bank was engaged in the business of providing Portfolio Management Services
to its customers, for which it took prior approval from RBI. Your firm has been
appointed as the statutory auditors of the Bank’s financial statements for the year
2019-20. Your senior has instructed you to verify the transactions of Portfolio
Management Services (PMS). While verifying the transactions you noticed that the
bank has not maintained separate record for PMS transactions from the Bank’s own
investments. As a statutory auditor what methodology will be adopted by you for

© The Institute of Chartered Accountants of India


70 FINAL (NEW) EXAMINATION: NOVEMBER, 2020

verification of PMS transactions?


(b) M/s MPS & Associates, Chartered Accountants started the statutory audit of their
client Contingencies Ltd., a General Insurance company, which has a paid-up capital
of ` 16,800/- lac. During the course of the audit, it was found that the Company was
not maintaining the required solvency margin as per the provisions of Insurance Act,
1938. When the issue was escalated to the management, they replied that solvency
margin needs to be maintained as per limits prescribed only on last day of the financial
year. Comment whether reply of management is tenable or not.
Audit under Fiscal Laws
17. (a) M/s PQRS & Associates is appointed for conducting tax audit as per Income Tax Act ,
1961 of QW Ltd., a cotton textile company. The Company had incurred ` 6 lac
towards advertisement expenditure on a brochure/ pamphlet published by a political
party in Pune. Advise the auditor whether such expenditure should be included in the
tax audit report or not.
(b) Mr. Ramanuj, a Chartered Accountant by profession, has been appointed as GST
auditor for ABC Ltd. The management has asked Mr. Ramanuj for GST audit and to
file GSTR-3B for the months of July and August 2019 and filing of annual return in
FORM GSTR-9. Mr. Ramanuj contended that he has been appointed only for GST
audit and the above are his scope limitations and cannot be conducted as the
compliances and returns are to be filed by the management. In context of above
dispute, you have to suggest whether the contention of Mr. Ramanuj is correct or not.
Justify.
Due Diligence, Investigation and Forensic Audit
18. APK Bank Ltd., received an application from a Pharmaceutical Company for takeover of
their outstanding term loans secured on its assets, availed from and outstanding with a
nationalised bank. APK Bank Ltd., requires you to conduct a due diligence audit in the
areas of assets of Pharmaceutical Company especially with reference to valuation aspect
of assets. State what may be your areas of analysis in order to ensure that the assets are
not stated at overvalued amounts.
Professional Ethics
19. Comment on the following with reference to the Chartered Accountants Act, 1949 and
schedules thereto:
(a) CA. Srishti and CA. Mishti are two partners of the CA firm ‘Srishti Mishti & Associates’.
Being very pious, CA. Srishti organised a religious ceremony at her home for which
she instructed her printing agent to add her designation “Chartered Accountant” with
her name in the invitation cards. Later on, the invitations were distributed to all the
relatives, close friends and clients of both the partners.

© The Institute of Chartered Accountants of India


PAPER – 3 : ADVANCED AUDITING AND PROFESSIONAL ETHICS 71

(b) Ms. Preeto, a CA, had an account with a bank. The normal balance in this account
remained at a level below ` 5,000. The bank inadvertently credited this account with
a cheque of ` 2,70,000 belonging to another account holder. When CA. Preeto came
to know about this she withdrew the amount of ` 2,75,000 and closed the bank
account. After 1 year the bank noticed the mistake and claimed ` 2,75,000 with
interest. CA. Preeto contested this claim. Can the bank approach the Institute of
Chartered Accountants of India for disciplinary action against CA. Preeto?
(c) CA. Moni is practicing since 2009 in the field of company audit. Due to her good
practical knowledge, she was offered editorship of a ‘Company Audit’ Journal which
she accepted. However, she did not take any permission from the Council regarding
such editorship.
20. Write a short note on the following:
(a) Selection of sample by the reviewer in case of peer review.
(b) Classification of frauds by NBFC.
(c) General steps in the conduct of risk based audit.
(d) Contents of an audit plan.

SUGGESTED ANSWERS/HINTS

PART A : ANSWERS TO MULTIPLE QUESTIONS


1. (a)
2. (d)
3. (c)
4. (b)
5. (d)
6. (b)
7. (c)
8. (d)
9. (c)
10. (b)
11. (a)
12. (a)

© The Institute of Chartered Accountants of India


72 FINAL (NEW) EXAMINATION: NOVEMBER, 2020

PART B
13. (a) Agreement on Audit Engagement Terms : As per SA 210, “Agreeing the Terms of
Audit Engagements”, the auditor shall agree the terms of the audit engagement with
management or those charged with governance, as appropriate.
Subject to prescribed details under Law or Regulations, the agreed terms of the audit
engagement shall be recorded in an audit engagement letter or other suitable form of
written agreement and shall include:
(i) The objective and scope of the audit of the financial statements;
(ii) The responsibilities of the auditor;
(iii) The responsibilities of management;
(iv) Identification of the applicable financial reporting framework for the preparation
of the financial statements; and
(v) Reference to the expected form and content of any reports to be issued by the
auditor and a statement that there may be circumstances in which a report may
differ from its expected form and content.
In the given scenario, MEA Limited appointed Mr. X, Mr. Y and Mr. Z, as its joint
auditors for the year 2019-20 and issued engagement letter to all of them. The
engagement letter contains the details on objective and scope of audit,
responsibilities of auditor, identification of framework applicable and reference to
expected form and content of report from all three joint auditors. However,
engagement letter issued by MEA Ltd. doesnot specify the responsibilities of
management, whereas as per SA 210, it should also specify responsibilities of
management.
(b) Reporting by the User Auditor: As per SA 402, “Audit Considerations Relating to
an Entity Using a Service Organisation”, the user auditor shall modify the opinion in
the user auditor’s report in accordance with SA 705, “Modifications to the Opinion in
the Independent Auditor’s Report”, if the user auditor is unable to obtain sufficient
appropriate audit evidence regarding the services provided by the service
organisation relevant to the audit of the user entity’s financial statements.
The user auditor shall not refer to the work of a service auditor in the user audit or’s
report containing an unmodified opinion unless required by law or regulation to do so.
If such reference is required by law or regulation, the user auditor’s report shall
indicate that the reference does not diminish the user auditor’s responsibility for the
audit opinion.
Thus, in view of above, contention of CA. Raman in removing reference of the work
done by service auditor is in order as in case of unmodified audit report, user auditor
cannot refer to the work done by service auditor.

© The Institute of Chartered Accountants of India


PAPER – 3 : ADVANCED AUDITING AND PROFESSIONAL ETHICS 73

14. (a) Applicability of Provisions related to Cost Records and Audit: The provisions
relating to cost records and audit are governed by section 148 of the Companies Act,
2013 read with the Companies (Cost Records and Audit) Rules, 2014.The audit
conducted under this section shall be in addition to the audit conducted under section
143.
Rule 3 of the Companies (Cost Records and Audit) Rules, 2014 provides the classes
of companies, engaged in the production of goods or providing services, required to
include cost records in their books of account.
However, the requirement for cost audit under these rules shall not be applicable to
a company which is covered under Rule 3, and,
(i) whose revenue from exports, in foreign exchange, exceeds 75 per cent of its
total revenue; or
(ii) which is operating from a special economic zone.
(iii) which is engaged in generation of electricity for captive consumption through
Captive Generating Plant.
In the given case, Petro Ltd. is engaged in generation of electricity for captive
consumption through Captive Generating Plant. Therefore, Petro Ltd. is not required
to conduct cost audit as it is falling under the exemption criteria. Hence, the opinion
of statutory auditor of the Company regarding non-applicability of cost audit is correct.
(b) Observation 1 - The management had disclosed in the financials that, during
the year, one of the warehouses of the Company was affected due to a major
flood. As a result of the same, the Company had incurred some losses. But the
management was of the view that it was not material: As per SA 706, “Emphasis
of Matter Paragraph & Other Matter Paragraph in the Independent Auditor’s Report”,
an Emphasis of Matter Paragraph refers to matter appropriately disclosed in the
financials, that in the auditor’s judgement is of such importance that it is fundamental
to users’ understanding of the financials. Hence, in this case, the auditor shall report
about the consequences of the flood which affected the Company’s warehouse under
Emphasis of Matter Paragraph.
Observation 2 - Due to flood, few records maintained by the Company with
respect to a particular transaction was completely destroyed and there was no
duplicate record maintained by the Company. However, those details were not
pervasive, but material: As per SA 705, “Modification to Opinion in the Independent
Auditor’s Report”, where the auditor is unable to obtain sufficient and appropriate
audit evidence and where such mater is material but not pervasive, the auditor shall
issue a qualified opinion.
Thus, in the given situation, on account of flood few records pertaining to particular
transactions was completely destroyed and in the absence of duplicate records, the
auditor was unable to obtain sufficient and appropriate audit evidence and those

© The Institute of Chartered Accountants of India


74 FINAL (NEW) EXAMINATION: NOVEMBER, 2020

details were material but not pervasive. Therefore, in accordance with SA 705, the
auditor is required to issue qualified opinion.
15. Observation No. 1 - Mr. Fine, one of the Director of the Company, also the Chairman
of the Stakeholder Relationship Committee, was acting as the Audit committee
Chairman as well in 4 other listed companies & 1 private Company, simultaneously:
As per Regulation 26 of SEBI (LODR) Regulations, a Director cannot be a Chairman in
more than 5 committees across all listed entities. However, for the purpose of reckoning
the limit under this Regulation, chairmanship of committees in a private company shall be
excluded. In this case, since Mr. Fine is the Chairman of audit committee in ABC Ltd. and
Chairman in 4 other listed companies, there is no violation of the limit specified under the
Regulation 26. Accordingly, this observation need not be reported by the auditor.
Observation No. 2: The Nomination & Remuneration Committee consisted of 6
members, who regularly met biannually: As per Regulation 19, Part D of Schedule II of
SEBI (LODR) Regulations, every listed company should have a Nomination &
Remuneration Committee, which shall meet at least once in a year. Since, in the given
case the committee met biannually (i.e. once in 2 years), the said observation needs
to be reported by the auditor.
Observation No. 3: The Risk Management committee consisted of 9 directors, out of
which the number of independent directors is the majority, but it was less than two
thirds of the total strength: As per Regulation 21 of SEBI (LODR) Regulations, only in
case of a listed entity having outstanding SR equity shares, at least two thirds of Risk
Management Committee shall comprise of independent directors. In the given case, ABC
Ltd. does not have outstanding SR equity shares. Accordingly, this observation need
not be reported by the auditor.
Thus, in the given scenario, only observation 2 will be reported.
16. (a) Separation of Investment Functions: The auditor needs to examine whether the
bank, as required by the RBI, is maintaining separate accounts for the i nvestments
made by it on their own Investment Account, PMS clients’ account, and on behalf of
other Constituents (including brokers). As per the RBI guidelines, banks are required
to get their investments under PMS separately audited by external auditors.
Thus, in the instant case, ABN Bank is required to prepare separate records for PMS
and as per RBI guidelines PMS investments need to be audited separately by the
external auditors and the auditors are required to give a certificate separately for the
same. So, in the above case the auditor should not verify the PMS transactions and
advise the bank to segregate the PMS transactions from its own investments and
provide the certificate of external auditor as described above. In case ABN Bank does
not provide the same the auditor may report accordingly.

© The Institute of Chartered Accountants of India


PAPER – 3 : ADVANCED AUDITING AND PROFESSIONAL ETHICS 75

(b) Maintenance of Solvency Margin: Section 64VA of the Insurance Act, 1938 as
amended by Insurance Laws (Amendment) Act, 2015 requires every insurer and re -
insurer to maintain an excess of the value of assets over the amount of liabilities at
all times which shall not be less than 50% of the amount of minimum capital as stated
under section 6 (requirement as to capital) of the Act and arrived at in the manner
specified by the regulations.
If, at any time, an insurer or re-insurer does not maintain the required control level of
solvency margin, he is required to submit a financial plan to the Authority indicating
the plan of action to correct the deficiency. If, on consideration of the plan, the
Authority finds it inadequate, the insurer has to modify the financial plan.
Sub-section (2) of section 64VA states that if an insurer or re-insurer fails to comply
with the prescribed requirement of maintaining excess of value of assets over amount
of liabilities, it shall deemed to be insolvent and may be wound up by the Court on an
application made by the authority.
Therefore, in the said case Contingencies Ltd has not maintained the Solvency
Margin throughout the year. Accordingly, contention of Contingencies Ltd. that
solvency margin is required to be maintained as per limits prescribed only on last day
of the financial year is not tenable.
17. (a) Expenses on Advertisement in the Media of a Political Party: In the given
situation, M/s PQRS & Associates is appointed for conducting tax audit as per Income
Tax Act, 1961 of QW Ltd., a cotton textile company. The Company had incurred ` 6
lac towards advertisement expenditure on a brochure/ pamphlet published by a
political party.
As per Clause 21(a) of Form 3CD, the auditor is required to furnish the details of
amounts debited to the Profit and Loss Account, being in the nature of advertisement
expenditure in any souvenir, brochure, tract, pamphlet or the like published by a
political party in his tax audit report.
Therefore, advertisement expenditure of ` 6 lac on brochure/pamphlet published by
a political party shall be reported in the tax audit report as per Clause 21 (a) of Form
3CD.
(b) Responsibility of Compliances and filing of Returns: GST auditor’s prime
responsibility on this engagement is limited to GST audit, audit of reconciliation
statement between books of accounts vis-a-vis GST returns prepared by the
Company. The GST auditor is, however, not responsible for any compliances like
uploading GST periodic returns for the relevant audit period.
Since, in the given situation, Management has asked the auditor Mr. Ramanuj to
conduct besides GST Audit filing of GSTR-3B for the months of July and August 2019
and filing of annual return in FORM GSTR-9.

© The Institute of Chartered Accountants of India


76 FINAL (NEW) EXAMINATION: NOVEMBER, 2020

Accordingly, Mr. Ramanuj has rightly refused that his scope is limited to GST Audit
and the scope does not cover any of the management functions.
In view of above, Contention of Management to Ramanuj is not tenable as
preparation of annual returns and its filing was the responsibility of management.
18. Over-Valued Assets : The areas of analysis in order to ensure that the assets are not
stated at overvalued amounts during conduct of Due Diligence are:
Uncollected/uncollectable receivables.
Obsolete, slow non-moving inventories or inventories valued above NRV; huge
inventories of packing materials etc. with name of company.
Underused or obsolete Plant and Machinery and their spares; asset values which
have been impaired due to sudden fall in market value etc.
Assets carried at much more than current market value due to capitalization of
expenditure/foreign exchange fluctuation, or capitalization of expenditure mainly in the
nature of revenue.
Litigated assets and property.
Investments carried at cost though realizable value is much lower.
Investments carrying a very low rate of income / return.
Infructuous project expenditure/deferred revenue expenditure etc.
Group Company balances under reconciliation etc.
Intangibles of no value.
19. (a) Printing of Designation “Chartered Accountant” on Invitations for Religious
Ceremony: As per Clause (6) of Part I of the First Schedule to the Chartered
Accountants Act, 1949, a Chartered Accountant in practice shall be deemed to be
guilty of professional misconduct if he solicits clients or professional work either
directly or indirectly by circular, advertisement, personal communication or interview
or by any other means.
However, the Council of the ICAI is of the view that the designation “Chartered
Accountant” as well as the name of the firm may be used in greeting cards, invitations
for marriages, religious ceremonies and any other specified matters, provided that
such greeting cards or invitations etc. are sent only to clients, relatives and close
friends of the members concerned.
In the given case, CA. Srishti has instructed to write designation “Chartered
Accountant” on invitation cards for a religious ceremony and distributed the same to
all the relatives, close friends and clients of both the partners.

© The Institute of Chartered Accountants of India


PAPER – 3 : ADVANCED AUDITING AND PROFESSIONAL ETHICS 77

In this context, it may be noted that the Council has allowed using designation
“Chartered Accountant” in invitations for religious ceremony, provided these are sent
to clients, relatives and close friends of the members concerned only.
Therefore, CA. Srishti would be held guilty of professional misconduct under the said
clause for sending such invitations to the relatives, close friends and clients of
CA. Mishti as well.
(b) Disrepute to the Profession: As per Clause 2 of Part IV of First Schedule of the
Chartered Accountant Act, 1949, a Chartered Accountant will be deemed to be guilty
of other misconduct if he in the opinion of the Council brings disrepute to the
profession or the Institute as a result of his action whether or not related to his
professional work.
In the instant case, CA. Preeto, a CA, had an account with a bank from which she
withdrew the amount of ` 2,75,000 and closed the account. This amount of ` 2,75,000
was pertaining to ` 5,000 minimum balance and ` 2,70,000 belonging to other
account holder and inadvertently credited to his account by the bank. The said act of
CA. Preeto to withdraw the money which does not belongs to her will bring disrepute
to the profession. Hence under this clause the bank can file a suitable complaint under
Clause 2 of Part IV of First Schedule of the Chartered Accountant Act, 1949 with the
Institute of Chartered Accountants of India.
(c) Permission from the Council: As per Clause (11) of Part I of First Schedule to the
Chartered Accountants Act, 1949, a Chartered Accountant in practice will be deemed
to be guilty of professional misconduct if he engages in any business or occupation
other than the profession of Chartered Accountant unless permitted by the Council
so to engage.
However, the Council has granted general permission to the members to engage in
certain specific occupation. In respect of all other occupations specific permission of
the Institute is necessary.
In the instant case, CA. Moni accepted editorship of a journal for which she did not
take any permission from the Council. In this context, it may be noted that the
editorship of professional journals is covered under the general permission and
specific permission is not required.
Therefore, CA. Moni shall not be held guilty of professional misconduct in terms of
Clause (11) of Part I of First Schedule to the Chartered Accountants Act, 1949.
20. (a) Selection of Sample by the Reviewer in case of Peer Review:
(i) The Reviewer shall within 15 days of receiving the information from the Practice
Unit select a sample of the assurance services that he would like to Review and
intimate the same to the Practice Unit.

© The Institute of Chartered Accountants of India


78 FINAL (NEW) EXAMINATION: NOVEMBER, 2020

(ii) The Reviewer may also seek further / additional clarification from the Practice
Unit on the information furnished / not furnished.
(iii) The Reviewer shall plan for an on–site Review visit or initial meeting in
consultation with the Practice Unit. The Reviewer shall give the Practice Unit at
least fifteen days’ time to keep ready the necessary records of the selected
assurance services.
(iv) The Reviewer and Practice Unit shall mutually cooperate and ensure that the
entire Review process is completed within 90 days from the date of notifying the
Practice Unit about its selection for Review.
(b) Classification of Frauds by NBFC: In order to have uniformity in reporting, frauds
have been classified as under based mainly on the provisions of the Indian Penal
Code:
(i) Misappropriation and criminal breach of trust.
(ii) Fraudulent encashment through forged instruments, manipulation of books of
account or through fictitious accounts and conversion of property.
(iii) Unauthorised credit facilities extended for reward or for illegal gratification.
(iv) Negligence and cash shortages.
(v) Cheating and forgery.
(vi) Irregularities in foreign exchange transactions.
(vii) Any other type of fraud not coming under the specific heads as above.
Cases of ‘negligence and cash shortages’ and ‘irregularities in foreign
exchange transactions’ referred to in items (d) and (f) above are to be reported as
fraud if the intention to cheat/ defraud is suspected/ proved. However, the following
cases where fraudulent intention is not suspected/ proved, at the time of detection,
will be treated as fraud and reported accordingly:
(I) cases of cash shortages more than ` 10,000/- and
(II) cases of cash shortages more than ` 5000/- if detected by management/ auditor/
inspecting officer and not reported on the occurrence by the persons handling
cash.
(c) General Steps in the Conduct of Risk Base Audit: RBA consists of four main
phases starting with the identification and prioritization of risks, to the determination
of residual risk, reduction of residual risk to acceptable level and the reporting to
auditee of audit results. These are achieved through the following:
Step 1 Understand auditee operations to identify and prioritize risks:
Understanding auditee operations involves processes for reviewing and
understanding the audited organization’s risk management processes for its

© The Institute of Chartered Accountants of India


PAPER – 3 : ADVANCED AUDITING AND PROFESSIONAL ETHICS 79

strategies, framework of operations, operational performance and information


process framework, in order to identify and prioritize the error and fraud risks that
impact the audit of financial statements. The environment in which the auditee
operates, the information required to monitor changes in the environment, and the
process or activities integral to the audited entity’s success in meeting its objectives
are the key factors to an understanding of agency risks. Likewise, a performance
review of the audited entity’s delivery of service by comparing expectations against
actual results may also aid in understanding agency operations.
Step 2 Assess auditee management strategies and controls to determine
residual audit risk: Assessment of management risk strategies and controls is the
determination as to how controls within the auditee are designed. The role of internal
audit in promoting a sound accounting system and internal control is recognized, thus
the SAI should evaluate the effectiveness of internal audit to determine the extent to
which reliance can be placed upon it in the conduct of substantive tests.
Step 3 Manage residual risk to reduce it to acceptable level: Management of
residual risk requires the design and execution of a risk reduction approach that is
efficient and effective to bring down residual audit risk to an acceptable level. This
includes the design and execution of necessary audit procedures and substantive
testing to obtain evidence in support of transactions and balances. More resources
should be allocated to areas of high audit risks, which were earlier known through the
analytical procedures undertaken.
Step 4 Inform auditee of audit results through appropriate report: The results of
audit shall be communicated by the auditor to the audited entity. The auditor must
immediately communicate to the auditee reportable conditions that have been
observed even before completion of the audit, such as weaknesses in the int ernal
control system, deficiencies in the design and operation of internal controls that affect
the organization’s ability to record, process, summarize and report financial data.

Assess auditee
Understand management Manage Inform auditee
auditee strategies and residual risk to of audit results
operations to controls to reduce it to through
identify and determine acceptable appropriate
prioritize risks residual audit level report
risk

(d) Contents of an Audit Plan: The auditor shall develop an audit plan that shall include
a description of-
(i) The nature, timing and extent of planned risk assessment procedures, as
determined under SA 315 “Identifying and Assessing the Risks of Material
Misstatement through Understanding the Entity and Its Environment”.

© The Institute of Chartered Accountants of India


80 FINAL (NEW) EXAMINATION: NOVEMBER, 2020

(ii) The nature, timing and extent of planned further audit procedures at the
assertion level, as determined under SA 330 “The Auditor’s Responses to
Assessed Risks”.
(iii) Other planned audit procedures that are required to be carried out so that the
engagement complies with SAs.
The audit plan is more detailed than the overall audit strategy that includes the nature,
timing and extent of audit procedures to be performed by engagement team members.
Planning for these audit procedures takes place over the course of the audit as the
audit plan for the engagement develops. For example, planning of the auditor's risk
assessment procedures occurs early in the audit process. However, planning the
nature, timing and extent of specific further audit procedures depends on the outcome
of those risk assessment procedures. In addition, the auditor may begin the execution
of further audit procedures for some classes of transactions, account balances and
disclosures before planning all remaining further audit procedures.

© The Institute of Chartered Accountants of India


PAPER – 3 : ADVANCED AUDITING AND PROFESSIONAL ETHICS
PART – I ACADEMIC UPDATE
(Legislative Amendments / Notifications / Circulars / Rules / Guidelines issued by
Regulating Authority)

CHAPTER 9 AUDIT OF BANKS


The Ministry of Corporate Affairs has further to amended the Companies (Audit and Auditors)
Rules, 2014, through the Companies (Audit and Auditors) Amendment Rules, 2021 vide
notification S.O. 206(E) dated 24 th March, 2021.
As per reporting requirements cast through Rule 11 of the Companies (Audit and Auditors)
Rules, 2014 the auditor’s report shall also include their views and comments on the following
matters, namely:
(1) whether the bank has disclosed the impact, if any, of pending litigations on its financial
position in its financial statement;
(2) whether the bank has made provision, as required under any law or accounting standards,
for material foreseeable losses, if any, on long term contracts including derivative
contracts;
(3) whether there has been any delay in transferring amounts, required to be transferred, to
the Investor Education and Protection Fund by the bank.
(4) (i) Whether the management has represented that, to the best of it’s knowledge
and belief, other than as disclosed in the notes to the accounts, no funds have
been advanced or loaned or invested (either from borrowed funds or share
premium or any other sources or kind of funds) by the banks to or in any other
person(s) or entity(ies), including foreign entities (“Intermediaries”), with the
understanding, whether recorded in writing or otherwise, that the Intermediary
shall, whether, directly or indirectly lend or invest in other persons or entities
identified in any manner whatsoever by or on behalf of the bank (“Ultimate
Beneficiaries”) or provide any guarantee, security or the like on behalf of the
Ultimate Beneficiaries;
(ii) Whether the management has represented, that, to the best of it’s knowledge
and belief, other than as disclosed in the notes to the accounts, no funds have
been received by the bank from any person(s) or entity(ies), including foreign
entities (“Funding Parties”), with the understanding, whether recorded in
writing or otherwise, that the bank shall, whether, directly or indirectly, lend or
invest in other persons or entities identified in any manner whatsoever by or on
behalf of the Funding Party (“Ultimate Beneficiaries”) or provide any guarantee,
security or the like on behalf of the Ultimate Beneficiaries; and

© The Institute of Chartered Accountants of India


2 FINAL (NEW) EXAMINATION: NOVEMBER, 2021

(iii) Based on such audit procedures that the auditor has considered reasonable
and appropriate in the circumstances, nothing has come to their notice that has
caused them to believe that the representations under sub-clause (i) and (ii)
contain any material mis-statement.
(5) Whether the dividend declared or paid during the year by the bank is in compliance
with section 123 of the Companies Act, 2013.
(6) [Whether the bank, in respect of financial years commencing on or after the 1st April,
2022,] has used such accounting software for maintaining its books of account
which has a feature of recording audit trail (edit log) facility and the same has been
operated throughout the year for all transactions recorded in the software and the
audit trail feature has not been tampered with and the audit trail has been preserved
by the company as per the statutory requirements for record retention.]
(Note: Students are required to refer Page no. 9.48 of Audit of Banks )

UNIT 1 OF CHAPTER 12: AUDIT UNDER FISCAL LAWS


Revision of Tax Audit Report [Notification No.28/2021 dated 1.4.2021]
1. Section 44AB lays obligation on certain persons mentioned thereunder carrying on
business or profession, to get their accounts audited before the “specified date” by a
Chartered Accountant, if their turnover exceeds the stipulated threshold or in cases where
they are eligible to declare income on presumptive basis, if they claim that their income is
lower than the income so computed.
These persons have to furnish by the specified date, a report of the audit in the prescribed
form. For this purpose, the CBDT has prescribed under Rule 6G, Forms 3CA/ 3CB and
Form 3CD.
The CBDT has, vide this notification, amended Rule 6G to provide that the audit report
furnished may be revised by the person by getting revised report of audit from a chartered
accountant, duly signed and verified by such chartered accountant, if there is payment by
such person after furnishing of report which necessitates recalculation of disallowance
under section 40 or section 43B. The said revised audit report has to be furnished before
the end of the relevant assessment year for which the report pertains.
2. In Form 3CD:
(i) In PART – A for clause 8A, the following clause shall be substituted, namely: - ―
8A Whether the assessee has opted for taxation under section
115BA/115BAA/115BAB/ 115BAC/ 115BAD?

© The Institute of Chartered Accountants of India


PAPER – 3 : ADVANCED AUDITING AND PROFESSIONAL ETHICS 3

(ii) In PART-B, for clause 17, the following clause shall be substituted, namely: -
Clause 17. Where any land or building or both is transferred during the previous year
for a consideration less than value adopted or assessed or assessable by any
authority of a State Government referred to in section 43CAor 50C, please
Details of Consideration Value Whether provisions of second
property received or adopted or proviso to subsection (1) of section
accrued assessed 43CA or fourth proviso to clause (x)
or of sub-section (2) of section 56
assessable applicable? [Yes/No] .’’;
(iii) In PART-B, in clause 18, for sub-clauses (ca) and (cb), the following sub-clauses,
shall be substituted namely:-
“(ca) Adjustment made to the written down value under section 115BAC/115BAD (for
assessment year 2021-2022 only)……
(cb) Adjustment made to written down value of Intangible asset due to excluding value
of goodwill of a business or profession…..
(cc) Adjusted written down value……….”;
(iv) In PART-B, for in clause 32, for sub-clause (a), the following sub-clause shall be
substituted, namely:-
(a) Details of brought forward loss or depreciation allowance, in the following
manner, to the extent available:
Serial Assess- Nature of Amount All Amount as adjusted Amounts Remarks
Number ment loss/ as losses/allowan by withdrawal of as
Year allowance returned* ces not additional assessed
(in rupees) (in allowed under depreciation on (give
rupees) section account of opting for reference
115BAA/ taxation under to relevant
115BAC/ section order)
115BAD 115BAC/115BAD^
(1) (2) (3) (4) (5) (6) (7) (8)
Note: *If the assessed depreciation is less and no appeal pending then take
assessed.
^To be filled in for assessment year 2021-2022 only.’’:
(v) In PART-B, clause 36 shall be omitted.

© The Institute of Chartered Accountants of India


4 FINAL (NEW) EXAMINATION: NOVEMBER, 2021

Students are required to refer updated chapters applicable for


November 2021 examination on the below mentioned link:
Chapter. Chapter Name Link
No.
Chapter 5 The Company Audit https://resource.cdn.icai.org/65691bos53002cp5
.pdf
Chapter 11 Audit of Non-Banking https://resource.cdn.icai.org/65692bos53002cp1
Financial Companies 1.pdf
Chapter 13 Audit of Public Sector https://resource.cdn.icai.org/65693bos53002cp1
Undertakings 3.pdf
Chapter 14 Liabilities of Auditors https://resource.cdn.icai.org/65694bos53002cp1
4.pdf

Note: Students are also advised to refer RTP of Paper 1 Financial Reporting (for AS, Ind AS
and other updates) and Paper 4 Part I -Corporate Laws (for academic updates relating to
Company Law).

PART – II : QUESTIONS AND ANSWERS

QUESTIONS
PART A: MULTIPLE CHOICE QUESTIONS
Integrated Case Scenario 1
Kiwspack Ltd. is an unlisted public company incorporated in the year 2009, having 90
shareholders with an equity share capital of ` 27 lakhs. There are total four directors in its
board.
For the financial year 2020-21 as well as for the quarter ended on 30 th June, 2021, Kiwspack
Ltd. had suffered a loss. Despite of such loss, the board of Kiwspack Ltd. declared a total interim
dividend of ` 20 lakhs for quarter ended March, 2021 on 25 th April, 2021.
The details of dividends declared by Kiwspack Ltd. during preceding financial years are
tabulated, as below:-
Financial Year Rate of Dividend Declared
2019-20 12%
2018-19 16%

© The Institute of Chartered Accountants of India


PAPER – 3 : ADVANCED AUDITING AND PROFESSIONAL ETHICS 5

2017-18 10%
2016-17 15%
2015-16 20%
2014-15 14%
The said dividend was paid to the shareholders on 10 th June, 2021, through account payee
cheque, by withdrawing an amount of 5% from the total free reserves available with Kiwspack
Ltd. The balance of free reserves after such withdrawal fell to 20% of its paid up share capital
as appearing in the latest audited financial statements.
One of the shareholders, Mr. Mahesh, had submitted a transfer deed to the company on 28 th
April, 2021, for registration relating to transfer of all shares held by him in Kiwspack Ltd. in the
name of Mr. Govardhan, along with an authorization letter for paying the amount of dividend on
his shares to Mr. Govardhan.
However, till 10 th June, 2020, due to certain reasons, Kiwspack Ltd. could not register the
aforesaid transfer of shares in the name of Mr. Govardhan.
The dividend remaining unpaid of ` 2 lakhs was transferred to the unpaid dividend account by
the company on 15 th June, 2021.
Kiwspack Ltd. prepared a statement on 30 th September, 2021, containing the names of
shareholders to whom payment of dividend had remained pending, their last known addresses
and the amount of dividend to be paid to them. The said statement was placed on the same
date on the company’s website and also on the website approved by the Central Government
for this purpose.
Rao & Co. is the statutory auditor of Kiwspack Ltd. for F.Y. 2020-21 which issued its audit report
on 30th June, 2021 on the financial statements approved on 20 th June, 2021.
On the basis of the abovementioned facts, you are required to answer the following MCQs:
Question No.: (1-5)
1. At what maximum rate, the board of Kiwspack Ltd. would have declared the interim
dividend for quarter ended March, 2021?
(a) 10.6%.
(b) 12.67%.
(c) 14.5%.
(d) 15%.
2. How much amount of interest shall be payable by Kiwspack Ltd. for delay in payment of
dividend to the shareholders?
(a) ` 13,151.

© The Institute of Chartered Accountants of India


6 FINAL (NEW) EXAMINATION: NOVEMBER, 2021

(b) ` 8,877.
(c) ` 10,521.
(d) ` 15,781.
3. In which account, Kiwspack Ltd. would have transferred the dividend amount in relation to
shares which were held by Mr. Mahesh?
(a) Account of Mr. Mahesh.
(b) Account of Mr. Govardhan.
(c) Unpaid Dividend Account.
(d) Investor Education and Protection Fund.
4. How much maximum amount of fine could be levied on every director of the company who
was knowingly a party to the default in payment of dividend to the shareholders?
(a) ` 9,000.
(b) ` 11,000.
(c) ` 16,000.
(d) ` 1,00,000.
5. By what date, the unpaid or unclaimed dividend amount should have been transferred to
Unpaid Dividend Account and also by what date, the statement in relation to details of such
Unpaid Dividend should have been prepared by Kiwspack Ltd. and placed on its website?
(a) 01st June, 2021 and 13 th September, 2021, respectively.
(b) 25th May, 2021 and 15 th July, 2021, respectively.
(c) 01st June, 2021 and 15 th July, 2021, respectively.
(d) 25th May, 2021 and 13 th September, 2021, respectively.
Integrated Case Scenario 2
Ulip Ltd. is a public company listed on the National Stock Exchange since the year, 2015, with
share capital of ` 150 crore.
SRS & Co. is being appointed as its statutory auditor for F.Y. 2020-21 and Mr. Raj is appointed
as the engagement partner, on behalf of the firm, to conduct the said audit assignment including
conducting of limited reviews and other statutory assignments.
Mr. Raj was conducting limited review for second quarter and during the same while adhering
to the responsibilities as conferred upon by SA 250, “Consideration of Laws and Regulations in
an Audit of Financial Statements”, he evaluated the implications of non-compliance in relation
to other aspects of the audit, including the auditor’s risk assessment and the reliability of written
representations and concluded that withdrawal from engagement was necessary in the given

© The Institute of Chartered Accountants of India


PAPER – 3 : ADVANCED AUDITING AND PROFESSIONAL ETHICS 7

circumstances, after seeking legal advice, even though the non-compliance was not material to
the financial statements but as the management or those charged with governance refrained
from taking the remedial action that he considered appropriate in the circumstances. Such a
withdrawal was not prohibited by any law or regulation.
Mr. Raj, on behalf of SRS & Co., brought to the notice of the Audit Committee of Ulip Ltd., all
his concerns with respect to the proposed resignation, along with relevant documents.
After issuing the necessary reports, as required in the circumstances, SRS & Co gave its
resignation letter to Ulip Ltd. at 1:00 p.m. on 20 th November, 2021 vide its official email-id, which
contained the detailed reasons for such resignation.
Such a letter was forwarded to the stipulated authority by Ulip Ltd. at 4:00 p.m. on 21 st
November, 2021 vide its official email-id.
SRS & Co. filed the statement with respect to its resignation as a statutory auditor in prescribed
form with Ulip Ltd. and the Registrar on 15 th January, 2021, respectively, after receiving a notice
from MCA.
For the purpose of filling the casual vacancy in the office of auditor, the Audit Committee of Ulip
Ltd. gave recommendation of an audit firm for being appointed as the statutory auditor to which
the Board disagreed and it referred back the recommendation to the committee for
reconsideration citing reasons for such disagreement.
However, the Audit Committee, after considering the reasons given by the Board, decided not
to reconsider its original recommendation, so, the Board of Ulip Ltd. after recording the reasons
for its disagreement with the committee appointed Chavda & Co. as its new statutory auditor on
15th December, 2021.
Such an appointment of Chavda & Co. was also approved by the members of Ulip Ltd. at a duly
convened general meeting on 3 rd February, 2022.
On the basis of the abovementioned facts, you are required to ans wer the following MCQs:
Question No.: (6-10)
6. Whether the reasons for withdrawal from the engagement by SRS & Co. can be considered
to be justifiable in the light of the fact that the non-compliance was not material to the
financial statements?
(a) Yes, as such a withdrawal was not prohibited by any law or regulation.
(b) Yes, as the auditor had obtained legal advice for the same and also such a withdrawal
was not prohibited by any law or regulation.
(c) Yes, in exceptional cases, the auditor may consider for such withdrawal provided that
such a withdrawal is not prohibited by any law or regulation.
(d) Yes, as it does not matter whether non-compliance is material or not, management
or those charged with governance should not refrain from taking the remedial action

© The Institute of Chartered Accountants of India


8 FINAL (NEW) EXAMINATION: NOVEMBER, 2021

which the auditor has considered necessary, provided that such a withdrawal is not
prohibited by any law or regulation.
7. In continuation of Question no. 6, above, if it is assumed that the auditor was prohibited by
any law or regulation from such withdrawal from engagement, then how he would have
reported the non-compliance in the audit report?
(a) In the “Basis for Qualified Opinion” paragraph.
(b) In the Other Matter(s) paragraph.
(c) In the Emphasis of Matter(s) paragraph.
(d) In the “Basis for Disclaimer of Opinion” paragraph.
8. Ulip Ltd. was required to disclose to which authority, the detailed reasons for resignatio n
of the auditor and by what time limit as per LODR 2015?
(a) Such reasons were required to be disclosed to MCA till 1:00 p.m. – 21st November,
2021.
(b) Such reasons were required to be disclosed to NSE & SEBI till 1:00 p.m. – 23rd
November, 2021.
(c) Such reasons were required to be disclosed to NSE till 1:00 p.m. – 21st November,
2021.
(d) Such reasons were required to be disclosed to the Registrar till 1:00 p.m. – 22nd
November, 2021.
9. What could be the penalty specified under the Company Act, 2013 that could be levied
upon SRS & Co. for failure in filing the statement with respect to its resignation, within the
prescribed time limit, with Ulip Ltd. and the Registrar, respectively, if its remuneration was
` 40,000?
(a) ` 62,500.
(b) ` 50,000.
(c) ` 40,000.
(d) ` 52,500.
10. What was the last date available with board of Ulip Ltd. for filing the casual vacancy in the
office of the auditor and by what last date, the general meeting for approving the auditor
as appointed by the board should have been made in accordance with the provisions of
the Companies Act, 2013?
(a) 27th November, 2021 and 27 th February, 2022.

© The Institute of Chartered Accountants of India


PAPER – 3 : ADVANCED AUDITING AND PROFESSIONAL ETHICS 9

(b) 20th December, 2021 and 15 th February, 2022.


(c) 20th January, 2021 and 20 th April, 2022.
(d) 20th December, 2021 and 15 th March, 2022.
PART B : DESCRIPTIVE QUESTIONS
Standards on Auditing, Statements and Guidance Notes
11. The audit report of Kolsi (P) Ltd. for F.Y. 2020-21 was issued by Bishnoi & Co. on 25 th July,
2021. However, a case was filed against Kolsi (P) Ltd. on 4 th August, 2021, with the Civil
Court, with respect to an incident caused in its factory on 17 th January, 2021, the outcome
of which may result in paying heavy penalty by Kolsi (P) Ltd.
Mr. Raj Bishnoi, the partner of Bishnoi & Co., discussed the said matter with the
management and it was determined to amend the financial statements for F.Y. 2020 -21.
Further, Mr. Raj inquired how the management intended to address the said matter in the
financial statements to which he was told that the said matter was going to be disclosed
as a “Contingent Liability for a Court case” to the foot note in the balance sheet with no
additional disclosures.
The management told Mr. Raj that such disclosure was enough as he would further going
a description of the said court case and its outcome in the ‘Emphasis of Matter’ paragraph
in his amended audit report.
In the context of aforesaid case scenario, please answer the following questions:-
(a) Whether Mr. Raj on behalf of Bishnoi & Co., has properly adhered to his
responsibilities in accordance with SA 560, on becoming aware of the court case filed
against Kolsi (P) Ltd.?
(b) Whether the contention of management of Kolsi (P) Ltd. is valid with respect to the
disclosure of the court case in the financial statements?
12. While formulating the audit plan and responding to the risks of material misstatement
identified and assessed in related party transaction and relationships, Ms. K the
engagement manager of the audit team of ABC Limited, decided to rely upon the internal
controls placed for identification and disclosure of related party relationships and
transactions in accordance with the applicable financial reporting framework.
You are requested to guide Ms. K regarding the necessity to test the controls to obtain
sufficient and appropriate audit evidence. Also guide, whether Ms. K can use the audit
evidence obtained, regarding operative effectiveness of control on identification and
disclosure of related party relationships and transactions, in the interim period.

© The Institute of Chartered Accountants of India


10 FINAL (NEW) EXAMINATION: NOVEMBER, 2021

The Company Audit & Audit Report


13. AB & Co. were appointed auditors for NOME Limited, a listed company, for the term of two
five consecutive years from 2010-11, 2011-12, 2012-13, 2013-14, 2014-15, 2015-16,
2016-17, 2017-18, 2018-19, 2019-20. As per the provision of the section 139(2)(b) “No
listed company or a company belonging to such class or classes of companies as may be
prescribed, shall appoint or re-appoint an audit firm as auditor for more than two terms of
five consecutive years”.
Hence, Management of NOME Limited reached out (based on the recommendation of Audit
Committee) to BCD & Co. for their nomination as the appointment of Statutory Auditor for
the financial year 2020-21. However, BCD & Co. did not provide any written consent to
such appointment neither they provided a certificate that the appointment, if made, shall
be in accordance with the conditions laid in the Act and Rules therein.
Still the management went ahead and proposed an appointment in AGM and BCD & Co.
were appointed as an auditor for the financial year 2020-21. Post appointment, those
charged with governance identified that majority of the partners in the BCD & Co. are same
which were there in AB & Co. Now, fearing the contravention of the provision of Companies
Act, 2013. Management, on guidance of those charged with governance, decided to file a
complaint with tribunal under section 140(5) of the Companies Act against statutory
auditors.
You are required to guide the BCD & Co. regarding the contravention of the provisions of
the Companies Act, 2013 with respect to appointment of Auditor.
14. Mr. Hemant Ramsey was appointed as the engagement partner for conducting the audit of
Kshetra Lap Ltd. for F.Y. 2020-21, on behalf of Ramsey & Associates. Mr. Vishay Tyagi
was appointed as the engagement quality control reviewer by the firm for th e said audit.
During F.Y. 2020-21, there was an implementation of ERP system in a phased manner, in
Kshetra Lap Ltd. due to which some of its business processes got automated. As a result
of the implementation of such a system, there was a significant effect on the auditor’s
overall audit strategy. Mr. Hemant discussed the implementation of such a system with Mr.
Vishay and also told him that such a matter may be a key audit matter to be reported in
the audit report.
Mr. Vishay considered the significance of such matter but however he was of the opinion
that such a matter did not appear to link with the matters disclosed in the financial
statements and so there was no need to disclose such matter as a key audit matter.
Whether the contention of Mr. Vishay is proper with respect to the matters to be
communicated as a key audit matter?

© The Institute of Chartered Accountants of India


PAPER – 3 : ADVANCED AUDITING AND PROFESSIONAL ETHICS 11

Audit Committee and Corporate Governance


15. Kayask Ltd. is a public company which got listed on BSE and NSE in the F.Y. 2015-16 and
is amongst the top 500 listed entities on the basis of market capitalization. JP Bhuj & Co.,
a CA firm, has been appointed as its statutory auditor for the F.Y. 2020-21.
Mr. Pankaj Bhuj was assigned its audit as an engagement partner and he was verifying
the composition of the Board of Director because of some changes in the same. The
present composition of the Board of Kayask Ltd. is as follows: -
(1) There are 9 directors out of which there are 4 non-executive directors and 3
independent directors. The board has only one woman director and she is an
executive director.
(2) Mr. Madhusudan Mehra has been appointed as the non-executive chairperson of the
Board. He is brother in law of the Managing Director of Kayask Ltd.
Whether present composition of the board of Kayask Ltd. complies with the requirement of
the provisions of SEBI LODR Regulations?
Audit of Banks & Insurance Company
16. (a) Gupta & Co. has been appointed as a statutory auditor of TCB Bank Ltd., a private
sector bank, registered with RBI. Mr. Kaival Gupta, the engagement partner, while
performing the audit as per the checklist, noted down the following points, which
would be part of the audit queries, as tabulated below:
Sr. Queries
No.
1 Interest on State Government Guaranteed advance has been taken to
income even though such advance has remained overdue for more than
90 days.
2 There is an account for which an ad hoc limit has not been reviewed for
180 days from the date of such ad hoc sanction and such account has
been treated as a performing asset in the books.
3 One of the NPAs was sold for a value higher than the net book value. Profit
was not recognized but the excess provision in respect of the same has
been reversed.
4 In case of one of the accounts, an additional temporary limit has been
sanctioned for 25% of the existing limit and for 120 days tenure.
5 On verification of outstanding forward exchange contracts, the ‘net
position’ in respect of one of the foreign currencies was not squared and
was uncovered by a substantial amount.

© The Institute of Chartered Accountants of India


12 FINAL (NEW) EXAMINATION: NOVEMBER, 2021

You are required to provide the reasons due to which such queries would have been
raised by Mr. Kaival and describe the actions that may be taken by the person
responsible on behalf of TCB Bank Ltd. for solving such queries.
(b) You have been appointed to carry out the audit of ‘The Blue Insurance Company Ltd.’
for the year 2019-20. In the course of your audit, you observed that the commission
paid to agents constituted a major expense in operating expenses of the Company.
Enumerate the audit concerns that address to the assertions required for the Auditor
to ensure the continued existence of internal control as well as fairness of the
amounts in accounting of commission paid to agents.
Audit under Fiscal Laws
17. UT & Co. is a Chartered Accountant Firm, that provides consultancy services. Recently, it
got queries from different clients with respect to applicability of tax audit provisions to their
businesses.
In response to such queries, UT & Co., asked from them details such as turnover, total
receipts and total payments made during the year respectively along with mode of
receipt/payment, whether filing return of Income under normal tax provisions or
presumptive tax provisions such as section 44AD, 44AE, etc.
So, in the trailing mail, UT & Co., got the aforesaid details from different clients, which it
classified into following categories for ease of framing an opinion, as follows:
Client Turnover % of Cash % of Cash Remarks
Sr. No. (` in crore) Receipts in Payments in
Total Total
Receipts Payments
1 4.5 5% 5% Has been filing return as per the
regular provisions of income tax.
2 1.8 7% 4% Has declared business income
as per presumptive taxation
under section 44AD of the
Income-tax Act, 1961.
3 0.85 6% 4% Has declared business income
as per presumptive taxation
under section 44AD of the
Income-tax Act, 1961 during last
2 previous years but during
current previous year has
declared income lower than as
per section 44AD and the total

© The Institute of Chartered Accountants of India


PAPER – 3 : ADVANCED AUDITING AND PROFESSIONAL ETHICS 13

income is less than basic


exemption limit.
4 3.2 8% 6% Has declared business income
as per presumptive taxation
under section 44AE of the
Income-tax Act, 1961 during
last 4 previous years but during
current previous year has
declared income lower than as
per section 44AE and the total
income is less than basic
exemption limit.

On behalf of UT & Co., please provide your opinion, along with reasons, as a consultant in
case of aforesaid clients that whether tax audit is applicable to them or not?
PEER REVIEW
18. (a) Roshan, a practicing Chartered Accountant is appointed to conduct the peer review
of another practicing unit. What areas Roshan should review in the assessment of
independence of the practicing unit?
(b) You are required to classify the following practice units into Level I entity or Level II
entity for the purpose of peer review along with providing the reason for such
classification, assuming the services have been undertaken in the period under
review by such CA firms:
Name of the Firm Data of assurance services provided by such firms
MT & Co. Conducted statutory audit of a private company which is an
associate of a company, the net worth of which is
` 300 crore.
GBL & Co. Conducted statutory audit of a Mutual Fund Company.
IML & Associates Conducted statutory audit of a company registered under
Section 8 of the Companies Act, 2013 but is not covered as
a public interested entity. However, it has raised a
contribution of ` 60 crore.
BTS & Co. Conducted statutory audit of an unlisted public company
having net worth of ` 4 crore and turnover of ` 55 crore. The
net worth of its parent company is ` 325 crore.

© The Institute of Chartered Accountants of India


14 FINAL (NEW) EXAMINATION: NOVEMBER, 2021

TJK & Associates Conducted statutory audit of LLP which has raised has a loan
of ` 35 crore from a bank and a loan of ` 10 crore from an
NBFC, respectively.

(c) Evaluating the professional judgment exercised by the auditor is one of the important
aspects under Quality review, please explain the situation with reference to applicable
SA.
Professional Ethics
19. The Director (Discipline) of the ICAI received information of alleged misconduct against
Mr. Jayprakash, the proprietor of JP & Associates, as follows:-
(i) Audit of a college was accepted by JP & Associates in which Mr. Jayprakash is
working as a part-time lecturer and also, he had not taken permission of the ICAI for
working as a part-time lecturer in the college.
(ii) An event relating to Corporate Social Responsibility was sponsored by JP &
Associates, whereby in the sponsorship banner, name of Mr. Jayprakash as
‘CA Jayprakash, Proprietor, JP & Associates’ was mentioned.
On the basis of above information and along with certain evidence against
Mr. Jayprakash, he was found guilty and so he was reprimanded and a fine of ₹ 1 lakh was
imposed by an order passed against him dated 12 th July, 2020.
Against the said order, Mr. Jayprakash preferred an appeal with the Appellate Authority on
17th August, 2020 by submitting a statement of appeal along with the application form of
appeal. During such appellate proceedings, it was discovered that the said statement of
appeal contained some facts which were false to which Mr. Jayprakash admitted it to be
false and apologized for it.
(a) Mr. Jayprakash has violated which of the provisions of the Chartered Accountants
Act, 1949?
(b) Before which authority, the matter of Mr. Jayprakash would have been placed and
what maximum punishment could have been imposed on him by the said authority in
accordance with the Chartered Accountant Act, 1949?
20. Write a short note on the following:
(a) Responsibility of holding company for preparation of Consolidated Financial
Statements.
(b) Summary Written Report.
(c) Direction by Tribunal in case auditor acted in a fraudulent manner.
(d) Example of Headings of a Due Diligence Report.

© The Institute of Chartered Accountants of India


PAPER – 3 : ADVANCED AUDITING AND PROFESSIONAL ETHICS 15

SUGGESTED ANSWERS/HINTS

PART A : ANSWERS TO MULTIPLE QUESTIONS


1. (b)
2. (d)
3. (b)
4. (c)
5. (a)
6. (c)
7. (b)
8. (c)
9. (d)
10. (d)
PART B
11. (a) As per SA 560, ‘Subsequent Events’, the auditor has no obligation to perform any
audit procedures regarding the financial statements after the date of the auditor’s
report. However, when, after the date of the auditor’s report but before the date the
financial statements are issued, a fact becomes known to the auditor that, had it been
known to the auditor at the date of the auditor’s report, may have caused the auditor
to amend the auditor’s report, the auditor shall:
(1) Discuss the matter with management and, where appropriate, those charged
with governance.
(2) Determine whether the financial statements need amendment and, if so,
(3) Inquire how management intends to address the matter in the financial
statements.
In the given case, on becoming aware of the court case filed against Kolsi (P) Ltd.,
Mr. Raj discussed the said matter with the management and it was determined to
amend the financial statements. Also, he inquired how the management intended to
address the said matter in the financial statements.
However, If management does not take the necessary steps to ensure that anyone in
receipt of the previously issued financial statements is informed of the situation and
does not amend the financial statements in circumstances where Mr. Raj (hereinafter
referred as ‘the auditor’) believes they need to be amended, the auditor shall notify

© The Institute of Chartered Accountants of India


16 FINAL (NEW) EXAMINATION: NOVEMBER, 2021

management and, those charged with governance (unless all of those charged with
governance are involved in managing the entity), that the auditor will seek to prevent
future reliance on the auditor’s report. If despite such notification the management or
those charged with governance do not take these necessary steps, the auditor shall
take appropriate action to seek to prevent reliance on the auditor’s report in
accordance with SA 560.
(b) As per SA 706, ‘Emphasis of Matter Paragraphs and Other Matter Paragraphs in the
Independent Auditor’s Report’, an Emphasis of Matter paragraph is not a substitute
for:
(a) A modified opinion in accordance with SA 705 (Revised) when required by the
circumstances of a specific audit engagement;
(b) Disclosures in the financial statements that the applicable financial reporting
framework requires management to make, or that are otherwise necessary to
achieve fair presentation; or
(c) Reporting in accordance with SA 570 (Revised) when a material uncertainty
exists relating to events or conditions that may cast significant doubt on an
entity’s ability to continue as a going concern.
In the given case, the management of Kolsi (P) Ltd. has presumed that as the auditor
was going to provide a description of the said court case and its outcome in the
‘Emphasis of Matter’ paragraph in his amended audit report, there was no further
need for it to provide additional disclosures about the court case in the financial
statements.
The said contention of management of Kolsi (P) Ltd. is not valid as ‘Emphasis of
Matter’ paragraph cannot be used as a substitute for disclosures required to be made
in the financial statements as per the applicable financial reporting framework or that
is otherwise necessary to achieve fair presentation, which is the responsibility of the
management.
12. As per SA 550, “Related Parties”, according to para on “Responses to the risks of material
misstatement associated with related party relationships and transactions”, the auditor
should design and performs further audit procedures to obtain sufficient appropriate audit
evidence about the assessed risks of material misstatement associated with related party
relationships and transactions.
Further, as per SA 330, “The Auditor’s Responses to Assessed Risks”, the auditor shall
design and perform tests of controls to obtain sufficient appropriate audit evidence as to
the operating effectiveness of relevant controls when:

© The Institute of Chartered Accountants of India


PAPER – 3 : ADVANCED AUDITING AND PROFESSIONAL ETHICS 17

(a) the auditor’s assessment of risks of material misstatement at the assertion level
includes an expectation that the controls are operating effectively (i.e., the auditor
intends to rely on the operating effectiveness of controls in determining the nature,
timing and extent of substantive procedures); or
(b) Substantive procedures alone cannot provide sufficient appropriate audit evidence at
the assertion level.
In designing and performing tests of controls, the auditor shall obtain more persuasive
audit evidence the greater the reliance the auditor places on the effectiveness of a control.
Moreover, the auditor shall test controls for the particular time, or throughout the period,
for which the auditor intends to rely on those controls, subject to when the auditor obtains
audit evidence about the operating effectiveness of controls during an interim period, and
the timing of test of controls over significant risks, in order to provide an appropriate basis
for the auditor’s intended reliance.
When the auditor obtains audit evidence about the operating effectiveness of controls
during an interim period, the auditor shall:
(a) Obtain audit evidence about significant changes to those controls subsequent to the
interim period; and
(b) Determine the additional audit evidence to be obtained for the remaining period.
In the current case, Ms. K shall design and perform tests of controls to obtain sufficient
appropriate audit evidence as to the operating effectiveness of relevant controls as she
intends to rely on the operating effectiveness of controls in determining the nature, timing
and extent of substantive procedures.
Further, she is also required to obtain the audit evidence about significant changes to those
controls subsequent to the interim period along with the additional audit evidence to be
obtained for the remaining period in accordance with the requirements of Standards on
Auditing as discussed above.
13. As per section 139(1) of the Companies Act, 2013, every company shall, at the first annual
general meeting, appoint an individual or a firm as an auditor who shall hold office from
the conclusion of that meeting till the conclusion of its sixth annual general meeting and
thereafter till the conclusion of every sixth meeting and the manner and procedure of
selection of auditors by the members of the company at such meeting shall be such as
may be prescribed.
It may be noted further that before such appointment is made, the written consent of the
auditor to such appointment, and a certificate from him or it that the appointment, if made,
shall be in accordance with the conditions as may be prescribed, shall be obtained from
the auditor.

© The Institute of Chartered Accountants of India


18 FINAL (NEW) EXAMINATION: NOVEMBER, 2021

It may also be noted that the certificate shall also indicate whether the auditor satisfie s the
criteria provided in section 141 of the Companies Act, 2013.
Further, as per section 139(2), “(2) No listed company or a company belonging to such
class or classes of companies as may be prescribed, shall appoint or re-appoint (a) an
individual as auditor for more than one term of five consecutive years; and (b) an audit firm
as auditor for more than two terms of five consecutive years.
It may also be noted further that as on the date of appointment no audit firm having a
common partner or partners to the other audit firm, whose tenure has expired in a company
immediately preceding the financial year, shall be appointed as auditor of the same
company for a period of five years:”
In the current case, while appointing the auditors of the company a written consent of the
auditor to such appointment was not obtained. Moreover a certificate from him that the
appointment if made shall be in accordance with the conditions laid down in the Act and
Rules was also not obtained. Further, majority of the partners of AB & Co. were partners
in BCD & Co. AB & Co. already served two terms of five consecutive years i.e., from 2010 -
11 to 2019-20 as a statutory auditor of the company.
Hence, BCD & Co. were not eligible to be appointed as an auditor of NOME Limited as all
partners of BCD & Co are partner of AB & Co. who have already served two terms of five
consecutive years as an auditor of NOME Limited. Since, before the appointment of
Statutory Auditor, the management should have obtained the required certification and
written consent from BCD & Co., therefore, in this case both, the management and the
auditors have contravened the provision of the Companies Act, 2013 as a result fine as
per section 147 of Companies Act will be applicable i.e. if any of the provisions of sections
139 to 146 (both inclusive) is contravened, the company shall be punishable with fine which
shall not be less than twenty-five thousand rupees but which may extend to five lakh rupees
and every officer of the company who is in default shall be punishable with fine which shall
not be less than ten thousand rupees but which may extend to one lakh rupees. If an
auditor of a company contravenes any of the provisions of section 139, section 144 or
section 145, the auditor shall be punishable with fine which shall not be less than twenty -
five thousand rupees, but which may extend to five lakh rupees or four times the
remuneration of the auditor, whichever is less.
It may be noted that if an auditor has contravened such provisions knowingly or willfully
with the intention to deceive the company or its shareholders or creditors or tax authorities,
he shall be punishable with imprisonment for a term which may extend to one year and
with fine which shall not be less than fifty thousand rupees, but which may extend to twenty -
five lakh rupees or eight times the remuneration of the auditor, whichever is less.

© The Institute of Chartered Accountants of India


PAPER – 3 : ADVANCED AUDITING AND PROFESSIONAL ETHICS 19

14. As per SA 701, ‘Communicating Key Audit Matters in the Independent Auditor’s Report’,
the auditor shall determine, from the matters communicated with those charged with
governance, those matters that required significant auditor attention in performing the
audit. In making this determination, the auditor shall take into account the following:
(i) Areas of higher assessed risk of material misstatement, or significant risks identified
in accordance with SA 315.
(ii) Significant auditor judgments relating to areas in the financial statements that
involved significant management judgment, including accounting estimates that have
been identified as having high estimation uncertainty.
(iii) The effect on the audit of significant events or transactions that occurred during the
period.
The auditor shall determine which of the aforesaid matters considered were of most
significance in the audit of the financial statements of the current period and therefore are
the key audit matters.
These aforesaid considerations focus on the nature of matters communicated with those
charged with governance. Such matters are often linked to matters disclosed in the
financial statements and are intended to reflect areas of the audit of the financial
statements that may be of particular interest to intended users.
The fact that these considerations are required is not intended to imply that matters related
to them are always key audit matters; rather, matters related to such specific
considerations are key audit matters only if they are determined to be of most significance
in the audit.
In addition to matters that relate to the specific required considerations, there may be other
matters communicated with those charged with governance that required significant
auditor attention and that therefore may be determined to be key audit matters. Such
matters may include, for example, matters relevant to the audit that was performed that
may not be required to be disclosed in the financial statements. For example, the
implementation of a new IT system (or significant changes to an existing IT system) during
the period may be an area of significant auditor attention, in particular if such a change
had a significant effect on the auditor’s overall audit strategy or related to a significant risk
(e.g., changes to a system affecting revenue recognition).
In the given case, there was implementation of ERP system in the company due to which
some of its business processes got automated and which had a significant effect on the
auditor’s overall audit strategy during the period.

© The Institute of Chartered Accountants of India


20 FINAL (NEW) EXAMINATION: NOVEMBER, 2021

Accordingly, such a matter can be considered as a key audit matter if according to Mr.
Hemant, such a matter required significant attention that had affected his overall audit
strategy.
Thus, the contention of Mr. Vishay is not proper as matters that do not link with the matters
disclosed in the financial statements can also be considered as a key audit matter if it
required significant attention of the auditor which had an impact on its audit.
15. As per Regulation 17 and 17A of the SEBI LODR Regulations, -
✓ The auditor should ascertain whether, throughout the reporting period, the Board of
Directors comprises an optimum combination of executive and non-executive
directors, with at least one woman director and not less than 50% of the Board of
Directors comprising non-executive directors.
It may be noted that the Board of directors of the top 1000 listed entities shall have
at least one independent woman director.
✓ The auditor should also verify that where the Chairperson of the Board is a non-
executive director, at least one-third of the Board should comprise of independent
directors.
✓ The auditor shall ensure that the Chairperson of the board of the top 500 listed entities
is - (a) a non-executive director; (b) not related to the Managing Director or the Chief
Executive Officer as per the definition of the term “relative” defined under the
Companies Act, 2013.
As per the term “relative” defined under the Companies Act, 2013 – Brother-in-law
i.e. sister’s husband is not included.
In the given case, Kayask Ltd. is a public company which got listed on BSE and NSE in
the F.Y. 2015-16 and is amongst the top 500 listed entities on the basis of market
capitalization. The present composition of the board of Kayask Ltd includes 9 directors out
of which there are 4 non-executive directors and 3 independent directors. The board has
only one woman director and she is an executive director. In addition , Chairperson of the
Board Mr. Madhusudan Mehra is brother in law of the Managing Director of Kayask Ltd.
and has been appointed as the non-executive Chairperson.
In view of Regulation 17 and 17A of the SEBI LODR Regulations, there should at least 5
non-executive directors and 3 Independent directors as its Chairperson is a non-executive
director.
Further as the company is amongst the top 500 listed entities, at least one independent
woman director should be there in its board.
Thus, it can be concluded that the present composition of the board of Kayask Ltd. does
not comply with the requirement of the provisions of SEBI LODR Regulations as the woman
director should be an independent director and there should be 5 non-executive directors.

© The Institute of Chartered Accountants of India


PAPER – 3 : ADVANCED AUDITING AND PROFESSIONAL ETHICS 21

16. (a)
Sr. Reason for such Query Action that may be taken in
No. response to the query
1 A State Government Guaranteed Interest income recognized on such
advance has to be treated as NPA advance would be reversed and would
even if it remains overdue for more be taken to income only when it is
than 90 days and in case of NPA, for realized.
the purpose of income recognition,
interest on such advance should not
be taken to income unless interest is
realized.
2 Accounts for which an ad hoc limit It’s treatment in the books would be
has not been reviewed for 180 days changed from performing asset to a
from the date of such ad hoc non-performing asset from the date
sanction, should be considered as when such change in the treatment
NPA. was required.
3 In case of sale of NPA, where the The entry for reversal of the excess
sale is for a value higher than the provision would be cancelled in the
NBV, the auditor is required to books and such excess provision
ensure that no profit is recognized, would be retained to meet the
and the excess provision has not shortfall/ loss that may arise because
been reversed but retained to meet of the sale of other non-performing
the shortfall/ loss that may arise financial assets.
because of the sale of other non-
performing financial assets.
4 Additional temporary limit may be The terms of additional temporary limit
sanctioned, for a maximum of 20% in case of such account would be
of the existing limit and 90 days revised to 20% of the existing limit and
maximum tenure. for 90 days maximum tenure.
5 Net position in respect of each of the The net “position” of the branch in
foreign currencies should be relation to each foreign currency
generally squared and should not be should be squared off and get covered
uncovered by a substantial amount. by a substantial amount.

(b) Commission/Brokerage: The commission is the consideration payable for getting


the insurance business. The term ‘commission’ is used for the payment of
consideration to get Direct business. Commission received on amount of premium
paid to a re-insurer is termed ‘Commission on reinsurance accepted’ and is reduced
from the amount of commission expenditure. The internal control with regard to
commission is aimed at ensuring that commission is paid in accordance with the rules

© The Institute of Chartered Accountants of India


22 FINAL (NEW) EXAMINATION: NOVEMBER, 2021

and regulations of the company and in accordance with the agreement with the agent,
commission is paid to the agent who brought the business and the legal compliances,
for example, tax deduction at sources, GST on reverse charge mechanism and
provisions of the Insurance Act, 1938 have been complied with.
It is a well-known fact that insurance business is solicited by insurance agents. The
remuneration of an agent is paid by way of commission which is calculated by
applying a percentage to the premium collected by him. Commission is payable to the
agents for the business procured through them and is debited to Commission on
Direct Business Account. There is a separate head for commission on reinsurance
accepted which usually arise in case of Head Office. It may be noted that under
section 40 of Insurance Act, 1938, no commission can be paid to a person who is not
an agent of the insurance company. Commission cannot be paid in excess of the
maximum rates of commission as framed by IRDAI. The rates of
commission/brokerage are agreed and documented with the agent an d filed with
IRDAI.
Role of Auditor: The auditor should, inter alia, do the following for verification of
commission:
• Ensure that commission/brokerage is not paid in excess of the limits specified
by IRDAI
• Ensure that commission/brokerage is paid as per rates with the agent and rates
filed with IRDAI
• Ensure that commission/brokerage is paid to the agent/broker who has solicited
the business
• Ensure that the agent/broker is not blacklisted by IRDAI and is not terminated
for fraud etc.
• Vouch disbursement entries with reference to the disbursement vouchers with
copies of commission bills and commission statements.
• Check whether the vouchers are authorised by the officers-in–charge as per
rules in force and income tax is deducted at source, as applicable.
• Test check correctness of amounts of commission allowed.
• Scrutinise agents’ ledger and the balances, examine accounts havi ng debit
balances, if any, and obtain information on the same. Necessary rectification of
accounts and other remedial actions have to be considered.
• Check whether commission outgo for the period under audit been duly
accounted.

© The Institute of Chartered Accountants of India


PAPER – 3 : ADVANCED AUDITING AND PROFESSIONAL ETHICS 23

17. Applicability of Tax Audit:


Client Opinion Reason
Sr. No. (Tax Audit
applicable
or not)
1 No As the turnover is upto ` 5 crore, Cash Receipts and Cash
Payments are upto 5% of total receipts & total payments,
respectively, and has been filing return as per the regular
provisions of income tax, so tax audit is not applicable.
2 No Even though turnover exceeds ` 1 crore and Cash Receipts are
greater than 5% of Total Receipts but as the business income
has been declared as per presumptive taxation under section
44AD of the Income-tax Act, 1961, so tax audit is not
applicable.
3 No Even though business income as per presumptive taxation
under section 44AD of the Income-tax Act, 1961 has been
declared during last 2 previous years but has not been declared
during the current previous year but as the total income is less
than basic exemption limit, so tax audit is not applicable.
4 Yes Has been declaring total income as per presumptive taxation
under section 44AE of the Income-tax Act, 1961 during last 4
previous years but during current previous year has declared
income lower than as per section 44AE, so tax audit is
applicable.

18. (a) Review in the Assessment of Independence of the Practicing Unit – The reviewer
should carry out the compliance review of the five general controls, i.e.,
independence, maintenance of professional skills and standards, outside
consultation, staff supervision and development and office administration and
evaluate the degree of reliance to be placed upon them. The degree of reliance will,
ultimately, affect the attestation service engagements to be reviewed.
Independence is the main quality expected of an auditor. That is the very basis for
the existence of the profession of auditing. Independence is a condition of mind as
well a personal character of a person. It is difficult to define but very easy to perceive.
Guidance Note on Independence of Auditors clarifies that independence is of two
types, viz. independence of mind and independence of appearance. The Guidance
Note further states that there are certain threats to independence which are classified
as self interest threats, self review threats, advocacy threats, familiarity threats and
intimidation threats.

© The Institute of Chartered Accountants of India


24 FINAL (NEW) EXAMINATION: NOVEMBER, 2021

The responsibility of the Peer Reviewer, therefore, is to ascertain the existence of


independence and the absence of threats to independence.
The reviewer should, therefore, check the following aspects in respect of assessment
of independence of the practicing unit:
(i) Does the practice unit have a policy to ensure independence, objectivity and
integrity, on the part of partners and staff? Who is responsibl e for this policy?
(ii) Does the practice unit communicate these policies and the expected standards
of professional behaviour to all staff?
(iii) Does the practice unit monitor compliance with policies and procedures relating
to independence?
(iv) Does the practice unit periodically review the practice unit's association with
clients to ensure objectivity and independence?
(v) How does the practice unit deal with the threats to independence ?
(b) Classification of Entity as per Statement of Peer Review
Name of Entity Type of Reason for such classification based on
Entity the Statement of Peer Review
MT & Co. Level I entity A Practice Unit which has undertaken
Statutory Audit of a company which is an
associate of an entity having net worth of
more than ` 250 Crores at any time during
the period under Review, shall be treated as
a Level I entity.
GBL & Co. Level I entity A Practice Unit which has undertaken
Statutory Audit of a mutual fund shall be
treated as a Level I entity.
IML & Associates Level I entity A Practice Unit which has undertaken
Statutory Audit of an Entity which has raised
donations and / or contributions over ` 50
crore during the period under Review, shall
be treated as a Level I entity.
BTS & Co. Level I entity A Practice Unit which has undertaken
Statutory Audit of a company which is a
subsidiary of an entity having net worth of
more than ` 250 Crores at any time during
the period under Review, shall be treated as
a Level I entity.

© The Institute of Chartered Accountants of India


PAPER – 3 : ADVANCED AUDITING AND PROFESSIONAL ETHICS 25

TJK & Associates Level II entity A Practice Unit which has undertaken
Statutory Audit of an entity which has raised
funds from public or banks or financial
institutions of more than ` 25 crore but less
than ` 50 crore during the period under
Review, shall be treated as a Level II entity.

(c) Evaluating the professional judgment exercised by the auditor: It is also


important for the Technical Reviewer (hereinafter referred as TR) to understand that
“professional judgment”, as defined in SA 200, “Overall Objectives of the Independent
Auditor and the Conduct of an Audit in Accordance with Standards on Auditing” is an
integral concept in the context of an audit and application of SAs in real life audit
scenarios. SA 200 defines professional judgment as “the application of relevant
training, knowledge and experience, within the context provided by auditing,
accounting and ethical standards, in making informed decisions about the course of
action that is appropriate in the circumstances of the audit engagement.”
The concept of “professional judgment” underscores the fact that Standards,
particularly, Standards on Auditing are written to lay down the fundamental principles
that would apply to an audit situation. Hence, no Standard can have straight jacketed
application/solutions for all audit scenarios. Above all, the Standards on Auditing
issued by the Institute of Chartered Accountants of India are principle based rather
than rule based. Hence, almost all the SAs envisage exercise of professional
judgment by the auditor in their application in real life audit scenarios.
The TR would need to appreciate that the exercise of professional judgment in any
particular case is based on the facts and circumstances that are known to the auditor
as at the time of exercising that professional judgment. Normally, exercise of
professional judgement by an auditor is preceded by consultation on the relevant
matters both within the engagement team and between the engagement team and
others at the appropriate level within or outside the firm.
In evaluating the professional judgment exercised by the auditor, the TR should
consider the following factors:
• whether the judgment reached reflects a due consideration and application of
the relevant auditing and accounting principles; and
• whether the judgment is appropriate in the light of, and consistent with, the facts
and circumstances that were known to the auditor up to the date of the auditor’s
report. Hence, the TR and the QR Team should not, under any circumstance,
use “hindsight” (i.e. perception or retrospection) in their evaluation of exercise
of professional judgment by the auditor.

© The Institute of Chartered Accountants of India


26 FINAL (NEW) EXAMINATION: NOVEMBER, 2021

Since the auditor needs to exercise professional judgment throughout the audit, the
latter also needs to be appropriately documented. Hence, the TR can expect to find
such audit documentation as a part of the audit engagement file. It is important to
note that professional judgment cannot be used by an auditor as a justification for
decisions that are not otherwise supported by the facts and circumstances of the
engagement or sufficient appropriate audit evidence.
19. (a) Mr. Jayprakash has violated following provisions of the Chartered Accountants
Act, 1949:
(i) As per Clause (4) of Part I of the Second Schedule to the Chartered Accountants
Act, 1949, a Chartered Accountant in practice shall be deemed to be guilty of
professional misconduct, if he expresses his opinion on financial statements of
any business or enterprise in which he, his firm, or a partner in his firm has a
substantial interest.
In this connection, as per the decision of the Council of the ICAI, a Chartered
Accountant should not by himself or in his firm name accept the audit of a
college, if he is working as a part-time lecturer in the college.
Thus, by accepting audit of a college in which he is working as a part-time
lecturer, Mr. Jayprakash has violated the restriction imposed under Clause (4)
of Part I of the Second Schedule to the Chartered Accountants Act, 1949.
(ii) As per Clause (11) of Part I of the First Schedule to the Chartered Accountants
Act, 1949, a Chartered Accountant in practice shall be deemed to be guilty of
professional misconduct, if he engages in any business or occupation other than
the profession of chartered accountant unless permitted by the Council so to
engage.
Members of the Institute in practice may engage in a part-time or full-time
tutorship under any educational institution other than the coaching organization
of the Institute, after obtaining the specific and prior approval of the Council in
each case.
Mr. Jayprakash had not taken permission of the ICAI for working as a part-time
lecturer in the college and so has violated the restriction imposed under Clause
(11) of Part I of the First Schedule to the Chartered Accountants Act, 1949.
(iii) As per Clause (6) of Part I of the First Schedule to the Chartered Accountants
Act, 1949, a Chartered Accountant in practice shall be deemed to be guilty of
professional misconduct, if he solicits clients or professional work either directly
or indirectly by circular, advertisement, personal communication or interview or
by any other means.

© The Institute of Chartered Accountants of India


PAPER – 3 : ADVANCED AUDITING AND PROFESSIONAL ETHICS 27

In this connection, members sponsoring activities relating to Corporate Social


Responsibility may mention their individual name with the prefix “CA”. However,
mentioning a firm’s name or CA Logo is not permitted.
An event relating to Corporate Social Responsibility was sponsored by JP &
Associates, whereby in the sponsorship banner, name of Mr. Jayprakash as ‘CA
Jayprakash, Proprietor, JP & Associates’ was mentioned. Thus, firm’s name was
mentioned which is not allowed and thus, Mr. Jayprakash has violated the
restriction imposed under Clause (6) of Part I of the First Schedule to the
Chartered Accountants Act, 1949.
(iv) As per Clause (3) of Part II of the Second Schedule to the Chartered
Accountants Act, 1949, a member of the ICAI shall be deemed to be guilty of
professional misconduct, if he includes in any information, statement, return or
form to be submitted to the Institute, Council or any of its Committees, Director
(Discipline), Board of Discipline, Disciplinary Committee, Quality Review Board
or the Appellate Authority, any particulars knowing them to be false.
Mr. Jayprakash in the statement of appeal submitted with the Appellate Authority
mentioned some facts knowing them to be false and thus, he has violated the
restriction imposed under Clause (3) of Part II of the Second Schedule to the
Chartered Accountants Act, 1949.
(b) As Mr. Jayprakash has been alleged of misconduct falling in First as well as Second
Schedule, so the matter would be placed before the Disciplinary Committee.
The maximum punishment which could have been imposed on him by the said
authority would be:-
(i) reprimanding the member.
(ii) removing name of the member permanently or for any duration, it thinks fit.
(iii) imposing fine upto ` 5,00,000.
20. (a) Responsibility of holding company for preparation of Consolidated Financial
Statements: The responsibility for the preparation and presentation of consolidated
financial statements, among other things, is that of the management of the parent.
This includes:
(a) identifying components, and including the financial information of the
components to be included in the consolidated financial statements;
(b) where appropriate, identifying reportable segments for segmental reporting;
(c) identifying related parties and related party transactions for reporting;

© The Institute of Chartered Accountants of India


28 FINAL (NEW) EXAMINATION: NOVEMBER, 2021

(d) obtaining accurate and complete financial information from components;


(e) making appropriate consolidation adjustments;
(f) harmonization of accounting policies and accounting framework; and
(g) GAAP conversion, where applicable.
Apart from the above, the parent ordinarily issues instructions to the management of
the component specifying the parent’s requirements relating to financial information
of the components to be included in the consolidated financial statements. The
instructions ordinarily cover the accounting policies to be applied, statutory and other
disclosure requirements applicable to the parent, including the identification of and
reporting on reportable segments, and related parties and related party transactio ns,
and a reporting timetable.
(b) Summary Written Report: These summary reports are also referred to as ‘flash’
reports’. In a number of companies, the practice has developed of issuing an annual
(or sometimes more frequent) report summarising the various individual reports
issued and describing the range of their content. These summary reports in some
cases are primarily for audit committees of Boards of Directors, but in other cases for
higher level management. They are especially useful to top level managers who do
not actively review the individual reports. They are also useful to the general auditor
in seeing his total reporting effort with more perspective and on an integrated basis.
(c) Direction by Tribunal in case auditor acted in a fraudulent manner: As per sub-
section (5) of the section 140, the Tribunal either suo motu or on an application made
to it by the Central Government or by any person concerned, if it is satisfied that the
auditor of a company has, whether directly or indirectly, acted in a fraudulent manner
or abetted or colluded in any fraud by, or in relation to, the company or its directors
or officers, it may, by order, direct the company to change its auditors.
However, if the application is made by the Central Government and the Tribunal is
satisfied that any change of the auditor is required, it shall within fifteen days of receipt
of such application, make an order that he shall not function as an auditor and the
Central Government may appoint another auditor in his place.
It may be noted that an auditor, whether individual or firm, against whom final order
has been passed by the Tribunal under this section shall not be eligible to be
appointed as an auditor of any company for a period of five years from the d ate of
passing of the order and the auditor shall also be liable for action under section 447.
It is hereby clarified that the case of a firm, the liability shall be of the firm and that of
every partner or partners who acted in a fraudulent manner or abe tted or colluded in
any fraud by, or in relation to, the company or its director or officers.

© The Institute of Chartered Accountants of India


PAPER – 3 : ADVANCED AUDITING AND PROFESSIONAL ETHICS 29

(d) Example of Headings of a Due Diligence Report


 Executive Summary
 Introduction
 Background of Target company
 Objective of due diligence
 Terms of reference and scope of verification
 Brief history of the company
 Share holding pattern
 Observations on the review
 Assessment of management structure
 Assessment of financial liabilities
 Assessment of valuation of assets
 Comments on properties, terms of leases, lien and encumbrances.
 Assessment of operating results
 Assessment of taxation and statutory liabilities
 Assessment of possible liabilities on account of litigation and legal proceedings
against the company
 Assessment of net worth
 Interlocking investments and financial obligations with group / associates
companies, amounts receivables subject to litigation, any other likely liability
which is not provided for in the books of account
 SWOT Analysis
 Comments on future projections
 Status of charges, liens, mortgages, assets and properties of the company
 Suggestion on ways and means including affidavits, indemnities, to be executed
to cover unforeseen and undetected contingent liabilities
 Suggestions on various aspects to be taken care of before and after the
proposed merger/acquisition.

© The Institute of Chartered Accountants of India


PAPER – 3 : ADVANCED AUDITING AND PROFESSIONAL ETHICS
Question No. 1 is compulsory
Answer any four from the rest
Question 1
(a) Moon Ltd. is a dealer in electronic appliances. The Company has a centralised warehouse
at the outskirts of Mumbai. The Auditors of the company M/s JK Associates normally attend
the physical verification of stocks carried out by the Management at the end of the financial
year. However, on account of certain disturbances in the region, the physical inventory
counting could not be carried out at the year end. The stock taking is decided to be done
by management at some other date subsequently, after a month.
In the light of the above facts:
Enumerate the audit procedures to be considered by M/s JK Associates, if physical
inventory counting is conducted at a date other than the date of the financial statements
with reference to the relevant Standard on Auditing. (5 Marks)
(b) You are the auditor of PQR Ltd. which is in the business of supplying food products to
various airline companies operating aircrafts in domestic circle only. As per terms of
agreement with airlines, the company needs to stock various non- perishable food items
for coming one month (average holding of inventory to the tune of INR 75 Crores). Also
the payment terms have been settled and the company receives payment in 45 days after
the supply of goods. Everything was going-on well till the end of March 2020 when
pandemic Covid hit the world and everything came to a standstill. Aviation sector was hit
hard and there were no flights from April 2020 onwards. Consequently, the business of
PQR Ltd. also got severely affected and the scheduled supplies of goods to airlines also
were not made. Also, the liquidity position of airline companies got hit and the schedule d
payments were also not received on due dates. As the auditor of PQR Ltd. what audit
procedures would you perform to ensure that all subsequent events are considered, so
that financial statements for the year ended 31.03.2020 represent true and fair view?
(5 Marks)
(c) GS & Co., Chartered Accountants, have been appointed Statutory Auditors of MAP Ltd. for
the F.Y 2019-20. The audit team has completed the audit and is in the process of preparing
audit report Management of the company has also prepared draft annual report.
Audit in-charge was going through the draft annual report and observed that the company
has included an item in its Annual Report indicating downward trend in market prices of
key commodities/raw material as compared to previous year. However, the actual profit
margin of the company as reported in financial statements has gone in the reverse
direction. Audit Manager discussed this issue with partner of the firm who in reply said
that auditors are not covered with such disclosures made by the management in its annual
report, it being the responsibility of the management.

© The Institute of Chartered Accountants of India


2 FINAL (NEW) EXAMINATION: NOVEMBER 2020

Do you think that the partner is correct in his approach on this issue.
Discuss with reference to relevant Standard on Auditing the Auditor's duties w ith regard to
reporting. (4 Marks)
Answer
(a) As per SA 501 “Audit Evidence- Specific Considerations for Selected Items”, when
inventory is material to the financial statements, the auditor shall obtain sufficient
appropriate audit evidence regarding the existence and condition of inventory.
For practical reasons, the physical inventory counting may be conducted at a date, or
dates, other than the date of the financial statements. This may be done irrespective of
whether management determines inventory quantities by an annual physical inventory
counting or maintains a perpetual inventory system. In either case, the effectiveness of the
design, implementation and maintenance of controls over changes in inventory determines
whether the conduct of physical inventory counting at a date, or dates, other than the date
of the financial statements is appropriate for audit purposes.
If physical inventory counting is conducted at a date other than the date of the financial
statements, the auditor, JK Associates, shall perform the following procedures:
(A) Attendance at physical inventory counting, unless impracticable, to:
(i) Evaluate management’s instructions and procedures for recording and
controlling the results of the entity’s physical inventory counting;
(ii) Observe the performance of management’s count procedures;
(iii) Inspect the inventory; and
(iv) Perform test counts; and
(B) Performing audit procedures over the entity’s final inventory records to determine
whether they accurately reflect actual inventory count results.
In addition to above, auditor shall also perform audit procedures to obtain audit
evidence about whether changes in inventory between the count date and the date
of the financial statements are properly recorded.
Relevant matters for consideration when designing audit procedures to obtain audit
evidence about whether changes in inventory amounts between the count date, or
dates, and the final inventory records are properly recorded include:
1. Whether the perpetual inventory records are properly adjusted.
2. Reliability of the entity’s perpetual inventory records.
3. Reasons for significant differences between the information obtained during the
physical count and the perpetual inventory records.
(b) As per SA 560 “Subsequent Events”, the auditor shall perform audit procedures
designed to obtain sufficient appropriate audit evidence that all events occurring between

© The Institute of Chartered Accountants of India


PAPER – 3 : ADVANCED AUDITING AND PROFESSIONAL ETHICS 3

the date of the financial statements and the date of the auditor’s report that require
adjustment of, or disclosure in, the financial statements have been identified. The auditor
is not, however, expected to perform additional audit procedures on matters to which
previously applied audit procedures have provided satisfactory conclusions.
The auditor shall perform the procedures required in above paragraph so that they cover
the period from the date of the financial statements to the date of the auditor’s report, or
as near as practicable thereto.
Being the auditor of PQR Ltd, to ensure that all subsequent events are considered so that
financial statements for the year ending 31.03.2020 represent true and fair view, the
auditor shall take into account the auditor’s risk assessment in determining the nature and
extent of such audit procedures, which shall include the following:
(a) Obtaining an understanding of any procedures management has established to
ensure that subsequent events are identified.
(b) Inquiring of management and, where appropriate, those charged with governance as
to whether any subsequent events have occurred which might affect the financial
statements.
(c) Reading minutes, if any, of the meetings, of the entity’s owners, management and
those charged with governance, that have been held after the date of the financial
statements and inquiring about matters discussed at any such meetings for which
minutes are not yet available.
(d) Reading the entity’s latest subsequent interim financial statements, if any.
When, as a result of the procedures performed as required above, the auditor identifies
events that require adjustment of, or disclosure in, the financial statements, the auditor
shall determine whether each such event is appropriately reflected in those financial
statements.
(c) Responding When the Auditor Concludes That a Material Misstatement of the Other
Information Exists:
As per SA 720, “The Auditor’s Responsibility in Relation to Other Information”,
Descriptions of trends in market prices of key commodities or raw materials is an example
of amounts or other Items that may be Included in the other information.
The auditor’s discussion with management about a material inconsistency (or other
information that appears to be materially misstated) may include requesting management
to provide support for the basis of management’s statements in the other information.
Based on management’s further information or explanations, the auditor may be satisfied
that the other information is not materially misstated. For example, management
explanations may indicate reasonable and sufficient grounds for valid differences of
judgment.

© The Institute of Chartered Accountants of India


4 FINAL (NEW) EXAMINATION: NOVEMBER 2020

Auditor’s duties with regard to reporting in the given case are given hereunder:
As per SA 720, “The Auditor’s Responsibility in Relation to Other Information”, if the
auditor concludes that a material misstatement of the other information exists, the auditor
shall request management to correct the other information. If management:
(i) Agrees to make the correction, the auditor shall determine that the correction has
been made; or
(ii) Refuses to make the correction, the auditor shall communicate the matter with those
charged with governance and request that the correction be made.
Contention of the partner of the firm that auditors are not concerned with such
disclosures made by the management in its annual report, is incorrect.
Question 2
(a) Mr. X, while conducting audit of PQR Ltd, comes across certain transactions which
according to him are significant transactions with related parties and identified to be
outside the entity's normal course of business. Guide Mr. X with examples of such
transactions and to understand the nature of significant transactions outside the entity's
normal course of business. (5 Marks)
(b) M/s PC & Co., Chartered Accountants are the statutory auditors of various categories of
companies and bodies corporate. In exercise of the powers conferred under sub -sections
(2) and (4) of section 132, of the Companies Act, 2013 the Central Government made the
National Financial Reporting Authority Rules, 2018 (NFRA Rules) (MCA Notification dated
13 November 2018). The audit firm seeks your guidance on the applicability of those
categories of companies and bodies corporate which are covered by NFRA Rules.
(5 Marks)
(c) Comment with reference to the Chartered Accountants Act, 1949 and schedules thereto:
CA Dice had signed the Balance sheet of QR Ltd. for the year ended 31st March, 2019
which failed to give disclosure of the charge created for ` 4.35 crores against the Corporate
Guarantee given in favour of a Group Company. The Balance Sheet size of the company
filed with the Registrar of Companies was ` 26.12 crores. (4 Marks)
Answer
(a) In the given case of PQR Ltd, Mr. X, while conducting audit has come across certain
significant related party transaction which are identified to be outside the entity’s normal
course of business. Mr. X wants guidance through examples of such significant
transactions which are given in SA 550
As per SA 550 “Related Parties”, examples of transactions outside the entity’s normal
course of business may include:
1. Complex equity transactions, such as corporate restructurings or acquisitions.

© The Institute of Chartered Accountants of India


PAPER – 3 : ADVANCED AUDITING AND PROFESSIONAL ETHICS 5

2. Transactions with offshore entities in jurisdictions with weak corporate laws.


3. The leasing of premises or the rendering of management services by the entity to
another party if no consideration is exchanged.
4. Sales transactions with unusually large discounts or returns.
5. Transactions with circular arrangements, for example, sales with a commitment to
repurchase.
6. Transactions under contracts whose terms are changed before expiry.
(b) As per NFRA rules, NFRA shall have power to monitor and enforce compliance with
accounting standards and auditing standards, oversee the quality of service under sub -
section (2) of section 132 or undertake investigation under sub-section (4) of such section
of the auditors of the following class of companies and bodies corporate:
(a) companies whose securities are listed on any stock exchange in India or outside
India;
(b) unlisted public companies having paid-up capital of not less than rupees five hundred
crores or having annual turnover of not less than rupees one thousand crores or
having, in aggregate, outstanding loans, debentures and deposits of not less than
rupees five hundred crores as on the 31 st March of immediately preceding
financial year;
(c) insurance companies, banking companies, companies engaged in the generation or
supply of electricity, companies governed by any special Act for the time being in
force or bodies corporate incorporated by an Act in accordance with clauses (b), (c),
(d), (e) and (f) of section 1 (4) of the Companies Act, 2013;
(d) any body corporate or company or person, or any class of bodies corporate or
companies or persons, on a reference made to the NFRA by the Central Government
in public interest; and
(e) a body corporate incorporated or registered outside India, which is a subsidiary or
associate company of any company or body corporate incorporated or registered in
India as referred to in clauses (a) to (d) above, if the income or net -worth of such
subsidiary or associate company exceeds 20% of the consolidated income or
consolidated net-worth of such company or the body corporate, as the case may
be, referred to in clauses (a) to (d) above.
(c) Failure to Disclose Material Facts: As per Clause (5) of Part I of Second Schedule to the
Chartered Accountants Act, 1949, a chartered Accountant in practice will be held liable for
misconduct if he fails to disclose a material fact known to him which is not disclosed in a
financial statement, but disclosure of which is necessary in making such financial
statement not misleading where he is concerned with that financial statement in a
professional capacity.

© The Institute of Chartered Accountants of India


6 FINAL (NEW) EXAMINATION: NOVEMBER 2020

It may be observed that this clause refers to failure to disclose a material fact, which is
known to him, in a financial statement reported on by the auditor. It is obvious, that before
a member could be held guilty of misconduct, materiality has to be established. The
determination of materiality has been provided in SA 320, “Materiality in Planning and
Performing an Audit”.
Financial reporting frameworks often discuss the concept of materiality in the context of
the preparation and presentation of financial statements. Although financial reporting
frameworks may discuss materiality in different terms, they generally explain, among other
points, that Judgments about materiality are made in the light of surrounding
circumstances, and are affected by the size or nature of a misstatement, or a combination
of both.
In this case, CA Dice has signed a Balance Sheet which failed to give disclosure of ` 4.35
crores (considered material fact applying above SA 320 principle) against the corporate
guarantee given in favour of a Group Company. Size of Balance Sheet of QR Ltd is ` 26.12
crore.
This material fact has to be disclosed in the financial statements. Keeping in view
the above, he is attracted by the provisions of professional misconduct under Clause (5)
of Part I of Second Schedule to the Chartered Accountants Act, 1949.
Question 3
(a) In the course of statutory Branch audit of KS Bank Ltd, you observe that some borrower
accounts have been regularised before Balance sheet date by payment of overdue amount.
Narrate the audit procedures to be carried out with special focus on the Classification of
advances and Provisioning for Non-Performing assets of the Branch. (5 Marks)
(b) CA. N has been appointed as an auditor of TRP Ltd. While conducting the audit he has
identified some deficiencies in the Internal control. He needs to determine whether a
deficiency or combination of deficiencies in internal control constitutes a "significant
deficiency" and has to communicate them in writing to those charged with Governance and
management on a timely basis. Guide CA. N with some examples of matters to be
considered while determining 'significant deficiency' in internal control with reference to
relevant SA. (5 Marks)
(c) Mr. Kushal, a practicing Chartered Accountant has signed the GST Audit Reports, Tax
Audit Reports u/s 44AB of the Income tax Act, 1961 for the financial year 2019 -20 that are
filed online using Digital Signature and without generating UDIN on the ground that there
is no field for mentioning UDIN on digitally signed online reports. Is the contention of Mr.
Kushal valid? Give your comments with reference to the Chartered Accountants Act, 1949
and schedules thereto. (4 Marks)

© The Institute of Chartered Accountants of India


PAPER – 3 : ADVANCED AUDITING AND PROFESSIONAL ETHICS 7

Answer
(a) The Audit procedures that need to be carried out with special focus on classification
of advances and provisioning for NPAs of KS Bank Ltd, in which the auditor has
observed that some borrower accounts have been regularized before balance sheet
date by payment of overdue amount shall be carried out as under :
(i) As per the Reserve Bank guidelines, if an account has been regularised before the
balance sheet date by payment of overdue amount through genuine sources, the
account need not be treated as NPA.
(ii) Where subsequent to repayment by the borrower (which makes the account regular),
the branch has provided further funds to the borrower (including by way of
subscription to its debentures or in other accounts of the borrower), the auditor should
carefully assess whether the repayment was out of genuine sources or not.
(iii) Where the account indicates inherent weakness based in the data available, the
account shall be deemed as a NPA.
Classification and Provision
a. Examine whether the classification made by the branch is appropriate. Particularly,
examine the classification of advances where there are threats to recovery.
b. Examine whether the secured and the unsecured portions of advances have been
segregated correctly and provisions have been calculated properly.
c. It is to be ensured that the classification is made as per the position as on date and
hence classification of all standard accounts be reviewed as on balance sheet date.
d. The date of NPA is significant to determine the classification and hence specific care
be taken in this regard.
e. NPA should be recognized only based on concept of Past Due/ Overdue concept, and
not based on the Balance Sheet date.
(b) As per SA 265 “Communicating Deficiencies in Internal Control to Those Charged with
Governance and Management”, significant deficiency in internal control means a
deficiency or combination of deficiencies in internal control that, in the auditor’s
professional judgement, is of sufficient importance to merit the attention of those charged
with governance.
Examples of matters that CA N, auditor of TRP Ltd may consider in determining whether
a deficiency or combination of deficiencies in internal control constitutes a significant
deficiency include:
(1) The likelihood of the deficiencies leading to material misstatements in the financial
statements in the future.
(2) The susceptibility to loss or fraud of the related asset or liability.

© The Institute of Chartered Accountants of India


8 FINAL (NEW) EXAMINATION: NOVEMBER 2020

(3) The subjectivity and complexity of determining estimated amounts, such as fair value
accounting estimates.
(4) The financial statement amounts exposed to the deficiencies.
(5) The volume of activity that has occurred or could occur in the account balance or
class of transactions exposed to the deficiency or deficiencies.
(6) The importance of the controls to the financial reporting process; for example:
• General monitoring controls (such as oversight of management).
• Controls over the prevention and detection of fraud.
• Controls over the selection and application of significant accounting policies.
• Controls over significant transactions with related parties.
• Controls over significant transactions outside the entity’s normal course of
business.
• Controls over the period-end financial reporting process (such as controls over
non-recurring journal entries).
(7) The cause and frequency of the exceptions detected as a result of the deficiencies in
the controls.
(8) The interaction of the deficiency with other deficiencies in internal control.
(c) Non Generation of UDIN: Whereas, to curb the malpractice of
false/certification/attestation by the unauthorized persons and to eradicate the practice of
bogus certificates and to save various regulators, banks, stakeholders etc. from being
misled, the Council of the Institute decided to implement an innovative concept to generate
Unique Document Identification Number (UDIN) mandatorily for all kinds of the
certificates/GST and tax audit reports and other attest function in phased manner, for which
members of the ICAI were notified through the various announcements published on the
website of ICAI at the relevant times.
In exercise of the powers conferred on it under clause 1 of Part II of the Second Schedule
to the Chartered Accountants Act,1949, the Council of the Institute of Chartered
Accountants of India issued the following guidelines for information of public and necessary
compliance by members of the Institute-
A member of the Institute in practice shall generate Unique Document Identification
Number (UDIN) for all kinds of the certification, GST and Tax Audit Reports and other
Audit, Assurance and Attestation functions undertaken/signed by him which are made
mandatory from the following dates through announcements published on the website of
the ICAI-
• For all Certificates w.e.f. 1 st February,2019.

© The Institute of Chartered Accountants of India


PAPER – 3 : ADVANCED AUDITING AND PROFESSIONAL ETHICS 9

• For all GST and Tax Audit Reports w.e.f. 1 st April, 2019.
• For all other Audit, Assurance and Attestation functions, w.e.f. 1 st July, 2019.
Conclusion: UDIN will be applicable to GST & Tax Audit Reports signed by Mr. Kushal for
the financial year 2019-20 that are filed online using Digital Signature. In case where there
is no field for mentioning UDOIN on digitally signed online reports, UDION has to be
generated and communicated to “Management” or “Those Charged with Governance” for
disseminating it to the stakeholders from their end.
Hence he will be held guilty under Clause 1 of Part II of the Second Schedule to the
Chartered Accountants Act,1949.
Alternative Answer
According to Clause (9) of Part I of Second Schedule to the Chartered Accountants Act,
1949, a Chartered Accountant in practice shall be deemed to be guilty of professional
misconduct if he fails to invite attention to any material departure from the generally
accepted procedure of audit applicable to the circumstances.
This clause implies that the audit should be performed in accordance with “generally
accepted procedure of audit applicable to the circumstances” and if for any reason the
auditor has not been able to perform the audit in accordance with such procedure, his
report should draw attention to the material departures from such procedures. What
constitutes “generally accepted audit procedure” would depend upon the facts and
circumstances of each case, but guidance is available in general terms from the various
pronouncements of the Institute is issued by way of statements and Guidance Notes and
SAs to members.
A  member of the Institute in practice shall generate Unique Document Identification
Number (UDIN) for all kinds of the certification, GST and Tax Audit Reports and other
Audit, Assurance and Attestation functions undertaken/signed by him. 
In the given case, Mr. Kushal has signed the GST Audit Reports, Tax Audit Reports under
section 44AB of the Income Tax Act, 1961 for the F.Y.2019-20 and also has filed online
using Digital Signature without generating UDIN on the ground that there is no field for
mentioning UDIN on digitally signed online reports. Applying the above clause, UDIN
provision etc. to the given case, Mr. Kushal would be held guilty of professional
misconduct.
Question 4
(a) PQ Limited, a listed entity, is in the business of manufacturing of specialty chemicals. The
company has appointed CA Jazz as CFO of the company. CA Jazz is concerned abo ut
compliance with the provisions of laws and regulations that determine the reported
amounts and disclosure in financial statements of PQ Limited. Accordingly, CA Jazz wants
to implement such policies and procedures that can assist him in the prevention a nd

© The Institute of Chartered Accountants of India


10 FINAL (NEW) EXAMINATION: NOVEMBER 2020

detection of non-compliance with laws and regulations. Help CA Jazz by citing examples
of such policies and procedures. (5 Marks)
(b) CA. Vimal is the auditor of Excellent Ltd., a parent company which presents Consolidated
Financial Statements. The management of Excellent Ltd. has provided the list of the
components included in the Consolidated Financial Statements. As an auditor of
Consolidated Financial Statements, CA Vimal has to verify that all the components have
been included in the Consolidated Financial Statements and review the information
provided by the management in identifying the components. State the procedures to be
followed by CA. Vimal in respect of completeness of this information. (5 Marks)
(c) In the course of audit of MM Ltd. for the financial year ended 31 st March, 2019, your audit
team has identified the following matter:
All amount of ` 4 Lakh per month for the marketing services rendered is paid to M /s. MG
Associates, a partnership firm in which Director of MM Ltd. is also a managing partner,
with a profit sharing ratio of 30%. Based on an independent assessment, the consideration
paid is higher than the arm's length pricing by ` 1.50 Lakh per month. Whilst the transaction
was accounted in the financial statements based on the amounts paid, no separate
disclosure has been made in the notes forming part of the accounts.
Give your comments for reporting under CARO 2016. (4 Marks)
Answer
(a) In PQ Ltd, listed entity, CA Jazz has been appointed as CFO. PQ Ltd is in the business o f
manufacturing of specialty chemicals. CA Jazz is concerned about compliance with the
provisions of Laws and regulations and wants to implement such policies and procedures
that would assist him in prevention and detection of non-compliance with laws and
regulations. CA Jazz is specifically wanting examples of types of policies and procedures
that PQ Ltd may implement so that relevant laws and regulations are properly complied
with. Such examples of policies and procedures are given in SA 250.
As per SA 250, “Consideration of Laws and Regulations in an Audit of Financial
Statements”,
The following are examples of the types of policies and procedures PQ Ltd. may implement
to assist in the prevention and detection of non-compliance with laws and regulations:
i. Monitoring legal requirements and ensuring that operating procedures are designed
to meet these requirements.
ii. Instituting and operating appropriate systems of internal control.
iii. Developing, publicizing and following a code of conduct.
iv. Ensuring employees are properly trained and understand the code of conduct.

© The Institute of Chartered Accountants of India


PAPER – 3 : ADVANCED AUDITING AND PROFESSIONAL ETHICS 11

v. Monitoring compliance with the code of conduct and acting appropriately to discipline
employees who fail to comply with it.
vi. Engaging legal advisors to assist in monitoring legal requirements.
vii. Maintaining a register of significant laws and regulations with which the entity has
to comply within its particular industry and a record of complaints.
(b) A parent which presents consolidated financial statements is required to consolidate all it s
components in the consolidated financial statements other than those for which exceptions
have been provided in the relevant accounting standards under the applicable financial
reporting framework.
The auditor should obtain a listing of all the components included in the consolidated
financial statements and review the information provided by the management of the parent
identifying the components. The auditor should verify that all the components have been
included in the consolidated financial statements unless these components meet criterion
for exclusion.
In the given case, Excellent Ltd has provided the list of components included in the
consolidated financial statements (CFSs). CA Vimal shall verify that all the components
have been included in the CFSs.
Further, in respect of completeness of this information, CA Vimal should perform the
following procedures:
i. review his working papers for the prior years for the known components;
ii. review the parent’s procedures for identification of various components;
iii. make inquiries of the management to identify any new components or any component
which goes out of consolidated financial statements;
iv. review the investments of parent as well as its components to determine the
shareholding in other entities;
v. review the joint ventures and joint arrangements as applicable;
vi. review the other arrangements entered into by the parent that have not been included
in the consolidated financial statements of the group;
vii. review the statutory records maintained by the parent, for example registers under
section 186, 190 of the Companies Act, 2013;
viii. Identify the changes in the shareholding that might have taken place during the
reporting period.

© The Institute of Chartered Accountants of India


12 FINAL (NEW) EXAMINATION: NOVEMBER 2020

(c) According to clause (xiii) of Para 3 of CARO, 2016, whether all transactions with the
related parties are in compliance with sections 177 and 188 of Companies Act, 2013 where
applicable and the details have been disclosed in the Financial Statements etc., as
required by the applicable accounting standards.
Therefore the duty of the auditor, under this clause is to report
i. Whether all transactions with the related parties are in compliance with section 177
and 188 of the Companies Act,2013:
ii. Whether related party disclosures as required by relevant Accounting Standards (AS
18, as may be applicable) are disclosed in the financial statements.
In the Instant case, MG Associates is a related party and also rendering marketing services
to MM Ltd. in return of Consideration of ` 4 Lakhs which is related party transaction. No
separate disclosure has been made in the notes to accounts in this context, which was
required to be made.
In view of above, Auditor shall report under the above clause as under:
1. Nature of the related party relationship and the underlying transaction-MG
Associates is a partnership firm in which Director of MM Ltd is also a managing
partner, with a profit sharing ratio of 30 %. Payment of ` 4 Lakhs to MG Associates
is a related party transaction.
2. Amount involved is Consideration for the Marketing services rendered by MG
Associates (` 4 Lakhs p.m.) is higher than the arm’s length pricing by ` 1.50 Lakh
p.m. (` 18 Lakhs p.a.)
Question 5
(a) The management auditor has to analyse the nature and causes of behavioural problems
in the discharge of the review function and to arrive at possible solutions to overcome these
problems. As a management auditor of Real Limited, how will you demonstrate in ar riving
at a solution to behavioural problems and create an atmosphere of trust and frien dliness,
so that audit reports will be understood in their proper perspective? (5 Marks)
(b) BG Limited is a large-sized listed company. The Board of directors have constituted
Nomination and Remuneration committee comprising of non-executive and independent
directors. The management seeks your advice on the composition and role of the
committee. Elucidate the composition and role of Nomination and Remuneration committee
as per SEBI (Listing Obligations and Disclosure Requirements) Regulations, 2015.
(5 Marks)
(c) Comment with reference to the Chartered Accountants Act, 1949 and schedules thereto:
Mr. Vineet, a chartered accountant in practice, created his own website in attractive format
and highlighted the contents in purple colour. The website also displayed the nature of
assignments handled along with the names of clients without such requirement from any

© The Institute of Chartered Accountants of India


PAPER – 3 : ADVANCED AUDITING AND PROFESSIONAL ETHICS 13

of the regulator. He also circulated the information contained in the website through
e-mail to acknowledge public at large about his expertise. However, he did not intimate his
website address to the Institute. (4 Marks)
OR
(c) CA Vipin has been appointed as Statutory Auditor by IG Insurance Co. Ltd. for 3 of its
branches for the F.Y. 2019-2020. Insurance Company is using a software called "Applied
Epic" wherein all transactions (policy issuance, premium receipts, expense of insurance
company, incomes, assets and liabilities) are recorded and financial statements generated
at the end of the financial year. CA Vipin not technically equipped and well versed with
technology, decided to follow traditional manual auditing approach and started the audit.
He is of the view that understanding and using the auditee's automated environment is
optional and not required. Do you agree with the approach and views of CA Vipin?
Answer
(a) The auditors, if they were to adopt the role of accuser or secret agency of the management
to try upon the happenings of the auditee division, they would be unwelcome. Relations
between the auditor and the auditee may improve if the auditor acts and is per ceived as a
professional advisor and consultant.
Management auditor of Real Limited shall demonstrate, in arriving at a solution, behavioral
problems in the discharge of the review function.
In this case, there is a need to demonstrate to the extent possible that:
1. the audit is part of an overall programme mandated by higher- level authority to meet
higher-level organizational needs for both protection and maximum constructive
benefit.
2. the objective of the review is to provide maximum service in all feasible managerial
dimensions.
3. the review will be conducted with minimum interference with regular operations of the
operating personnel.
4. the responsible officers will be kept fully informed and have an opportunity to review
findings and recommendations before any audit report is formally released.
It is essential to create an atmosphere of trust and friendliness so that audit reports will be
understood in their proper perspective.
1. Constructive criticism - It is essential that the auditor should concentrate only on
constructive criticism. He should also make obvious in his report the value of his
comments in tangible terms. Only then would suggestions carry weight with the
auditees and they will feel convinced that the auditor has been objective in his
remarks in the report.

© The Institute of Chartered Accountants of India


14 FINAL (NEW) EXAMINATION: NOVEMBER 2020

2. Reporting methods - To achieve this objective, the auditor has to make a concerted
effort to convey effectively his role by adopting a friendly but firm tone in his report. It
is always possible to disagree without being disagreeable, to criticize without being
critical. The reports should concentrate on areas which need improvement rather than
listing inefficiencies and deficiencies in performance of the auditee.
3. Participative approach - It is well established that auditor’s reports have better
acceptability if the improvements suggested are discussed with those who have to
implement them and made to feel that they have participated in the recommendations
made for improvements. On the other hand, it has been observed that either oral or
written appreciation of the auditee’s achievements not only encourages the auditees
to develop a friendly attitude towards the auditors but look forward to their guidance
in a more receptive fashion.
The participative approach to the internal audit process has proved to be success. Feelings
of hostility disappear giving room to feelings of mutual trust. Team spirit is developed.
Proposed recommendations are discussed with the auditee and such modifications as may
be mutually agreed upon are incorporated.
(b) Advice to the Management of BG Limited on the Composition of Nomination and
Remuneration Committee
i. The Board of Directors of every listed public company shall constitute the Nomination
and Remuneration Committee which shall comprise of at least three directors, all of
whom shall be non-executive directors and at least half shall be independent
directors,
ii. However, in case of a listed entity having outstanding SR equity shares, two thirds
shall comprise of independent directors.
iii. Chairperson of the committee shall be an independent director. It may be noted that
the Chairperson of the company (whether executive or nonexecutive) may be
appointed as a member of the Nomination and Remuneration Committee but shall not
chair such committee.
Advice to the Management of BG Limited on the Role of Nomination and Remuneration
Committee.
The role of such committee shall, inter-alia, include the following:
(i) Formulation of the criteria for determining qualifications, positive attributes and
independence of a director and recommend to the Board of Directors a policy, relating
to the remuneration of the directors, key managerial personnel and other employees;
(ii) Formulation of criteria for evaluation of performance of independent directors and the
Board of Directors;

© The Institute of Chartered Accountants of India


PAPER – 3 : ADVANCED AUDITING AND PROFESSIONAL ETHICS 15

(iii) Devising a policy on Board diversity;


(iv) Identifying persons who are qualified to become directors and who may be appointed
in senior management in accordance with the criteria laid down, and recomm end to
the Board their appointment and removal;
(v) whether to extend or continue the term of appointment of the independent director,
on the basis of the report of performance evaluation of independent directors.
(vi) recommend to the board, all remuneration, in whatever form, payable to senior
management.
(c) Circulating Information Contained in Own Website: As per Clause (6) of Part I of the
First Schedule to the Chartered Accountants Act, 1949, a Chartered Accountant in practice
is deemed to be guilty of professional misconduct if he solicits clients or professional work
either directly or indirectly by circular, advertisement, personal communication or interview
or by any other means.
1. However, the guidelines approved by the Council of the Institute of Chartered
Accountants of India permit creation of own website by a chartered accountant in his
or his firm name and no standard format or restriction on colours is there. Hence there
is no misconduct as per Clause (6) of Part I of the First Schedule to the Chartered
Accountants Act, 1949.
2. The chartered accountant or firm, as per the guidelines, should ensure that none of
the information contained in the website be circulated on their own or through E-mail
or by any other mode except on a specific “Pull” request. Mr. Vineet has circulated
the information contained in the website through e-mail to public at large. Therefore,
he is guilty of professional misconduct under Clause (6) of Part I of the First Schedule
to the said Act.
3. Nature of assignments handled (to be displayable only on specific “pull” request).
Names of clients and fee charged cannot be given without such requirement from any
of the regulator. Mr. Vineet has displayed the nature of assignments handled along
with the name of clients without such requirement from the regulator. Therefore, he
is guilty of professional misconduct under Clause (6) of Part I of the First Schedule to
the said Act.
4. "The website address of the member be obtained on annual basis in the annual form
required to be filed by the member while paying fee and the same be taken as entry
on record." Thus guilty of professional misconduct
(c) As per SA315, understanding of the automated environment of a company is required. The
auditor’s understanding of the automated environment should include the following:
i. The applications that are being used by the company;
ii. Details of the IT infrastructure components for each of the application;

© The Institute of Chartered Accountants of India


16 FINAL (NEW) EXAMINATION: NOVEMBER 2020

iii. The organisation structure and governance;


iv. The policies, procedures and processes followed;
v. IT risks and controls.
The auditor is required to document the understanding of a company’s automated
environment as per SA 230.
Thus the approach of CA Vipin is not correct considering the above mentioned
requirements of SA 315 and SA 230.
Question 6
(a) CA Nadar is conducting the statutory audit of RHL Ltd., a non-banking financial company.
It has branches in various parts of India. The company with a focus on housing finance,
has outstanding non convertible debentures worth ` 150 Crores. The company reportedly
missed interest payments of INR 15 Crores on its debts because of inadequate liquidity.
As a result, RHL Ltd. faced a series of downgrades by rating agencies on its debts over
the past two months. Rating was cut to D from A4 implying that the company was in default
or expected to be in default soon. What aspects CA Nadar should look into in relation to
the activity of mobilization of public deposits (particularly in relation to downgrading of
credit facilities) by RHL Ltd? (5 Marks)
(b) In the course of audit of Quick Ltd, you suspect that the management has made
misstatements in the financial statements intentionally to deceive the users and to
succumb to pressures to meet market expectations. Elucidate how the fraudulent financial
reporting may be accomplished and also discuss the techniques of committing fraud by
management overriding controls. (5 Marks)
(c) CA Robo has been appointed as Forensic Auditor by BMY Bank Limited for one of its
borrowal accounts WRONG Ltd. CA Robo started the audit by first reviewing the
transactions of the borrower in Bank statement as per Bank records to identify any hidden
patterns in that information. She had to review huge volume of data, as the number of
transactions per day were in hundreds and the data was to be reviewed for the last three
years. So, she was stuck up as to how to proceed further to identify any hidden patterns in
information, if any. Guide CA Robo, suggesting which technique to be used for identifying
any hidden patterns in the information. (4 Marks)
Answer
(a) CA Nadar has to ascertain whether the company has complied with the following
aspects in relation to the activity of mobilization of public deposits: -
i. The ceiling on quantum of public deposits has been linked to its credit rating as given
by an approved credit rating agency. In the event of a upgrading/downgrading of
credit rating, the auditor should bear in mind that the NBFC will have to
increase/reduce its public deposits in accordance with the revised credit rating

© The Institute of Chartered Accountants of India


PAPER – 3 : ADVANCED AUDITING AND PROFESSIONAL ETHICS 17

assigned to it within a specified time frame and should ensure that the NBF Chas
informed about the same to the RBI inwriting.
ii. In the event of downgrading of credit rating below the minimum specified investment
grade, a non-banking financial company, being an investment and credit company or
a factor, shall regularise the excess deposit as provided hereunder:
a. with immediate effect, stop accepting fresh public deposits and renewing
existing deposits;
b. all existing deposits shall run off to maturity; and
c. report the position within 15 working days, to the concerned Regional Office of
the RBI where the NBFC is registered.
d. No matured public deposit shall be renewed without the express and voluntary
consent of the depositor.
(b) In the given case, management of Quick Ltd has made intentional misstatements to
deceive the users in order to meet market expectations. Auditor is suspecting such
intentional behavior of the management and in such situations, SA 240 discusses
how fraudulent financial reporting may be accomplished and also discusses
techniques of committing fraud by management overriding controls.
As per SA 240 on “The Auditor’s Responsibilities Relating to Fraud in an Audit of Financial
Statements”
Fraudulent financial reporting may be accomplished by the following:
i. Manipulation, falsification (including forgery), or alteration of accounting records or
supporting documentation from which the financial statements are prepared.
ii. Misrepresentation in or intentional omission from, the financial statements of events,
transactions or other significant information.
iii. Intentional misapplication of accounting principles relating to amounts, classification,
manner of presentation, or disclosure.
Fraudulent financial reporting often involves management override of controls that
otherwise may appear to be operating effectively. Fraud can be committed by
management overriding controls using such techniques as:
i. Recording fictitious journal entries, particularly close to the end of an accounting
period, to manipulate operating results or achieve other objectives.
ii. Inappropriately adjusting assumptions and changing judgments used to estimate
account balances.
iii. Omitting, advancing or delaying recognition in the financial statements of events and
transactions that have occurred during the reporting period.

© The Institute of Chartered Accountants of India


18 FINAL (NEW) EXAMINATION: NOVEMBER 2020

iv. Concealing, or not disclosing, facts that could affect the amounts recorded in the
financial statements.
v. Engaging in complex transactions that are structured to misrepres ent the financial
position or financial performance of the entity.
vi. Altering records and terms related to significant and unusual transactions.
(c) Data Mining Techniques:
i. Data mining techniqueis a set of assisted techniques designed to automatically mine
large volumes of data for new, hidden or unexpected information or patterns.
ii. It discovers the usual knowledge or patterns in data, without a predefined idea or
hypothesis about what the pattern may be, i.e. without any prior knowledge of fraud.
iii. It explains various affinities, association, trends and variations in the form of
conditional logic.
iv. Data mining techniques are categorized in three ways: Discovery, Predictive
modeling and Deviation and Link analysis.
In the given case of BMY Bank Ltd., CA Robo appointed as forensic auditor for its borrower,
WRONG Ltd, shall use above stated data mining techniques to identify any hidden patterns
of information.

© The Institute of Chartered Accountants of India


PAPER – 3 : ADVANCED AUDITING AND PROFESSIONAL ETHICS
Question No. 1 is compulsory
Answer any four from the rest.
Question 1
(a) M/s NK & Co., Chartered Accountants were appointed as Statutory Auditors of Fresh Juice
Limited for the F.Y 2019-2020. The previous year's audit was conducted by M/s. LP &
Associates. After the audit was completed and report submitted, it was found that closing
balances of last financial year i.e., 2018-19 were incorrectly brought forward. It was found
that M/s NK & Co. did not apply any audit procedures to ensure that correct opening
balances have been brought forward to the current period.
Accordingly, a complaint was filed against NK & Co. in relation to this matter.
You are required to inform what policies are required to be implemented by NK & Co. for
dealing with such complaints and allegations as required by Standard on Quality Control
(SQC). (5 Marks)
(b) GHK Associates, Chartered Accountants, conducting the audit of PBS Ltd., a listed
company for the year ended 31.03.2020 is concerned with the presentation and disclosure
of segment information included in Company's Annual Report. GHK Associates want to
ensure that methods adopted by management for determining segment information have
resulted in disclosure in accordance with the applicable financial reporting framework.
Guide GHK Associates with 'Examples of Matters' that may be relevant when obtaining an
understanding of the methods used by the management with reference to the relevant
Standards on Auditing. (5 Marks)
(c) Nam & Co., conducted Stock Audit of DEF Ltd. as per instructions issued by HEG Bank.
However instead of visiting the site where the stock was lying, the firm relied on the
Management Information Systems report along with inspections reports and photographs
of Stock taken by the employees of DEF Ltd. The photographs were also carrying the date
and time printed on them. Comment with reference to the Chartered Accountants Act, 1949
and its schedules thereto. (4 Marks)
Answer
(a) In the given question, NK & Co. did not apply audit procedures to ensure that opening
balances had been correctly brought forward. A complaint was filed against the auditors in
this context. As per Standard on Quality Control (SQC) 1 “Quality Control for Firms that
Perform Audits and Reviews of Historical Financial Information, and Other Assurance and
Related Services Engagements”,
(i) The firm should establish policies and procedures designed to provide it with
reasonable assurance that it deals appropriately with:

© The Institute of Chartered Accountants of India


2 FINAL (NEW) EXAMINATION: JANUARY 2021

(a) Complaints and allegations that the work performed by the firm fails to comply
with professional standards and regulatory and legal requirements; and
(b) Allegations of non-compliance with the firm’s system of quality control.
(ii) Complaints and allegations (which do not include those that are clearly frivolous) may
originate from within or outside the firm. They may be made by firm personnel, clients
or other third parties. They may be received by engagement team members or other
firm personnel.
(iii) As part of this process, the firm establishes clearly defined channels for firm
personnel to raise any concerns in a manner that enables them to come forward
without fear of reprisals.
(iv) The firm investigates such complaints and allegations in accordance with established
policies and procedures. The investigation is supervised by a partner with sufficient
and appropriate experience and authority within the firm but who is not otherwise
involved in the engagement, and includes involving legal counsel as necessary. Small
firms and sole practitioners may use the services of a suitably qualified external
person or another firm to carry out the investigation. Complaints, allegations and the
responses to them are documented.
(v) Where the results of the investigations indicate deficiencies in the design or operation
of the firm’s quality control policies and procedures, or non-compliance with the firm’s
system of quality control by an individual or individuals, the firm takes appropriate
action.
(b) The auditors, GHK Associates wanted to ensure and obtain sufficient appropriate audit
evidence regarding the presentation and disclosure of segment information in accordance
with the applicable financial reporting framework by obtaining an understanding of the
methods used by management in determining segment information. SA 501 guides in this
regard. As per SA 501- “Audit Evidence—Specific Considerations for Selected Items”,
example of matters that may be relevant when obtaining an understanding of the methods
used by management in determining segment information and whether such methods are
likely to result in disclosure in accordance with the applicable financial reporting framework
include:
(i) Sales, transfers and charges between segments, and elimination of inter-segment
amounts.
(ii) Comparisons with budgets and other expected results, for example, operating profits as
a percentage of sales.
(iii) The allocation of assets and costs among segments.
(iv) Consistency with prior periods, and the adequacy of the disclosures with respect to
inconsistencies.

© The Institute of Chartered Accountants of India


PAPER – 3 : ADVANCED AUDITING AND PROFESSIONAL ETHICS 3

(c) According to Clause (7) of Part I of Second Schedule to the Chartered Accountants Act,
1949, a Chartered Accountant in practice is deemed to be guilty of professional misconduct
if he “does not exercise due diligence or is grossly negligent in the conduct of his
professional duties”.
It is a vital clause which usually gets attracted whenever it is necessary to judge whether
the accountant has honestly and reasonably discharged his duties. The expression
negligence covers a wide field and extends from the frontiers of fraud to collateral minor
negligence.
In the instant case, CA. Nam &Co. did not exercise due diligence and is grossly negligent
in the conduct of his professional duties since it did not visit the site where the stock was
lying and instead the firm relied on the MIS report along with inspection reports and
photographs of stock taken by the employees of DEF Ltd, which is incorrect.
To conduct stock audit, ascertainment of existence and physical condition of stocks, cross
tallying the stock with Stock statement submitted by bank borrower, correct classification
of stocks for valuation purpose etc. is essential. Further submitting stock audit report
without physically verifying the stock amounts to gross negligence.
From the above, it can be concluded that Nam & Co. is guilty of professional misconduct
under Clause (7) of Part I of Second Schedule to the Chartered Accountants Act, 1949.
Question 2
(a) Auditors are required to obtain an understanding of internal control relevant to the audit
when identifying and assessing its effectiveness and risk of material misstatement. During
the course of audit of ABC Ltd., you observed that significant deficiency exists in the
internal control system and you want to ascertain the same. Elucidate the various
indicators of significant deficiencies which will help you in assessing the efficiency of
internal control system of the organization. (5 Marks)
(b) A European company engaged in the business of manufacturing and distribution of
industrial gases, is interested in acquiring a listed Indian Company having a market share
of 51% and assets over ` 1000 Crores. It requests you to conduct "Due Diligence" of assets
of this Indian Company to find out, if any of the assets is overvalued. List down the areas
of due diligence exercise to find out overvalued assets. (5 Marks)
(c) M/s. NKB Ltd. is engaged in the manufacturing of textile products having an annual
capacity of producing 1,00,000 units of garments. NKB Ltd. is covered under the provisions
of Goods and Service Tax Act with an applicable rate of 12%. During the financial year
2019-2020, NKB Ltd. received a demand notice of ` 15.00 Lacs pertaining to the F.Y
2013-14 when the provisions of Central Excise Act were applicable. NKB Ltd. deposited
the demand amount after discussing with its legal department. Are you, as a tax auditor of
NKB Ltd., required to report the same? (4 Marks)

© The Institute of Chartered Accountants of India


4 FINAL (NEW) EXAMINATION: JANUARY 2021

Answer
(a) In the given case of ABC Ltd, Auditors, while conducting audit has come across significant
deficiency existing in the internal control system and also auditors wanted to ascertain that
deficiency.
As per SA 265, “Communicating Deficiencies in Internal Control to Those Charged with
Governance and Management “, Indicators of significant deficiencies in internal control include,
for example:
(i) Evidence of ineffective aspects of the control environment, such as:
(a) Indications that significant transactions in which management is financially
interested are not being appropriately scrutinised by those charged with
governance.
(b) Identification of management fraud, whether or not material, that was not prevented
by the entity’s internal control.
(c) Management’s failure to implement appropriate remedial action on significant
deficiencies previously communicated.
(ii) Absence of a risk assessment process within the entity where such a process would
ordinarily be expected to have been established.
(iii) Evidence of an ineffective entity risk assessment process, such as management’s
failure to identify a risk of material misstatement that the auditor would expect the
entity’s risk assessment process to have identified.
(iv) Evidence of an ineffective response to identified significant risks (e.g., absence of
controls over such a risk).
(v) Misstatements detected by the auditor’s procedures that were not prevented, or
detected and corrected, by the entity’s internal control.
(vi) Disclosure of a material misstatement due to error or fraud as prior period items in
the current year’s Statement of Profit and Loss.
(vii) Evidence of management’s inability to oversee the preparation of the financial
statements.
(b) A European company which is manufacturing and distributing industrial gases is looking
forward to acquire an Indian company having 51% market share and assets beyond ` 1000
crores. Areas to be covered as a part of due diligence exercise to find out over valued
assets would be as under:
1. Uncollected/uncollectable receivables.
2. Obsolete, slow non-moving inventories or inventories valued above NRV; huge
inventories of packing materials etc. with name of company.

© The Institute of Chartered Accountants of India


PAPER – 3 : ADVANCED AUDITING AND PROFESSIONAL ETHICS 5

3. Underused or obsolete Plant and Machinery and their spares; asset values which
have been impaired due to sudden fall in market value etc.
4. Assets carried at much more than current market value due to capitalization of
expenditure/foreign exchange fluctuation, or capitalization of expenditure mainly in
the nature of revenue.
5. Litigated assets and property.
6. Investments carried at cost though realizable value is much lower.
7. Investments carrying a very low rate of income / return.
8. Infructuous project expenditure/deferred revenue expenditure etc.
9. Group Company balances under reconciliation etc.
10. Intangibles of no value.
(c) NKB Ltd. Is a manufacturer of textile products and is covered under GST Act. During
financial year 2019-2020 NKB has received a demand notice of 15 lakhs which pertains to
financial year 2013-2014 when the Central Excise Act was prevalent. As a tax auditor of
NKB Ltd., reporting would be under Clause 41 which is given hereunder:
“Please furnish the details of demand raised or refund issued during the previous year
under any tax laws other than Income Tax Act, 1961 and Wealth tax Act, 1957 along with
details of relevant proceedings. “
It may be noted that even though the demand/refund order is issued during the previous
year, it may pertain to a period other than the relevant previous year. In such cases also,
reporting has to be done under this clause. If there is any adjustment of refund against any
demand, the auditor shall also report the same under this clause.
In this case, liability is of excise duty i.e. under Central Excise Act, other than Income Tax
Act and Wealth Tax Act, thus this clause gets attracted and the reporting has to be done
as per format:
S No. Name of Demand/ Date of Financial Amount of Adjustment Remarks
the Refund demand year to demand/raise of refund
Applicable Order raised/ref which the d/refund against
Act no., if any und demand/ref issued demand, if
issued und relates any

Question 3
(a) You are engaged by M/s. Real Ltd. as an internal auditor for the financial year 2020-2021.
While applying risk assessment procedures of inquiring from management and various
analytical procedures, you have identified some risks which in your opinion may le ad to

© The Institute of Chartered Accountants of India


6 FINAL (NEW) EXAMINATION: JANUARY 2021

material misstatement at the financial level and assertion level. Which factors as an auditor
will you consider while exercising judgement as to whether such risks are significant risks
or not? (5 Marks)
(b) LDH Ltd., a company incorporated in India and Listed on a recognized Stock Exchange in
India has entered into various related parties transactions during the financial year. You
are required to answer the following keeping in mind the Listing Obligations and Disclosure
Requirements (LODR) on corporate Governance.
(i) Who should sign the report of material transactions with related parties? (1 Mark)
(ii) What type of transactions and policy are required to be disclosed in relation to related
party transactions? (2 Marks)
(iii) Whether disclosures of related party transactions on consolidated financial
statements are required to be made? If yes, what are the guidelines? (2 Marks)
(c) Please state which of the following is to be reported under Form 9C under GST audit for
the period 2019-2020 ?
(i) The GST registered firm donates the old washing plant as on 25.12.2019 to a
charitable institute. This machinery was purchased on 10.05.2018 for ` 1,75,000 Plus
GST @ 12%.
(ii) Goods Transport Agent (GTA) issued a consignment note on 05.04.19. The
consignment note does not charge GST. The consignor has booked the GTA. The
recipient has paid the freight to GTA on 'to collect' basis.
(iii) Sale of a building after completion certification has been obtained for ` 50,00,000 on
14.11.2019. (1 + 1 + 2 = 4 Marks)
Answer
(a) The internal auditor of Real Ltd. has identified some risks while he was applying risk
assessment procedures and various analytical procedures.
As per SA 315, “Identifying and Assessing the Risks of Material Misstatements through
Understanding the Entity and its Environment”, in exercising judgment as to which risks
are significant risks, the auditor shall consider at least the following:
(1) Whether the risk is a risk of fraud;
(2) Whether the risk is related to recent significant economic, accounting, or other
developments like changes in regulatory environment, etc., and, therefore, requires
specific attention;
(3) The complexity of transactions;
(4) Whether the risk involves significant transactions with related parties;

© The Institute of Chartered Accountants of India


PAPER – 3 : ADVANCED AUDITING AND PROFESSIONAL ETHICS 7

(5) The degree of subjectivity in the measurement of financial information related to the
risk, especially those measurements involving a wide range of measurement
uncertainty; and
(6) Whether the risk involves significant transactions that are outside the normal course of
business for the entity, or that otherwise appear to be unusual.
(b) An Indian company, LDH Ltd., listed on stock exchange entered into various related
party transactions.
(i) The report shall be signed either by the compliance officer or the chief executive
officer of the listed entity.
(ii) (a) The company shall disclose the policy on dealing with related party transactions
on its website and a web link thereto shall be provided in the Annual Report.
(b) The listed entity shall disclose the transactions with any person or entity
belonging to the promoter/ promoter group which hold(s) 10% or more
shareholding in the listed entity, in the format prescribed in the relevant
accounting standards for annual results.
(iii) (a) Yes, disclosures of related party transactions on consolidated financial
statements are required to be made by the listed entity within 30 days from the
date of publication of its standalone and consolidated financial results for the
half year.
(b) The listed entity shall disclose related party transactions on a consolidated
basis, in the format specified in the relevant accounting standards for annual
results to the stock exchanges and publish the same on its website.
(c) (i) Permanent Transfer or disposal of business assets where input tax credit has been
availed on such assets should be reported in table 5D as Deemed supply under
Schedule I. The current value of washing plant in books should be reported as
deemed sale in Table 5D.
(ii) Since consignment note has not charged GST @ 12%, reverse charge provisions
would apply. Tax is to be paid by the person liable to pay freight, that is, the recipient
and not the GTA under forward charge. Because of this, the impugned transaction
has to be entered in Table 7D.
(iii) (a) No supplies include the activities covered under Schedule III which are neither
a supply of goods nor a supply of services. Examples- Sale of land or completed
building, actionable claims, other than lottery, betting, and gambling.
(b) Table 7B requires reduction of value of Exempted, Nil rated, Non-GST supplies,
No-Supply turnover from the Annual turnover after adjustments to arrive at
taxable turnover. Therefore, Sale of a building after completion certificate has
been obtained is treated as no supply under GST law. The same has to be
reported under Table 7B in form no 9C.

© The Institute of Chartered Accountants of India


8 FINAL (NEW) EXAMINATION: JANUARY 2021

Question 4
(a) CA N was appointed as an auditor of JAL Ltd. The company has branches all over the
state of Haryana. CA N, in consultation with management, decided to Visit 6 out of 10
branches. Management decided to pay him advance of ` 2.00 Lacs against the estimated
expenses of ` 2.50 Lacs on visits to be conducted as a part of services rendered. As
agreed, ` 2.00 Lacs was transferred in his bank account from which he met all the
expenses. Comment with reference to Chartered Accountants Act, 1949 whether the action
of CA N of receiving the advance money in his saving accounts and not keeping it in
separate bank account is valid. (4 Marks)
(b) The Comptroller and Auditor General of India has appointed a chartered accountant firm
to conduct the comprehensive audit of Metro Company Limited (a listed government
company) which is handling the Metro project of the metropolitan city for the period end ing
31-03-2020. The work to be conducted under Project A handled by the Metro Company
Limited was of laying down railway line of 124 kilometres. [The chartered accountant firm
reviewed the internal audit report and observed the shortcoming reported about the
performance of Project A regarding the understatement of the Current liabilities and Capital
work in progress by ~ 84.68 crore.] Explain some of the matters to be undertaken by the
chartered accountant firm while conducting the comprehensive audit of Metro Company
Limited. (5 Marks)
(c) JRS Limited holds the majority ownership of R Ltd. & K Ltd. S Ltd. is an intermediate
subsidiary of JRS Limited in Surat. The JRS Limited presents the consolidated financial
statements for audit purposes to MMT & Co. As a statutory auditor MMT & Co. obtain a
listing of all the components and verify that all the components included in financial
statements unless any component meet criterion for exclusion. Explain any two reasons
which are considered by MMT & Co. for exclusion of components from the consolidated
financial statements and reporting of reasons of exclusion thereof. (5 Marks)
Answer
(a) As per Clause (10) of Part I of Second Schedule to the Chartered Accountant Act,
1949, a Chartered Accountant in practice will be deemed to be guilty of professional
misconduct if he fails to keep moneys of his client other than the fees or remuneration or
money meant to be expended in a separate banking account or to use such moneys for
purposes for which they are intended within a reasonable time.
In the course of his engagement as a professional accountant, a member may be entrusted
with moneys belonging to his client. If he should receive such funds, it would be his duty
to deposit them in a separate banking account, and to utilize such funds only in accordance
with the instructions of the client or for the purposes intended by the client.
In this connection the Council has considered some practical difficulties of the members
and the following suggestion, among other suggestions, has been made to remove these
difficulties:

© The Institute of Chartered Accountants of India


PAPER – 3 : ADVANCED AUDITING AND PROFESSIONAL ETHICS 9

“An advance received by a Chartered Accountant against services to be rendered does not
fall under Clause (10) of Part I of the Second Schedule”
In the given case, CA N was given an advance of ` 2 Lakhs against the estimated expenses
of ` 2.50 Lakhs on visits to be conducted as a part of services rendered.
Applying the above, it can be concluded that CA N is not guilty of professional misconduct
under Chartered Accountants Act, 1949.
(b) A CA Firm has been appointed to conduct comprehensive audit of Metro Company Limited,
which is a listed Govt Company handling the Metro project. CA firm has observed the
shortcomings as stated in internal audit report regarding understatement of Current
liabilities and CWIP by ` 84.68 crore.
Matters to be undertaken by the CA Firm while conducting the comprehensive audit
of Metro Company Limited are:
(i) How does the overall capital cost of the project compare with the approved planned
costs? Were there any substantial increases and, if so, what are these and whether
there is evidence of extravagance or unnecessary expenditure?
(ii) Have the accepted production or operational outputs been achieved? Has there been
under-utilisation of installed capacity or shortfall in performance and, if so, what has
caused it?
(iii) Has the planned rate of return been achieved?
(iv) Are the systems of project formulation and execution sound? Are there inadequacies?
What has been the effect on the gestation period and capital cost?
(v) Are cost control measures adequate and are there inefficiencies, wastages in raw
materials consumption, etc.?
(vi) Are the purchase policies adequate? Or have they led to piling up of inventory
resulting in redundancy in stores and spares?
(vii) Does the enterprise have research and development programmes? What has been
the performance in adopting new processes, technologies, improving profits and in
reducing costs through technological progress?
(viii) If the enterprise has an adequate system of repairs and maintenance?
(ix) Are procedures effective and economical?
(x) Is there any poor or insufficient or inefficient project planning?

© The Institute of Chartered Accountants of India


10 FINAL (NEW) EXAMINATION: JANUARY 2021

(c) Where a component is excluded from the consolidated financial statements, the
auditor should examine the reasons for exclusion and whether such exclusion is in
conformity with the applicable financial reporting framework.
(i) Under Companies (Accounting Standards) Rules, 2006, there could be two reasons
for exclusion of subsidiary, associate or jointly controlled entity - one, that the
relationship of parent with the subsidiary, associate or jointly controlled entity is
intended to be temporary or the subsidiary, associate or joint venture operates under
severe long-term restrictions which significantly impair its ability to transfer funds to
the parent.
(ii) Similarly, under the Companies Act, 2013, intermediate subsidiary in India is not
required to present consolidated financial statements. Ind AS 110 also prescribes
certain criteria where consolidated financial statements are not required. In such cases,
the auditor should satisfy himself that the exclusion made by the management falls
within these categories, example in the case of an entity which is excluded from
consolidation on the ground that the relationship of parent with the other entity as
subsidiary, associate or joint venture is temporary, the auditor should verify that the
intention of the parent, to dispose off the subsidiary, investment in associate or interest
in jointly controlled entity, in the near future, existed at the time of acquisition of the
subsidiary, making investment in associate or jointly controlled entity.
(iii) The auditor should also verify that the reasons for exclusion are given in the
consolidated financial statements. If an entity is excluded from the consolidated
financial statements for reasons other than those allowed by the applicable financial
reporting framework, the auditor should consider its effect on the auditor’s report to
be issued.
Question 5
(a) M/s. HK & Co. was appointed as an auditor of GSB Limited, a company operating its
business in telecom sector. As per spectrum allocation agreement with Government, GSB
Limited is required to pay certain percentage of its annual revenue as license fee. GSB
Limited paid the license fee on its core business for last two years. At the end of third year,
the communication was received from Government that it needs to pay agreed percentage
on its total revenues and not only on core business revenues. Matter was disputed a nd
went to court of law. On prudence basis, GSB Limited made a provision on estimated
business in its books of accounts of agreed percentage on non-core business receipts
also. The amount of provision was of such huge amount that the GSB Limited's profit a nd
loss account for that quarter reflected loss due to that provision. How you as an auditor
can evaluate this accounting estimate which involves significant risk and what if
Management has not addressed the effects of estimation uncertainty on provision m ade?
(4 Marks)

© The Institute of Chartered Accountants of India


PAPER – 3 : ADVANCED AUDITING AND PROFESSIONAL ETHICS 11

(b) The audit team is preparing to conduct audit for ABC Company for the period ending
31.3.2020. However, the audit team has not received its audit fees from ABC Company for
its audit concluded for the period ended 31.3.2019. The audit team might be tempted to
issue a favorable report so that ABC Company is able to secure a loan to settle the fees
outstanding for their 31.3.2019 audit. The audit team is not complying the fundamental
principles of auditing hence hindering the Auditor's Independence. Explain the types of
threats that may hinder Auditor's Independence while issuing Audit Report. (5 Marks)
(c) Explain the concept of Integrated framework issued by Committee of the Sponsoring
Organisations of the Treadway Commission (COSO Framework) duly mentioning its four
out of five components and discuss the three category of objectives that can be achieved
as per COSO framework. (5 Marks)
Answer
(a) In the given case, HK & Co. was appointed as an auditor of GSB Ltd., operating in Telecom
sector. GSB Ltd paid the license fee on its core business revenue whereas Govt required
it to pay on non-core business receipts as well. Consequently, the amount of provision was
of such a huge amount that GSB Ltd.’s profit and loss account reflected a lo ss due to that
provision. As an auditor evaluation would be done as under:
For accounting estimates that give rise to significant risks, in addition to other substantive
procedures performed to meet the requirements of SA 330, the auditor shall evaluate t he
following:
(i) How management has considered alternative assumptions or outcomes, and why it has
rejected them, or how management has otherwise addressed estimation uncertainty in
making the accounting estimate.
(ii) Whether the significant assumptions used by management are reasonable.
(iii) Where relevant to the reasonableness of the significant assumptions used by
management or the appropriate application of the applicable financial reporting
framework, management’s intent to carry out specific courses of action and its ability to
do so.
(iv) If, in the auditor’s judgment, management has not adequately addressed the effects of
estimation uncertainty on the accounting estimates that give rise to significant risks, the
auditor shall, if considered necessary, develop a range with which to evaluate the
reasonableness of the accounting estimate.
(b) In the given case of ABC Company, audit team is preparing to conduct its audit for the F.Y
ending on 31.03.2020. Audit firm did not receive its fees for the F.Y ending on 31.03.2019.
Audit team is tempted to issue a favorable report so that auditee can secure a loan to settle
auditor’s outstanding fees. The audit team did not comply with fundamental principles of
auditing and hence compromising Auditor’s Independence.
Compliance with the fundamental principles may potentially be threatened by a broad
range of circumstances. Many threats fall into the following categories:

© The Institute of Chartered Accountants of India


12 FINAL (NEW) EXAMINATION: JANUARY 2021

The Code of Ethics for Professional Accountants prepared by the International Federation
of Accountants (IFAC) identifies five types of threats. These are:
1. Self-interest threats, which occur when an auditing firm, its partner or associate
could benefit from a financial interest in an audit client. Examples include (i) direct
financial interest or materially significant indirect financial interest in a client, (ii) loan
or guarantee to or from the concerned client, (iii) undue dependence on a client’s fees
and, hence, concerns about losing the engagement, (iv) close business relationship
with an audit client, (v) potential employment with the client, and (vi) contingent fees
for the audit engagement.
2. Self-review threats, which occur when during a review of any judgement or
conclusion reached in a previous audit or non-audit engagement (Non audit services
include any professional services provided to an entity by an auditor, other than audit
or review of the financial statements. These include management services, internal
audit, investment advisory service, design and implementation of information
technology systems etc.), or when a member of the audit team was previously a
director or senior employee of the client. Instances where such threats come into play
are (i) when an auditor is having recently been a director or senior officer of the
company, and (ii) when auditors perform services that are themselves subject matters
of audit.
3. Advocacy threats, which occur when the auditor promotes, or is perceived to
promote, a client’s opinion to a point where people may believe that objectivity is
getting compromised, and e.g. when an auditor deals with shares or securities of the
audited company, or becomes the client’s advocate in litigation and third party
disputes.
4. Familiarity threats are self-evident, and occur when auditors form relationships with
the client where they end up being too sympathetic to the client’s interests. This can
occur in many ways: (i) close relative of the audit team working in a senior position in
the client company, (ii) former partner of the audit firm being a director or senior
employee of the client, (iii) long association between specific auditors and their
specific client counterparts, and (iv) acceptance of significant gifts or hospitality from
the client company, its directors or employees.
5. Intimidation threats, which occur when auditors are deterred from acting objectively
with an adequate degree of professional skepticism. Basically, these could happen
because of threat of replacement over disagreements with the application of
accounting principles, or pressure to disproportionately reduce work in response to
reduced audit fees.
(c) Concept of COSO:
COSO’s Internal Control – Integrated Framework was introduced in 1992 as guidance on
how to establish better controls so companies can achieve their objectives. COSO

© The Institute of Chartered Accountants of India


PAPER – 3 : ADVANCED AUDITING AND PROFESSIONAL ETHICS 13

categorizes entity-level objectives into operations, financial reporting, and compliance. The
framework includes more than 20 basic principles representing the fundamental concepts
associated with its five components: control environment, risk assessment, control
activities, information and communication, and monitoring. Some of the principles include
key elements for compliance, such as integrity and ethical values, authorities and
responsibilities, policies and procedures, and reporting deficiencies.
Five Components of COSO are as follows:
(i) Control Environment
(ii) Risk Assessment
(iii) Control Activities
(iv) Information and Communication
(v) Monitoring
The COSO Framework is designed to be used by organizations to assess the effectiveness
of the system of internal control to achieve objectives as determined by management. The
Framework lists three categories of objectives as below:
1 Operations Objectives – related to the effectiveness and efficiency of the entity’s
operations, including operational and financial performance goals, and safeguarding
assets against loss.
2 Reporting Objectives – related to internal and external financial and non-financial
reporting to stakeholders, which would encompass reliability, timeliness, transparency, or
other terms as established by regulators, standard setters, or the entity’s policies.
3 Compliance objectives – In the Framework, the compliance objective was described as
“relating to the entity’s compliance with applicable laws and regulations.” The Framework
considers the increased demands and complexities in laws, regulations, and accounting
standards.
Question 6
(a) CA K have been doing audit of branch of LUD Bank Ltd. The principal business of the
branch is lending advances to large corporates. Since last one year, many large accounts
have become Non-Performing Asset (NPA) as per guidelines. The Management of the
Bank decided to sell one of the NPA account and consequently one NPA namely DEF Ltd.
amounting to ~ 10.00 Crores was sold to Asset Reconstruction Company. What audit
points CA K should keep in mind while doing audit of this transaction? (5 Marks)
(b) CA S has been appointed as peer reviewer of Shivam & Co. LLP. Shivam & Co. LLP
submitted a list of its assurance and due diligence services for the peer review. CA S is in
the process of deciding as to how many assurance services should be reviewed. Guide
CA S in deciding the number of assurance services engagement to be reviewed. (5 Marks)

© The Institute of Chartered Accountants of India


14 FINAL (NEW) EXAMINATION: JANUARY 2021

(c) TNT Limited is engaged in the Life Insurance business. The company's operations have
been considerable in the Northern India and its Head Office is also based at New Delhi.
TNT Ltd. while preparing financial statements have classified administrative expenses
under 14 heads as mentioned in Schedule 3 forming part of financial statements given
under schedule A to IRDA Regulations, 2002. What is your responsibility as an auditor
particularly in relation to administrative/expenses of management? (4 Marks)
OR
CA AB, a practicing chartered accountant, is a promoter director of ABG Pvt. Ltd. and
moreover he is also a sleeping partner in his family business of garments manufacturing
firm. Is CA. AB liable for professional misconduct as per Chartered Accountant Act 1949?
(4 Marks)
Answer
(a) CA K conducting audit of branch of LUD Bank Ltd. whose principal business is lending
money to large corporates. Many large accounts of this branch have turned NPA category
and Management sold DEF Ltd.’s NPA account amounting to ` 10 Crores to Asset
Reconstruction Company.
CA K should proceed as under:
In case of Sale of NPA by Bank, the auditor should examine
(i) the policy laid down by the Board of Directors in this regard relating to procedures,
valuation and delegation of powers.
(ii) only such NPA has been sold which has remained NPA in the books of the bank for
at least 2 years.
(iii) the assets have been sold “without recourse’ only.
(iv) subsequent to the sale of the NPA, the bank does not assume any legal, operational
or any other type of risk relating to the sold NPAs.
(v) the NPA has been sold at cash basis only.
(vi) on the sale of the NPA, the same has been removed from the books of the account.
(vii) the short fall in the net book value has been charged to the profit and loss account.
(viii) where the sale is for a value higher than the NBV, no profit is recognised and the excess
provision has not been reversed but retained to meet the shortfall/ loss because sale of
other non-performing financial assets.
(b) In the given case, CA S appointed as peer reviewer of Shivam & Co. LLP, is in the process
of deciding as to how many assurance services need to be reviewed.
The number of assurance service engagements to be reviewed shall depend upon:
(i) Standard of quality controls generally prevailing;

© The Institute of Chartered Accountants of India


PAPER – 3 : ADVANCED AUDITING AND PROFESSIONAL ETHICS 15

(ii) The size and nature of assurance service engagements undertaken by the Practice
Unit.
(iii) The methodology generally adopted by the Practice Unit in providing assurance
services.
(iv) The number of partners / members involved in assurance service engagements in
the Practice Unit;
(v) The number of locations / branch offices of the practice Unit;
(vi) The Fees charged / received / service tax paid by the Practice unit.
(c) Operating Expenses related to Insurance Business (Expenses of Management):
(i) Any major expenses (` 5 lacs or in excess of 1% of net premium, whichever is higher)
are required to be shown separately.
(ii) The auditor should ensure that these expenses are first aggregated and then
apportioned to the Revenue Account of each class of business on a reasonable and
equitable basis.
(iii) The accounting policy should clearly indicate the basis of apportionment of these
expenses to the respective Revenue Accounts (i.e., Participating and Non-
participating policies and in between Linked and Non- Linked business) along with
the certificate that all expenses of management, wherever incurred, directly or
indirectly, read with the accounting policy, have been fully debited to the respective
Revenue Account as expenses.
(iv) Any expenses which are not covered under the 14 heads as mentioned in Schedule 3
are required to be disclosed under the head ‘Others’.
OR
(c) Clause (11) of Part I of the First Schedule to the Chartered Accountants Act, 1949
debars a chartered accountant in practice from engaging in any busines s or occupation
other than the profession of chartered accountancy unless permitted by the Council of the
Institute so to engage.
Promoter/Promoter Director - There is no bar for a member to be a promoter / signatory
to the Memorandum and Articles of Association of any company. There is also no bar for
such a promoter / signatory to be a Director Simplicitor of that company irrespective of
whether the object of the company include areas which fall within the scope of the
profession of chartered accounts. Therefore, members are not required to obtain specific
permission of the Council in such cases.
Sleeping partner in the family business-
Prior Approval - Members of the Institute in practice may engage in the following category,
among other points, of business or occupations, after obtaining the specific and prior
approval of the Council in case of:

© The Institute of Chartered Accountants of India


16 FINAL (NEW) EXAMINATION: JANUARY 2021

Interest in family business concerns (including such interest devolving on the members as
a result of inheritance / succession / partition of the family business) or concerns in which
interest has been acquired as a result of relationships and in the management of which no
active part is taken.
In the given case, CA AB is a promoter director of ABG Pvt Ltd and also he is a sleeping
partner in his family business of garments manufacturing firm. Applying the above to the
given case, it can be concluded that-CA AB:
 As Promoter Director- Not guilty of professional misconduct under Chartered Accountants
Act, 1949
 As Sleeping Partner- guilty of professional misconduct under Chartered Accountants Act,
1949 as he did not obtain prior approval of the Council.

© The Institute of Chartered Accountants of India


© The Institute of Chartered Accountants of India
© The Institute of Chartered Accountants of India
© The Institute of Chartered Accountants of India
© The Institute of Chartered Accountants of India
© The Institute of Chartered Accountants of India
© The Institute of Chartered Accountants of India
© The Institute of Chartered Accountants of India
© The Institute of Chartered Accountants of India
Test Series: March, 2021
MOCK TEST PAPER -1
FINAL (NEW) COURSE: GROUP – I
PAPER – 3: ADVANCED AUDITING AND PROFESSIONAL ETHICS
All MCQs are compulsory
Question No. 1 is compulsory.
Attempt any four questions from the Rest.
Time Allowed – 3 Hours Maximum Marks – 100
DIVISION A – MCQs (30 Marks)
Questions no. (1-10) carry 1 Mark each and Questions no. 11-20 carry 2 Marks each.
1. A bank has some non-interest-bearing staff advances. In the Balance Sheet these should be presented
under:
(a) ‘Term loans’ under ‘Advances’.
(b) ‘Cash Credits, Overdrafts and Loans Repayable on Demand’ under ‘Advances’.
(c) ‘Advances in India – Others’ under ‘Advances’ Schedule.
(d) ‘Others’ under ‘Other assets.
2. CA Dharma has established another branch in the same city. Branch was inaugurated on 3rd October
2020 and on 4th October 2020, friends of CA Dharma gave an article on the front page of local
newspaper congratulating CA Dharma on opening of another branch which also includes half page
photograph of CA Dharma with his consent. In your opinion was the news in newspaper a misconduct
on the part of CA Dharma and what actions can be taken against him?
(a) Yes, it is a misconduct under clause 8 of Part I of Second Schedule and he can be reprimanded,
his name can be removed from the register of members for 3 years and fine upto Rs. 5,00,000.
(b) Yes, it is a misconduct under under clause 5 Part I of First Schedule and he can be reprimanded,
his name can be removed from the register of members for 3 months and fine upto Rs. 1,00,000.
(c) Yes, it is a misconduct under clause 7 of Part I of First Schedule and he can be reprimanded, his
name can be removed from the register of members for 3 months and fine upto Rs. 1,00,000.
(d) Yes, it is a misconduct under clause 8 of Part I of Second Schedule and he can be reprimanded,
his name can be removed from the register of members permanently and fine upto Rs. 5 ,00,000.
3. CA Ram identified that there was a misstatement last year and the same is still not corrected. Although
unmodified audit report was issued last year by CA Ram. Guide CA Ram on the audit opinion considering
the fact that the last year’s misstatement has been identified in the current year and unmodified opinion
was issued in the last year?
(a) In accordance with SA 710, CA Ram should give unmodified opinion, but include Other matters
paragraph in the audit report as last year’s profit is being reflected in reserve and surplus.
(b) In accordance with SA 710, CA Ram should seek legal opinion.
(c) In accordance with SA 710, CA Ram should qualify current period audit report with respect to
corresponding figures only.
(d) In accordance with SA 710, CA Ram should give unmodified opinion, but last period’s modified
opinion should be highlighted in Emphasis of matter paragraph.

© The Institute of Chartered Accountants of India


4. For the year ending 31st March 2021, SabkaVikas & Sons has made a claim for refund of custom duty
for Rs. 2 crore but such refund was as admitted as due by authority in April 2021. SabkaVikas & Sons
neither credited the claim in Profit and Loss account nor reported the same in clause 16 of Form 3CD.
Can you please guide the auditor of SabkaVikas & Sons for reporting of refund of custom duty in
accordance with clause 16 of Form 3CD?
(a) Refund of custom duty to the extent of Rs. 2 crore should be reported in clause 16 as the same is
admitted by the custom authorities.
(b) Refund of custom duty to the extent of Rs. 2 crore need not be reported in clause 16 as it is admitted
by custom authorities in the next financial year.
(c) No disclosure is required as refund of custom duties is not covered under clause 16.
(d) Auditor should take a written representation from the management stating that refund of custom
duty of Rs. 2 crore will be credited to profit and loss account for the financial year ending 31st
March 2022 and thus, no reporting is required.
5. B Limited controls entity C Limited (75%) and entity A Limited (an investment company). Entity B Limited
reduced the control of entity C Limited from 75% to 60%. With regard to that certain adjustments were
made to account for the change in the shareholding of entity C Limited which is consolidated. These
adjustments are known as:
(a) Memorandum adjustments.
(b) Current period consolidation adjustments.
(c) Permanent consolidation adjustments.
(d) Temporary period consolidation adjustments.
6. CA Kamal is the statutory auditor of Autocover Ltd. for the FY 2020-21. The company is engaged in the
business of manufacture of car accessories. CA Kamal noticed that the inventories of the company
amounting to Rs. 46 crores (equal to 25% of the total assets of the company) at the end of the year do
not exist. Also, sales amounting to Rs. 33 crores (equal to 10% of the total sales during the year) have
not actually occurred. CA Kamal noticed both the material discrepancies just before the finalisation of
the audit report for the year ending 31.03.2021. CA. Kamal considers that the above misstatement would
distort the true and fair view to a greater extent.
What is correct course of action that CA Kamal should consider in suc h a situation?
(a) CA Kamal should consider withdrawing from the audit engagement or issuing a disclaimer of
opinion for the FY 2020-21.
(b) CA Kamal should consider issuing an adverse opinion and mentioning both the material
discrepancies in the basis for adverse opinion paragraph of the auditor’s report.
(c) CA Kamal should ask the management to explain both the discrepancies in the notes to accounts
and he himself should highlight the matter in the Key Audit matter paragraph of the auditor’s report.
(d) CA Kamal should give a qualified opinion along with the specific mention of the matters in the
Emphasis of matter paragraph in the auditor’s report along with appropriate disclosure in the notes
to accounts to be made by the management of Autocover Ltd.
7. Preparing the financial statements in accordance with the applicable financial reporting framework is the
responsibility of the management of ABC Ltd. Which of the following is correct in regard to the disclosure
of such management responsibility?
(a) This is implied responsibility of management and is presumed in an audit of financial statements
and therefore need not be specifically mentioned anywhere.
(b) The management may undertake to accept such responsibility through an engagement letter itself.

© The Institute of Chartered Accountants of India


(c) The auditor’s report should describe the management responsibility in a section with heading
“responsibility of management for financial statements”.
(d) The auditor’s report should refer to the responsibility of auditors and not that of the management
as the same is obvious.
8. The firm from which you are pursuing your articleship training is the internal auditor of ABC Ltd. While
conducting the audit of the medical expense reimbursements of the company employees, you come
across some bills which are clearly not medical in nature, and some others which have been overwritten.
During the discussions, the accountant points out that the employee is a functional head who enjoys a
significantly higher medical expense reimbursement limit, and that you should ignore those bills as the
amount is not material. You will:
(a) Accept the explanation and the bills.
(b) Recommend that the claim should be reduced, and clear guidelines should be issued to all
employees on the matter, with a provision for disciplinary action.
(c) Recommend that the employee be asked to submit fresh bills to avail the tax benefit.
(d) Recommend that the employee be taxed on the aggregate amount of the suspect bills.
9. Sudarshan Roh (P) Ltd. is having 5 branches across India. Its branch-wise turnover during the
financial year 2019-20 is:
Branch Turnover (Rs. in Crore)
Bangalore 1.65
Mumbai 2.95
Delhi 3.35
Chennai Nil
Calcutta 3.00
The Company would be subject to audit under section 35(5) of the CGST Act, select the correct option
from the following-
(a) Only Mumbai and Delhi would be subject to audit.
(b) None of the branch would be subject to audit.
(c) All the branches would be subject to audit except Chennai.
(d) All the branches would be subject to audit.
10. The acceptable detection risk needs to be ______ in order to reduce the audit risk to ______ in the area
of inventories management and handling.
(a) low in order to reduce audit risk to an acceptably high level.
(b) high in order to reduce audit risk to an acceptably high level.
(c) low in order to reduce audit risk to an acceptably low level.
(d) high in order to reduce audit risk to an acceptably low level. (10 x 1 = 10 Marks)
Questions (11-20) carry 2 Marks each
MCQ 11. -15.
Integrated Case Scenario 1
CA D was a practicing Chartered Accountant in Kolkata from last 15 years. He was appointed as the statutory
auditor of Giant Motors Ltd, a listed entity, which was involved in the business of manufacturing of motor cars
for FY 2019-20. CA D was appointed as joint auditor along with CA T and CA P. They have divided the
3

© The Institute of Chartered Accountants of India


responsibility for conducting audit in accordance with SA 299. As the company has huge amount of property,
plant and equipment, it was decided that all 3 auditors will verify the records relev ant to property, plant and
equipment. While forming an opinion, CA D was having a different opinion on property, plant and equipment
but CA T and CA P were having same opinion. CA D wants to qualify capitali sation of post-acquisition costs
incurred on machinery whereas CA T and CA P were of the opinion that the treatment done by Giant Motor
is correct. Both of them contended that as they are forming a majority, CA D will have to certify common audit
report which is in accordance with the opinion of CA T and CA P. While assessing the applicability of CARO,
2016, CA D found that issued share capital of Giant Motors Ltd is Rs. 500 crore along with Rs. 30 crore of
calls which are being unpaid as they are receivable from retail investors. In the month of July 201 9, Giant
Motors Ltd. forfeited shares of worth Rs. 10 crore. There were no reserve and surplus as it was transferred
to parent entity. Also, along with equity shares of Rs. 300 crore, there was preference share capital of
Rs. 200 crore. CA T while reporting under clause (vi) of CARO, 2016 did not report anything under clause
(vi) of CARO 2016 as the government has not ordered Giant Motors Ltd. to conduct cost audit for its books
of account. Hence CA T did not report anything under clause (vi). Giant Motors Ltd has a total number of 11
directors. Mr. Talent is the Executive Chairman of the company. Out of 11 directors, 5 were independent
directors.
Mrs. D was not aware that CA D was the statutory auditor of Giant Motors Ltd. She purchased shares of Giant
Motors Ltd worth Rs. 1,50,000 (book value) on 3rd October 2020 but when she came to know about the
statutory auditor of Giant Motors Ltd, she sold her shares on 10th November 2020. One of the shareholders
of Giant Motors Ltd contended that CA D is disqualified and shall vacate his office of statutory auditor.
On the basis of the abovementioned facts, you are required to answer the following MCQs:
Multiple Choice Questions (5 questions of 2 Marks each):
11. Can you please guide whether CA D really needs to go with the opinion formed by CA T and CA P or
not?
(a) CA D will have to go with the opinion formed by majority auditors.
(b) CA D can add a separate audit opinion paragraph in the common audit report and the same should
be highlighted in emphasis of matter paragraph.
(c) CA D can go with the opinion formed by the majority auditors, but CA D had a difference of opinion
should be highlighted in emphasis of matter paragraph.
(d) CA D can altogether issue a separate audit report and reference of other audit report issued by
majority auditors should be made in the emphasis of matter paragraph.
12. What should have been CA D’s opinion on applicability of CARO, 2016 for FY 2019 -20 assuming
forfeited shares are not included in equity share capital?
(a) CARO will be applicable as paid up share capital and reserves are Rs. 480 crore which is more
than Rs. 1 crore.
(b) CARO will be applicable as paid up share capital and reserves are Rs. 480 crore which is more
than Rs. 10 crore.
(c) CARO will be applicable as paid up share capital and reserves are Rs. 280 crore which is more
than Rs. 1 crore.
(d) CARO will be applicable as paid up share capital and reserves are Rs. 280 crore which is more
than Rs. 10 crore.
13. Was the approach followed by CA T for not reporting under clause (vi) of CARO correct?
(a) Yes, as reporting under said clause is required only if the Giant Motors Ltd were ordered by
government to conduct cost audit under section 148(1).

© The Institute of Chartered Accountants of India


(b) Yes, reporting under this clause is only applicable to entities involved in production of electricity.
(c) No, Clause (vi) should be reported irrespective of whether Giant Motors Limited has been ordered
to conduct cost audit by the Central Government or not.
(d) No, should be reported only if there is any discrepancy found while examining the cost records.
14. Was there any non-compliance on the part of Giant Motors Ltd in case of appointment of independent
directors?
(a) No, there was no non- compliance as independent directors were more than 2 directors specified
in the Companies Act, 2013.
(b) Yes, there was a non-compliance as there should have been more than 6 independent directors
specified in Regulation 17 and Regulation 17A.
(c) No, there was no non-compliance as independent directors were 5, which is more than 2/3 of the
total directors in accordance with Regulations 17 and Regulation 17A.
(d) Yes, there was a non-compliance as all the directors should have been independent directors
except the Chairman of the company.
15. Was the contention of shareholder that CA D should vacate the office of statutory auditor correct?
(a) No, as Mrs. D has sold the shares within a grace period of 60 days.
(b) No, as Mrs. D is holding shares of less than book value of Rs. 2,00,000.
(c) Yes, as Mrs. D has purchased shares which are more than book value of Rs.1,00,000.
(d) Yes, as Mrs. D hold share during the financial year and his husband is statutory auditor of Giant
Motors Ltd.
MCQ 16. -20.
Integrated Case Scenario 2
Well & Associates, an audit firm, was selected for the purpose of Quality Review by the Quality Review Board
(QRB) as it was having many of statutory audit assignments of clients engaged into sectors identi fied as
prone to fraud.
There were adverse findings by the Technical Reviewer in the Quality review conducted in the past of
Mr. Ramesh an engagement partner of Well & Associates because of which the QRB selected 5 audit
engagements of the firm for Quality review.
Mr. Jay, a practicing CA for more than 25 years was appointed as the Technical Reviewer to conduct the
Quality Review of the said firm and accordingly, Mr. Jay, after conducting the Quality review with a team of 3
assistants, submitted his preliminary report to Well & Associates with qualifications as under:
Sr. No. Description of Qualifications
1 The AFUR (Audit Firm Under Review) had not obtained a written confirmation of compliance
with its policies and procedures on independence from all firm personnel for the past 2 financial
years.
2 The AFUR had established the policies and procedures for assembling of the final audit file in
accordance with the time limit prescribed in SA 230 but there were delays observed in the
same. (Please Refer Note, as below, for the same)
3 For two of the audit engagements of the AFUR, no engagement documentations were available
for the same and as per the statement of the partner of the AFUR, after retaining them for 4
years and 6 years, respectively, were sent to the Principal Auditors of the said audit
engagements.

© The Institute of Chartered Accountants of India


4 There were also instances of delays observed in communicating the significant deficiencies to
those charged with governance. (Please Refer Note, as below, for the same)
5 The AFUR had revised its performance materiality level in case of one of its statutory audit
assignments with respect to auditing of Financial Leasing transactions and the AFUR had only
documented such revision in the performance materiality level.
Note:

Name of the Type of Date of Date of Date of Date of written


entity under Entity Approval Audit assembly of communication of
audit of AFUR Financial Report Final Audit significant deficiencies in
Statements File internal control by AFUR
Req Ltd. Listed 31.05.2021 25.06.2021 03.09.2021 05.06.2021
TIMCO (P) Ltd. Unlisted 15.06.2021 18.08.2021 05.11.2021 25.08.2021
Gles Pvt. Ltd. Unlisted 16.07.2021 28.07.2021 15.09.2021 18.09.2021
Findey Ltd. Listed 12.05.2021 01.06.2021 01.08.2021 05.05.2021
DM Ltd. Unlisted 25.04.2021 18.05.2021 25.06.2021 04.07.2021

On the basis of the abovementioned facts, you are required to answer the following MCQs:
Multiple Choice Questions (5 questions of 2 Marks each):
16. Well & Associates should have obtained a written confirmation of compliance with its policies and
procedures on independence from all of its firm personnel as per requirements of which Statue /
Standard and in what frequency?
(a) As per the requirements of Council Central Guidelines, 2008, at least annually, Well & Associates
should have obtained a written confirmation from all of its firm personnel.
(b) As per the requirements of Standard on Quality Control 1 at least annually, Well & Associates
should have obtained a written confirmation from all of its firm personnel.
(c) As per the requirements of SA 220 at least annually, Well & Associates should have obtained a
written confirmation from all of its firm personnel.
(d) As per the requirements of Code of Ethics at least half yearly, Well & Associates should have
obtained a written confirmation from all of its firm personnel.
17. In case of which entities under audit of Well & Associates, there was delay in assembly of Final Audit
File?
(a) Req Ltd., TIMCO (P) Ltd., Gles Pvt. Ltd. and Findey Ltd., respectively.
(b) Req Ltd., TIMCO (P) Ltd. and Findey Ltd., respectively.
(c) Req Ltd. and TIMCO (P) Ltd., respectively.
(d) Req Ltd., TIMCO (P) Ltd., Gles Pvt. Ltd., Findey Ltd. and DM Ltd., respectiv ely.
18. In case of which entities under audit of Well & Associates, there was delay in written communication of
significant deficiencies in internal control?
(a) TIMCO (P) Ltd., Gles Pvt. Ltd. and DM Ltd., respectively.
(b) Req Ltd., TIMCO (P) Ltd., Gles Pvt. Ltd. and DM Ltd., respectively.
(c) DM Ltd.
(d) Req Ltd., Gles Pvt. Ltd. and DM Ltd., respectively.

© The Institute of Chartered Accountants of India


19. For at least how many more years, Well & Associates should have retained the engagement
documentation in respect of the two audit engagements as referred above?
(a) 3 years and 1 year, respectively.
(b) 4 years and 2 years, respectively.
(c) 1 year and for other audit engagement documentation was retained for requisite period.
(d) 6 years and 4 years, respectively.
20. How many audit engagements of Well & Associates the QRB might have selected if there were no
adverse findings by the Technical Reviewer in the Quality review conducted in the past of Mr. Ramesh,
partner of Well & Associates?
(a) QRB might have selected up to 3 audit engagements of Well & Associates for review and not more
than 2 audit engagements of Mr. Ramesh.
(b) QRB might have selected up to 5 audit engagements of Well & Associates for review and not more
than 1 audit engagement of Mr. Ramesh
(c) QRB might have selected up to 5 audit engagements of Well & Associates for review and not more
than 2 audit engagements of Mr. Ramesh
(d) QRB might have selected up to 3 audit engagements of Well & Associates for review and not more
than 1 audit engagement of Mr. Ramesh. (10 x 2 = 20 Marks)
Division B- Descriptive Questions-70 Marks
Question No. 1 is compulsory.
Attempt any four questions from the Rest.
1. (a) J.A.C.K. & Co., a Chartered Accountant firm was appointed as the statutory auditor of Falcon Ltd.
after ensuring the compliance with relevant provisions of the Companies Act, 2013. Mr. Jay was
the engagement partner for the aforesaid audit and prior to commencement of the audit, Mr. Jay
had called for a meeting of the engagement team in order to direct them and assign them their
responsibilities. At the end of meeting, Mr. Jay assigned review responsibilities to two of the
engagement team members who were the most experienced amongst all, for reviewing the work
performed by the less experienced team members. While reviewing the work performed by the less
experienced members of the engagement team, what shall be the considerations of the reviewers ?
(5 Marks)
(b) Rajul Ltd had a net worth of INR 2500 crores because of which Ind AS became applicable to them.
The company had various derivative contracts – options, forward contracts, interest rate swaps
etc. which were required to be fair valued for which company got the fair valuation done through
an external third party. The statutory auditors of the company involved an auditor’s expert to audit
valuation of derivatives. Auditor and auditor’s expert were new to each other i.e., they were working
for the first time together but developed a good bonding during the course of the audit. The auditor
did not enter into any formal agreement with the auditor’s expert. Please advise. (5 Marks)
(c) Mokshda & Co is the statutory auditor of Get My Trip Ltd. The company is in the business of tours
and travels. Annual turnover of the company is INR 2765 crore and profits are INR 285 crore.
During the planning meeting of the management and the auditors, it was discussed that the
management needs to provide written representation letter to the auditors for the preparation of
the financial statements and for the completeness of the information provided to the auditor. At the
time of closure of the audit, there has been some confusion about the requirements of the written
representation letter. Management argued that representation need not be written, it can also be
verbal which has been provided to the audit team during the course of their audit. Auditors have
completed their documentation and hence in a way, representation based on verbal discussions
7

© The Institute of Chartered Accountants of India


with the auditors has also got documented. Auditors explained that this is mandatory to obtain
written representation in accordance with the requirements of SA 580. However, still some
confusion remains regarding the date and period covered by the written representation. You are
required to advise about the date of and period covered by written representation in view of
SA 580. (4 Marks)
2. (a) RAO & Co., a Chartered Accountant Firm, is appointed as the principal auditor of a listed company,
Triumph Ltd.
Figures of income and net-worth of five out of seven components of Triumph Ltd., which are its
unlisted subsidiaries, is tabulated below for the immediate preceding financial year along with the
consolidated amount:- (Rs. in crores)
Particulars Consolidated Component Component Component Component Component
‘A’ ‘B’ ‘C’ ‘D’ ‘E’
Income 300 35 10 70 65 20
Net Worth 800 40 20 140 180 50

The remaining two components i.e., Component ‘F’ & Component ‘G’ of Triumph Ltd. were
unaudited. According to Mr. RAO, the engagement partner, Component ‘F’ is material to the
consolidated financial statements whereas Component ‘G’ is not material to consolidated financial
statements and this fact has also been discussed in writing with those charged with governance of
Triumph Ltd. and it will also form part of report as a ‘Key audit matter’ in accordance with SA 701.
(i) Which of the components of Triumph Ltd. can be termed as “material subsidiary” and in the board
of which of the unlisted subsidiaries at least one independent director of Triumph Ltd. needs to be
appointed or would be appointed? (5 Marks)
(ii) What shall be the audit consideration in relation to reporting in case of unaudited components of
Triumph Ltd. by RAO & Co. and how RAO & Co. as a principal auditor shall report in case of
Component ‘F’ & Component ‘G’, respectively? (4 Marks)
(b) M/s SS limited is a partly owned subsidiary of M/s HH limited. For the upcoming financial year, M/s
DD & Co., Chartered Accountants, were appointed as the statutory auditors of SS limited. The CEO
of the holding company was impressed with the knowledge and experience of Mr. D, one of the
partners of the firm and hence, he offered Mr. D to take up the position of Director (not MD/ whole-
time director) of HH limited. At the same time, Mr. D’s friend approaches him with an assignment
to act as a Recovery Consultant for a bank. Mr. D is now confused whether to accept or reject the
offers. He approaches you and seeks your advice on the same. Advise what Mr. D about what he
can do with the offers with reference to the Chartered Accountants Act, 1949 and Schedules
thereto. (5 Marks)
3. (a) R.O.K. & Co. and TNK & Co. were appointed as the joint statutory auditors at the AGM of Auspic
General Insurance Co. Ltd. Apart from the aforesaid audit, R.O.K. & Co. is also being appointed
as a joint statutory auditor of one another General Insurance Company and TNK & Co. is appointed
as a joint statutory auditor of Life Insurance Company. How many further audits can be accepted
by R.O.K. & Co. and TNK & Co., respectively, of either general or life insurance companies?
(4 Marks)
(b) Mr. Raj, the engagement partner of R.O.K. & Co., in connection with statutory audit of Waria Ltd.,
had assigned the responsibility of enquiring into propriety matters of the Company as required by
section 143(1) of the Companies Act, 2013, to Mr. Samay, an engagement team member. Mr.
Samay while making such enquiries, was having following queries, as tabulated below, which he
ought to get resolved from Mr. Raj, as follows:-

© The Institute of Chartered Accountants of India


Sr. No. Query of Mr. Samay
1 What documents to be seen in case of loan given by the company in lieu of
hypothecation of goods from lender as a security for the purpose of reporting as per
clause (a) of section 143(1) of the Companies Act, 2013?
2 What shall be the cost of Debentures and Bonus Shares sold by the company for
which the cost is not ascertainable for the purpose of reporting as per clause (c) of
section 143(1) of the Companies Act, 2013?
3 Whether the shares allotted by Waria Ltd. against a loan taken by it from a NBFC can
be considered to be allotted for cash for the purpose of reporting as per clause (f) of
section 143(1) of the Companies Act, 2013?
Assuming that you are Mr. Raj the engagement partner, please provide answe r to the queries of
Mr. Samay? (6 Marks)
(c) A letter is sent by Mr. Raja, a Chartered Accountant in practice, to the Ministry of Finance inquiring
whether a panel of auditors is being maintained by the Ministry and if so to include his name in the
panel. He also enclosed his CV. Comment on the above with reference to the Chartered
Accountants Act, 1949 and Schedules thereto. (4 Marks)
4. (a) In course of audit of Decent Samaritan Bank as at 31st March, 20 you observed the following:
(i) In a particular account there was no recovery in the past 18 months. The bank has not applied
the NPA norms as well as income recognition norms to this particular account. When queried
the bank management replied that this account was guaranteed by the Central Government
and hence these norms were not applicable. The bank has not invoked the guarantee. Please
respond. Would your answer be different if the advance is guaranteed by a State
Government?
(ii) The bank’s advance portfolio comprised of significant loans against Life Insurance Policies.
Write suitable audit program to verify these advances. (6 Marks)
(b) CA. Sudarshan, appointed as a Peer Reviewer for M/s. Preet Associates, has asked for all the
management consultancy engagements and engagements solely to assist the client in preparing,
compiling or collating information other than financial statements carried out by M/s. Preet
Associates for peer review during the period considered for peer review purposes by the board.
Peer Reviewer CA. Sudarshan has also sent out a mail to Peer Review Board regarding his
selection. Mr. Preet, the managing partner of the firm seeks your advise on this matter.
(4 Marks)
(c) Mr. Abhinandan engaged in business as a sole proprietor presented the following information to
you for the FY 2020-21. Turnover expected to be made during the year Rs. 524 lacs. Goods
returned in respect of sales made during FY 2019-20 is Rs. 20 lacs not included in the above. Cash
discount allowed to his customers Rs. 1 lac for prompt payment. Special rebate allowed to customer
in the nature of trade discount Rs. 5 lacs. Further, the aggregate of all amounts received including
amount received for sales, turnover or gross receipts during the previous year, in cash, does not
exceed five per cent of the said amount and aggregate of all payments made including amount
incurred for expenditure, in cash, during the previous year does not exceed five per cent of the
said payment. Kindly advise him whether he has to get his accounts audited u/s 44AB of the Income
Tax Act, 1961. (4 Marks)
5. (a) Mr. Z, a newly qualified chartered accountant started his practice in February 2018 by setting up
an office in the hill station Kodaikanal. Initially, since he was getting very less assignments, he
decided to set up a temporary office in the nearby city Marudai, situated at about 100 kms from the
main office. As planned, he took an office space on rent for the months of April, May & June. During
these months, his regular office was not closed and Mr. Z was in-charge for both the offices. Mrs.
9

© The Institute of Chartered Accountants of India


A, another newly qualified chartered accountant who is also in practice in Marudai came to know
about the new office of Mr. Z. Thinking that he could be a potential competitor, she informed the
institute stating that Mr. Z had violated the provisions of the Chartered Accountant Act. As a
member of the Board of Discipline of ICAI, you are requested to analyse this complaint.
(5 Marks)
(b) The Marketing Department of ISHITA Ltd. has been consistently showing a lower performance
whereas the cost of the department is increasing in spurts over the years. The management
believes that since the marketing department is under a regular radar of the CFO, an audit might
result in the employee hostility. Also, an operational audit of Marketing Department was done two
years back however, the recommendations of the previous audit were not followed by the
concerned employees. Please advise the management if another audit is the solution and whether
only one-time operational audit is enough? Further, advise on the ways to deal with the employee
hostility. (5 Marks)
(c) VM Ltd., a company wholly owned by Central Government was disinvested during the previous
year, resulting in 45% of the shares being held by public. The shares were also listed on the BSE.
Since the shares were listed, all the listing requirements were applicable, including publication of
quarterly results, submission of information to the BSE etc.
Gautam, the Finance Manager of the Company is of the opinion that now the company is subject
to stringent control by BSE and the markets, therefore the auditing requirements of a limited
company in private sector under the Companies Act 2013 would be applicable to the company and
the C&AG will not have any role to play. Comment. (4 Marks)
6. (a) LMN Ltd. entered into a deal with SP Ltd. for buying its business of manufacturing wooden
products/ goods. LMN Ltd. has appointed your firm for conducting due diligence review and they
want to know the cash generating abilities of SP Ltd. What points will you check in order to ensure
that the manufacturing unit of SP Ltd. will be able to meet the cash requirements internally?
(5 Marks)
(b) Entertainment Paradise, a movie theatre complex, is the foremost theatre located in Ch ennai.
Along with the sale of tickets over the counter and online booking, the major proportion of income
is from the cafe, shops, pubs etc. located in the complex. Its other income includes advertisements
exhibited within/outside the premises such as hoardings, banners, slides, short films etc. The
facility for parking of vehicles is also provided in the basement of the premises.
Entertainment Paradise appointed your firm as the auditor of the entity. Being the head of the audit
team, you are, therefore, required to draw an audit programme initially in respect of its revenue
and expenditure considering the above mentioned facts along with other relevant points relating to
a complex. (5 Marks)
(c) As auditor of ZED Ltd., you would like to limit your examination of account balance tests. What are
the control objectives you would like the accounting control system to achieve to suit your purpose?
(4 Marks)
OR
A real-time environment is a type of automated environment in which business operations and
transactions are initiated, processed and recorded immediately (without any delay) as they happen.
It has several critical IT components that enable anytime, anywhere transactions to take place. You
are required to name the components and its example of real-time environment. (4 Marks)

10

© The Institute of Chartered Accountants of India


Test Series: March, 2021
MOCK TEST PAPER - 1
FINAL (NEW) COURSE: GROUP – I
PAPER – 3: ADVANCED AUDITING AND PROFESSIONAL ETHICS
SUGGESTED ANSWERS/HINTS
DIVISION A - MCQs (30 Marks)
Questions no. (1-10) carry 1 Mark each and Questions no. 11-20 carry 2 Marks each.
1. (d)
2. (c)
3. (c)
4. (b)
5. (c)
6. (b)
7. (c)
8. (b)
9. (d)
10. (c)
Questions (11-20) carry 2 Marks each
11. (d)
12. (a)
13. (c)
14. (b)
15. (a)
16. (b)
17. (b)
18. (d)
19. (a)
20. (d)
DIVISION B - DESCRIPTIVE QUESTIONS (70 Marks)
1. (a) As per SQC 1, “Quality Control for Firms that Perform Audits and Reviews of Historical Financial
Information, and Other Assurance and Related Services Engagements”, review responsibilities
are determined on the basis that more experienced team members, including the engagement
partner, review work performed by less experienced team members.
In the given situation, Mr. Jay, engagement partner assigned review responsibilities to two of the
engagement team members who were the most experienced team members.
While reviewing the work performed by less experienced members of the engagement team, both
the more experienced Reviewers should consider whether:

© The Institute of Chartered Accountants of India


(i) The work has been performed in accordance with professional standards and regulatory and
legal requirements.
(ii) Significant matters have been raised for further consideration.
(iii) Appropriate consultations have taken place and the resulting conclusions have been
documented and implemented.
(iv) There is a need to revise the nature, timing and extent of work performed.
(v) The work performed supports the conclusions reached and is appropriately documented.
(vi) The evidence obtained is sufficient and appropriate to support the report; and
(vii) The objectives of the engagement procedures have been achieved.
(b) As per SA 620, Using the work of an Auditor’s Expert, the nature, scope and objectives of the
auditor’s expert’s work may vary considerably with the circumstances, as may the respective
roles and responsibilities of the auditor and the auditor’s expert, and the nature, timing and extent
of communication between the auditor and the auditor’s expert. It is therefore required that these
matters are agreed between the auditor and the auditor’s expert.
In certain situations, the need for a detailed agreement in writing is required like -
• The auditor’s expert will have access to sensitive or confidential entity information.
• The matter to which the auditor’s expert’s work relates is highly complex.
• The auditor has not previously used work performed by that expert.
• The greater the extent of the auditor’s expert’s work, and its significance in the context of
the audit.
In the given case, considering the complexity involved in the valuation and volume of d erivatives
and also due to the fact that the auditor and auditor’s expert were new to each other, auditor
should have signed a formal agreement/ engagement letter with the auditor’s expert in respect of
the work assigned to him.
(c) As per SA 580, “Written Representations”, as written representations are necessary audit
evidence, the auditor’s opinion cannot be expressed, and the auditor’s report cannot be dated,
before the date of the written representations. Furthermore, because the auditor is concerned
with events occurring up to the date of the auditor’s report that may require adjustment to or
disclosure in the financial statements, the written representations are dated as near as
practicable to, but not after, the date of the auditor’s report on the financial statements.
In some circumstances it may be appropriate for the auditor to obtain a written representation
about a specific assertion in the financial statements during the course of the audit. Where this is
the case, it may be necessary to request an updated written representation.
The written representations are for all periods referred to in the auditor’s report because
management needs to reaffirm that the written representations it previously made with respect to
the prior periods remain appropriate. The auditor and management may agree to a form of written
representation that updates written representations relating to the prior periods by addressing
whether there are any changes to such written representations and, if so, what they are.
Situations may arise where current management were not present during all periods referred to in
the auditor’s report. Such persons may assert that they are not in a position to provide some or
all of the written representations because they were not in place during the period . This fact,
however, does not diminish such persons’ responsibilities for the financial statements as a whole.
Accordingly, the requirement for the auditor to request from them written representations that
cover the whole of the relevant period(s) still applies.

© The Institute of Chartered Accountants of India


2. (a) (i) As per Regulation 16(c) of the SEBI (LODR) Regulations, 2015, “material subsidiary” shall
mean a subsidiary, whose income or net worth exceeds ten percent of the consolidated
income or net worth respectively, of the listed entity and its subsidiaries in the immediately
preceding accounting year. [Explanation- The listed entity shall formulate a policy for
determining ‘material’ subsidiary.]
Regulation 24(1) of the SEBI (LODR) Regulations, 2015, provides that at least one
independent director on the board of directors of the listed entity shall be a director on the
board of directors of an unlisted material subsidiary, whether incorporated in India or not.
[Explanation- For the purposes of Regulation 24(1), notwithstanding anything to the contrary
contained in regulation 16, the term “material subsidiary” shall mean a subsidiary, whose
income or net worth exceeds twenty percent of the consolidated income or net worth
respectively, of the listed entity and its subsidiaries in the immediately preceding accounting
year]
On the basis of above provisions, following information is tabulated as below:
Particulars Share in Consolidated Share in Consolidated
Income Net Worth
Component ‘A’ 11.67% 5%
Component ‘B’ 3.33% 2.5%
Component ‘C’ 23.33% 17.5%
Component ‘D’ 21.67% 22.5%
Component ‘E’ 6.67% 6.25%
It can be observed that Component ‘A’, Component ‘C’ and Component ‘D’, respectively,
can be termed as “material subsidiary” as their shares in either consolidated Income or net
worth exceeds 10%.
Further, at least one independent director from the board of directors of Triumph L td. shall
be appointed or would have been appointed on the board of Component ‘C’ and Component
‘D’, respectively, as their shares in either consolidated income or net worth exceeds 20%.
(ii) Generally, the financial statements of all components included in consolidated financial
statements should be audited or subjected to audit procedures in the context of a multi -
location group audit. Such audits and audit procedures can be performed by the auditor
reporting on the consolidated financial statements or by the components’ auditor.
Where the financial statements of one or more components continue to remain unaudited,
the auditor reporting on the consolidated financial statements should consider unaudited
components in evaluating a possible modification to his report on the consolidated financial
statements. The evaluation is necessary because the auditor (or other auditors, as the case
may be) has not been able to obtain sufficient appropriate audit evidence in relation to such
consolidated amounts/balances. In such cases, the auditor should evaluate both qualitative
and quantitative factors on the possible effect of such amounts remaining unaudited when
reporting on the consolidated financial statements using the guidance provided in SA 705,
“Modifications to the Opinion in the Independent Auditor’s Report”.
In the given situation, two out of seven components of Triumph Ltd. have remained
unaudited where Component ‘F’ is material and Component ‘G’ is not material to the
consolidated financial statements. Since Component ‘F’ is material, therefore, it may be
assumed that reporting of Key Audit Matter in accordance with SA 701 is being done for
Component ‘F’ and not for Component ‘G’.
Thus, in case of Component ‘F’, the Principal Auditor needs to consider its impact on the
3

© The Institute of Chartered Accountants of India


auditor’s opinion on the consolidated financial statements of the group, in terms of the
principles laid down in SA 705, Modifications to the Opinion in the Independent Auditor’s
Report. Whereas in case of Component ‘G’, the principal auditor should make appropriate
reporting under the “Other Matters” paragraph, pursuant to SA 706, Emphasis of Matter
Paragraphs and Other Matter Paragraphs, in the Independent Auditor’s Report.
(b) As per Clause (11) of Part I of First Schedule of Chartered Accountants Act, 1949, a Chartered
Accountant in practice is deemed to be guilty of professional misconduct if he engages in any
business or occupation other than the profession of Chartered Accountant unless permitted by
the Council so to engage.
Provided nothing contained herein shall disentitle a chartered accountant from being a director of
a company (not being MD or whole-time director) unless he or his partners is interested in such
company as auditor.
The Ethical Standards Board (ESB) noted that Public conscience is expected to be ahead of law.
Members, therefore, are expected to interpret the requirement as regards independence much
more strictly than what the law requires and should not place themselves in positions which
would either compromise or jeopardise their independence. In the view of the above, the Board,
via a clarification, decided that the auditor of a Subsidiary company cannot be a Director of its
Holding company, as it will affect the independence of the auditor.
However, the Council has granted general permission to the members to engage in certain
specific occupation. In respect of all other occupations specific permission of the Institute is
necessary. ‘acting as Recovery Consultant in the banking sector’ is covered under general
permission.
In the given situation, M/s SS limited is a partly owned subsidiary of M/s HH limited. For the
upcoming financial year, M/s DD & Co., Chartered Accountants, were appointe d as the statutory
auditors of SS limited. The CEO of the holding company was impressed with the knowledge and
experience of Mr. D, one of the partners of the firm and hence, he offered Mr. D to take up the
position of Director (not MD/ whole-time director) of HH limited. Further, Mr. D’s friend
approached him for an assignment for acting as a Recovery Consultant for a bank.
Therefore, in view of above in the given case, Mr. D should not accept the offer to be appointed
as director of HH Limited.However, he can accept the assignment offered by his friend and can
act as a recovery consultant for a bank.
3. (a) The appointment of statutory auditors in the General Insurance Corporation of India, and its
subsidiaries and the divisions as well as other public sector Insurance Companies is made by the
Comptroller and Auditor General of India, as in the case of other public sector undertakings .
However, in the case of others, auditor is appointed at the AGM after ensuring that the auditor
satisfies the compliance requirements with the relevant sections of the IRDAI Guidelines on
Corporate Governance. These guidelines pose certain restrictions on the number of insurance
companies a statutory auditor can audit. Currently, an auditor can conduct audit only for three
insurance companies and not more than 2 life or 2 general. The Guidelines also mandate a
mandatory joint audit for all insurance companies.
In the given case, R.O.K. & Co. is joint statutory auditor of Auspic General Insurance Co. Ltd.
And of one another General Insurance Company. Accordingly, it can now, further, accept only
one audit and that too of a Life Insurance Company only.
Further, TNK & Co. is joint statutory auditor of Auspic General Insurance Co. Ltd. as well as of
one Life Insurance Company. Accordingly, it can now, further, accept only one audit of either a
Life Insurance Company or a General Insurance Company.

© The Institute of Chartered Accountants of India


(b)
Sr. Query of Mr. Samay Response to Query
No.
1 What documents to be seen in case of Mr. Samay should see deed of Hypothecation or
loan given by the company in lieu of other document creating the charge, together
hypothecation of goods from lender as a with a statement of stocks held at the balance
security for the purpose of reporting as sheet date in order.
per clause (a) of section 143(1) of the
Companies Act, 2013?
2 What shall be the cost of Debentures For Debentures sold: Where the cost of
and Bonus Shares sold by the company debentures sold is not ascertainable, the book
for which the cost is not ascertainable for value thereof at the date of sale may be treated
the purpose of reporting as per clause as the cost for the purposes of this clause.
(c) of section 143(1) of the Companies For Bonus Shares sold: When bonus shares are
Act, 2013? received, the number of shares in the portfolio
would be increased by the bonus shares while
the cost of the total portfolio would remain the
same as before. The result would be that the
average cost per unit of the total holding would
come down proportionately. The usual
accounting practice for apportioning the cost of a
part of the total holding on the sale thereof is to
take it at its average cost.
3 Whether the shares allotted by Waria The law on the subject has hitherto been that,
Ltd. against a loan taken by it from a where the consideration for the issue of shares
NBFC can be considered to be allotted is an adjustment against a bona fide debt
for cash for the purpose of reporting as payable in money on demand by the company,
per clause (f) of section 143(1) of the the shares are deemed to have been
Companies Act, 2013? subscribed in cash (vide the decision in
Spargo’s Case – 1873, 8, Ch. A. 407). According
to the legal opinion obtained by the ICAI, the
expression “shares allotted for cash” may also
include shares allotted against a debt.
Therefore, in cases which are covered by the
decision in Spargo’s case, no comment is
required by the auditor, even though the
company may have in the Return of Allotment
under Section 75, shown such shares as allotted
against adjustment of a debt.
Thus, the shares allotted by Waria Ltd. against a
loan taken by it from a NBFC can be considered
to be allotted for cash.

(c) Making Roving Inquiries: Clause (6) of Part I of the First Schedule to the Chartered
Accountants Act, 1949 states that a Chartered Accountant in practice shall be deemed to be

© The Institute of Chartered Accountants of India


guilty of misconduct if he solicits clients or professional work either directly or indirectly by a
circular, advertisement, personal communication or interview or by any other means. Such a
restraint has been put so that the members maintain their independence of judgement and may
be able to command respect from their prospective clients.
In case of making an application for the empanelment for the allotment of audit and other
professional work, the Council has opined that, “where the existence of such a panel is within the
knowledge of the member, he is free to write to the concerned organization with a request to
place his name on the panel. However, it would not be proper for the member to make roving
inquiries by applying to any such organization for having his name included in any such panel.”
Accordingly, Mr. Raja is guilty of misconduct in terms of the above provision as he has solicited
professional work from the Finance Ministry, by inquiring about the maintenance of the panel .
4. (a) (i) Government Guaranteed Advance: If a government guaranteed advance becomes NPA,
then for the purpose of income recognition, interest on such advance should not to be taken
to income unless interest is realized. However, for purpose of asset classification, credit
facility backed by Central Government Guarantee, though overdue, can be treated as NPA
only when the Central Government repudiates its guarantee, when invoked.
Since the bank has not invoked the guarantee, the question of repudiation does not arise.
Hence the bank is correct to the extent of not applying the NPA norms for provisioning
purpose. But this exemption is not available in respect of income recognition norms. Hence
the income to the extent not recovered should be reversed.
The situation would be different if the advance is guaranteed by State Government because
this exception is not applicable for State Government Guaranteed advances, where advance
is to be considered NPA if it remains overdue for more than 90 days.
In case the bank has not invoked the Central Government Guarantee though the amount is
overdue for long, the reasoning for the same should be taken and duly reported in LFAR.
(ii) The Audit Programme to Verify Advances against Life Insurance Policies is a s under-
(i) The auditor should inspect the policies and see whether they are assigned to the bank
and whether such assignment has been registered with the insurer.
(ii) The auditor should also examine whether premium has been paid on the policies and
whether they are in force.
(iii) Certificate regarding surrender value obtained from the insurer should be examined.
(iv) The auditor should particularly see that if such surrender value is subject to payment of
certain premium, the amount of such premium has been deducted from the surrender
value.
(b) Selection of Assurance Service Engagements for Review: The Statement on Peer Review
defines the scope of peer review which revolves around compliance with technical, ethical and
professional standards; quality of reporting; office systems and procedures with regard to
compliance of assurance engagements; and, training programmes for staff including articled and
audit assistants involved in assurance engagements. The entire peer review process is
directed at the assurance services.
Assurance Services means assurance engagements services as specified in the “Framework
for Assurance Engagements” issued by the Institute of Chartered Accountants of India and as
may be amended from time to time. Assurance engagements does not include management
consultancy engagements or engagements solely to assist the client in preparing, compiling or
collating information other than financial statements.

© The Institute of Chartered Accountants of India


In the given situation, CA. Sudarshan is appointed as a peer reviewer for M /s Preet Associates,
has asked for all management consultancy engagements and engagements solely to assist the
client in preparing, compiling or collating information other than financial statements carried out
by M/s Preet Associates for her peer review. In view of above, Peer Review of management
consultancy engagements and engagements solely to assist the client in preparing, compiling or
collating information other than financial statements at the time of execution step by CA.
Sudarshan is not correct as management consultancy engagements and engagements solely
to assist the client in preparing, compiling or collating information other than financial statements
are not covered in the scope of Assurance engagement and Peer Review is directed at
assurance engagement only.
(c) Turnover limit for the purpose of Tax Audit: The following points merit consideration as stated
in the Guidance note on Tax Audit issued by the Institute of Chartered Accountants of India -
(i) Price of goods returned should be deducted from the figure of turnover even if the return are
from the sales made in the earlier years.
(ii) Cash discount otherwise than that allowed in a cash memo/sales invoice is in the nature of
a financing charge and is not related to turnover. The same shoul d not be deducted from the
figure of turnover.
(iii) Special rebate allowed to a customer can be deducted from the sales if it is in the nature of
trade discount.
Applying the above stated points to the given problem,
1. Total Turnover 524 Lac
2. Less – (i) Goods Returned 20 Lac
(ii) Special rebate allowed to customer in the nature of trade
discount would be deducted 5 Lac
Balance 499 Lac

Since the aggregate of all amounts received including amount received for sales, turnover or
gross receipts during the previous year, in cash, does not exceed five per cent of the said
amount and aggregate of all payments made including amount incurred for expenditure, in cash,
during the previous year does not exceed five per cent of the said pay ment, limit for tax audit is
five crore rupees. In the given situation, Abhinandan would not be required to get his accounts
audited under section 44AB of the Income Tax Act, 1961 as Rs. 499 lac is below prescribed tax
audit limit i.e. five crore rupees.
5. (a) As per section 27 of Chartered Accountants Act 1949, if a Chartered Accountant in practice or a
Firm of Chartered Accountants has more than one office in India, each one of such offices should
be in the separate charge of a member of the Institute. Failure on the part of a member or a firm
to have a member in charge of its branch and a separate member in case of each of the
branches, where there is more than one, would constitute professional misconduct. This
condition applies to any additional office situated at a place beyond 50 kms from the municipal
limits in which any office is situated.
However, exemption has been given to members in practicing in hill areas subject to certain
conditions such as:
− Such member/ firm be allowed to open temporary offices in a city in the plains for a limited
period not exceeding 3 months in a year.
− The regular office need not be closed during this period and all correspondence can

© The Institute of Chartered Accountants of India


continue to be made at the regular office.
− The name board of the firm in temporary office should not be displayed at times other than
the period such office is permitted to function.
− The temporary office should not be mentioned in letter head, visiting card, any other
documents as a place of business of the member/ firm.
− Before commencement of every winter, it shall be obligatory on the member/firm to inform
the Institute that he/it is opening the temporary office from a particular date and after the
office is closed at the expiry of the period of permission, an intimation to that effect shou ld
also be sent to the office of the Institute by registered post.
In the given case, Mr. Z has set up his regular office in the hill area of Kodaikanal , he decided to
set up a temporary office in the nearby city Marudai, situated at about 100 kms from the main
office. As planned, he took an office space on rent for the months of April, May & June. During
these months, his regular office was not closed. Further he was in-charge for both the offices. In
view of abovementioned criteria’s, he is eligible to avail the benefits of the above exemptions.
Also, it is given that the temporary office was open in Madurai for only 3 months and not beyond
that. The fact that Mr. Z is in-charge for both the offices, the temporary office being set-up in the
plains which is 100 kms away and the regular office kept open during the 3 months does not
constitute any violation of the provisions of the Chartered Accountant Act. Assuming Mr. Z has
informed the Institute regarding such temporary office in the prescribed mann er.
Therefore, in the given case, no penal action needs to be taken on the basis of complaint
registered by Mrs. A, as Mr. Z is not guilty of professional misconduct.
(b) The Operational Audit is not one-time activity. It should be viewed as a continuous
improvement cycle:

Plan

Act Do

Check

The continuous improvement cycle of Operational Audit can be depicted through Plan, Do, Check and Act diagram.

All the significant operations must be subjected to the scrutiny of operational audit, at least, once
in three years. Therefore, the operational audit should be done in the current scenario. However,
to deal with the employee hostility the participative approach of the audit should be adopted .
In this approach the auditor discusses the ideas for improvements with those managers that have
to implement them and make them feel that they have participated in the recommendations made
for improvements. By soliciting the views of the operating personnel, the operational audit
becomes a co-operative enterprise.
This participative approach encourages the auditee to develop a friendly attitude towards the
auditors and look forward to their guidance in a more receptive fashion. When the participative
method is adopted then the resistance to change becomes minimal, feelings of hostility disappear
and gives room for feelings of mutual trust. Team spirit is developed. The auditors and the
auditee together try to achieve the common goal. The proposed recommendations are discussed
with the auditee and modifications as may be agreed upon are incorporated in the operational

© The Institute of Chartered Accountants of India


audit report. With this attitude of the auditor, it becomes absolutely easy to implement the
proposed suggestions as the auditee themselves take initiative for implementing and the auditor
does not have to force any change on the auditee.
(c) Section 2(45) of the Companies Act, 2013, defines a “Government Company” as a company in
which not less than 51% of the paid-up share capital is held by the Central Government or by any
State Government or Governments or partly by the Central Government an d partly by one or
more State Governments, and includes a company which is a subsidiary company of such a
Government company. The auditors of these government companies are firms of Chartered
Accountants, appointed by the Comptroller & Auditor General, who gives the auditor directions on
the manner in which the audit should be conducted by them.
In the given scenario, VM Ltd., a company wholly owned by Central Government was disinvested
during the previous year, resulting in 45% of the shares being held by public. Since, shares were
listed on the BSE therefore all the listing requirements were applicable.
Opinion of Finance Manager of the Company Mr. Gautam that since company is subject to
stringent control by BSE and the markets, therefore the auditing requirements of a limited
company in private sector under the Companies Act 2013 would be applicable to the Company
and the C&AG will not have any role to play, is not correct as listing of company’s shares on a
stock exchange is irrelevant for this purpose.
6. (a) In order to ensure that the manufacturing unit of SP Ltd. will be able to meet the cash
requirements internally, one is required to verify:
(i) Is the company able to honour its commitments to its trade payables, to the banks, to the
government and other stakeholders?
(ii) How well is the company able to convert its trade receivables and inventories?
(iii) How well the Company deploys its funds?
(iv) Are there any funds lying idle or is the company able to reap maximum benefits out of the
available funds?
(v) What is the investment pattern of the company and are they easily realizable?
(b) Audit Programme of Movie Theatre Complex:
(i) Peruse the Memorandum of Association and Articles of Association of the entity.
(ii) Ensure the object clause permits the entity to engage in this type of business.
(iii) In the case of income from sale of tickets:
(1) Verify the control system as to how it is ensured that the collections on sale of tickets
of various shows are properly and accurately accounted.
(2) Verify the system relating to online booking of various shows and the system of
realization of money.
(3) Check that there is overall system of reconciliation of collections with the number of
seats available for different shows in a day.
(iv) Verify the internal control system and its effectiveness relating to the income from café,
shops, pubs, game zone etc., located within the multiplex.
(v) Verify the system of control exercised relating to the income receivable from advertisements
exhibited within the premises and inside the hall such as hoarding, banners, slides, short
films etc.
(vi) Verify the system of collection from the parking areas in respect of the vehicles parked by
the customers.

© The Institute of Chartered Accountants of India


(vii) In the case of payment to the distributors verify the system of payment which may be either
through out right payment or percentage of collection or a combination of both. Ensure at
the time of settlement, any payment of advance made to the distributor is also adjusted
against the amount due.
(viii) Verify the system of payment of salaries and other benefits to the employees and ensure
that statutory requirements are complied with.
(ix) Verify the payments effected in respect of the maintenance of the building and ensure the
same is in order.
(x) Verify the insurance premium paid and ensure it covers the entire assets.
(c) Basic Accounting Control Objectives: The basic accounting control objectives which are
sought to be achieved by any accounting control system are -
(i) Whether all transactions are recorded;
(ii) Whether recorded transactions are real;
(iii) Whether all recorded transactions are properly valued;
(iv) Whether all transactions are recorded timely;
(v) Whether all transactions are properly posted;
(vi) Whether all transactions are properly classified and disclosed;
(vii) Whether all transactions are properly summarized.
or
Real Time Environment: IT Components: To facilitate transactions in real-time, it is essential to
have the systems, networks and applications available during all times. A real-time environment
has several critical IT components that enable anytime, anywhere transactions to take place. Any
failure even in one component could render the real-time system unavailable and could result in a
loss of revenue. IT Components include:
(i) Applications: For example, ERP applications SAP, Oracle E-Business Suite, Core banking
applications.
(ii) Middleware.: For example, Webservers like Apache, Oracle Fusion, IIS.
(iii) Networks: For example, Wide Area Networks, Local Area Network.
(iv) Hardware: For example, Servers, Backup and Storage devices.

10

© The Institute of Chartered Accountants of India


Test Series : April, 2021
MOCK TEST PAPER -II
FINAL (NEW) COURSE: GROUP – I
PAPER – 3: ADVANCED AUDITING AND PROFESSIONAL ETHICS
All MCQs are compulsory
Question No. 1 is compulsory.
Attempt any four questions from the Rest.
Time Allowed – 3 Hours Maximum Marks – 100
DIVISION A – MCQs (30 Marks)
Questions no. (1-10) carry 1 Mark each and Questions no. 11-20 carry 2 Marks each.
1. KB Associates a chartered accountant firm has been appointed as an auditor of the company for the
financial year 2020-21. It consists of two partners CA K & CA B. CA K is brother of the father of the
finance director of the company. CA B is an old friend of the finance director of the company.
What kind of ethical threat is associated with appointment of KB Associates as an auditor of ABC LTD.?
(a) Self Interest Threat.
(b) Advocacy Threat.
(c) Familiarity Threat.
(d) Self-Review Threat.
2. The audit team has obtained the following results from the trade receivables circularization of Nemi Co
for the year ended 31 March 2021.
Customer Balance as per Balance as per Comment
sales ledger customer confirmation
Rs. Rs.
AM Co 2,25,000 2,25,000
AN Co 3,50,000 2,75,000 Invoice raised on 29 March 2021
AO Co 6,20,000 4,80,000 Payment made on 30 March 2021
AP Co 5,35,000 5,35,000 A balance of Rs. 45,000 is currently
being disputed by AP Co.
AR Co 1,78,000 No reply
Which of the following statements in relation to the results of the trade receivables circularisation is
TRUE?
(a) No further audit procedures need to be carried out in relation to the outstanding balances with
AM Co. and AP Co.
(b) The difference in relation to AN Co. represents a timing difference and should be agreed to a pre-
year-end invoice.
(c) The difference in relation to AO Co. represents a timing difference and should be agreed to pre-
year-end bank statements.
(d) Due to the non-reply, the balance with AR Co. cannot be verified and a different customer balance
should be selected and circularized.

© The Institute of Chartered Accountants of India


3. Mr. R, (friend of Mr. P) a CA in practice invited Mr. P to set up a ‘Network Firm’ along with 2 more friends.
All the four auditors agreed to the same and decided to start a network firm by the name M/s RP & Co.
However, one of the auditors suggested that they cannot use the term ‘& Co.’ and it needs to be changed.
But Mr. R informed that there is no such Regulation regarding the firm’s name. Which among the name
shall be suitable to the newly started ‘Network Firm’, in accordance with the provisions of Chartered
Accountant Act and Regulation?
(a) RP and Co.
(b) RP & Associates.
(c) RP and Networks.
(d) RP & Affiliates.
4. AJ Associates a chartered accountant firm is acting as an auditor of the XYZ Ltd. Provisions of the cost
audit are also applicable to the XYZ Ltd. Two years ago, ABC Associates was appointed as cost auditor
of the XYZ Ltd. However, this year due to some reason the Cost Auditor firm resigned. The management
being of the opinion that AJ Associates being the auditor of the company knows everything about the
company so AJ Associates should be appointed as the cost auditor of the XYZ Ltd for the current year.
Select the correct statement with respect to the appointment of AJ Associates as the cost auditor of the
Company?
(i) Practicing CA/CWA/CMA can be appointed as cost auditor of the company, so appointment of AJ
Associates as cost auditor being a company auditor is valid.
(ii) Only Practicing CWA/CMA can be appointed as cost auditor of the company, but appointment of
AJ Associates as cost auditor being a company auditor is invalid.
(iii) Company Auditor can be appointed as cost auditor subject to fulfilment of certain conditions as
specified under section 148 of the Companies Act 2013.
(iv) Company Auditor cannot be appointed as cost auditor of the company.
(a) (i) and (ii).
(b) (ii) and (iv).
(c) (ii) and (iii).
(d) (i) and (iv).
5. During the conduct of audit, it was found that the management has intentionally made material
misstatements in the several items of the financial statements to deceive the users of the financial
statements, to reduce the pressures of meeting market expectations and to increase the reputation of
the company. What would be the implications on the auditor’s report if no adjustments are made to the
financial statements regarding the misstatements made by the management?
(a) The auditor would issue a qualified audit opinion stating that ‘except for’ these matters the financial
statements are fairly presented. The auditor should also include a ‘Basis for Qualified Opinion’
paragraph below the opinion paragraph.
(b) The auditor would issue an adverse audit opinion stating that ‘except for’ these matters the financial
statements are fairly presented. The auditor should also include a ‘Basis for Qualified Opinion’
paragraph below the opinion paragraph.
(c) The auditor would issue an adverse audit opinion stating that financial statements ‘do not give a
true and fair view’. The auditor should also include a ‘Basis for Adverse Opinion’ paragraph below
the opinion paragraph.

© The Institute of Chartered Accountants of India


(d) The auditor would issue an adverse audit opinion stating that financial statements ‘do not g ive a
true and fair view’. The auditor should also include a ‘Basis for Qualified Opinion’ paragraph below
the opinion paragraph.
6. While verifying the salary expense of employees, the auditor has been asked to rely on the values as
per SAP software and some hard copy reports and documents as the HRMS package (source software)
has become corrupt during the year and the management is not having any data backup. How should
the auditor deal with this issue?
(a) The auditor should issue a disclaimer of opinion as records are destroyed and he is unable to
obtain sufficient appropriate audit evidence.
(b) The auditor should perform alternative procedures to obtain sufficient and appropriate audit
evidence before disclaiming the opinion.
(c) The auditor should issue an adverse opinion stating that it is deficiency in internal controls.
(d) The auditor can rely on the SAP data and there is no need for qualification of report.
7. You are the audit manager of Ranker & Co are responsible for the audit work to be managed for the
fixed assets of the company. Ranker & Co has 5 properties amounting to Rs.11.5 crore. One of the
important tasks ahead for you is to confirm the ownership of these properties.
Which of the following would provide the most persuasive evidence of the ownership?
(a) To conduct a physical inspection of all the properties located at different areas.
(b) To ask the management registration documents of these properties and inspect and verify them.
(c) To check whether all the properties are recorded properly in the fixed asset register and
depreciation has been calculated correctly.
(d) Enquire with the management if these properties are insured and review the insurance
documentation.
8. ABC Private Limited uses in-house developed application system for Accounting. The auditor observed
that user ID and password is mandatory to access the application system and felt that this is a good
control. What type of control is this?
(a) IT General Control.
(b) Application Control.
(c) Detective Control.
(d) Preventive Control.
9. While examining the computation of Demand and Time liabilities which of the following is to be included
in liabilities:
(a) Part amounts of recoveries from the borrowers in respect of debts considered bad and doubtful of
recovery.
(b) Amounts received in Indian Currency against import bills and held in sundry deposits pending
receipts of final rates.
(c) Net credit balance in branch adjustment accounts including these relating to foreign branches.
(d) Margins held and kept in sundry deposits for funded facilities.
10. In Case of PSU, Direct Reporting Engagement does not include
(a) Performance audits.
(b) Compliance audits.

© The Institute of Chartered Accountants of India


(c) Financial audits.
(d) Comprehensive Audit. (10 x 1 = 10 Marks)
Questions (11-20) carry 2 Marks each
MCQ 11. -15.
Integrated Case Scenario 1
Arogya Pradhan Limited is a public company incorporated in September 2011 with a registered office in
Chennai. The company is in business of Healthcare services. The company has 151 Ayurvedic clinics and
303 Ayurvedic pharmacies throughout the country.
In the previous year, company achieved turnover of Rs. 3,000 crore and had earned the Net Profit of Rs. 25
crore. The company had borrowed a term loan of Rs. 100 crore from State Bank of India.
M/s Bright Moon LLP are appointed as statutory Auditors of the company for the year 2019 -20. After
completing the initial engagement procedures and audit planning, the audit team started with the verificati on
of Internal Financial Controls of the company.
While understanding the controls established by the management in the ‘Revenue Process’, the audit team
observed that there is only one Review Control wherein 20 executives had to prepare the sales invoice and
Mr. Darshan - Sales Manager, had to review and authorise all the invoices. It was observed that on many
occasions, Mr. Darshan had more than 1000 invoices to authorise in a single day. Further, he has frequently
asked 2 senior most executive to review pending invoices and he has relied on them by directly giving his
authorization on the invoice. It was observed that Mr. Darshan did not take any leave during the entire year.
It was observed that Mr. Darshan’s performance bonus was linked with number of invoices authorised by him.
In addition, Mr. Darshan was the sole authority to approve the sales commission and sales discount which
was to be applied by the customers.
The audit team has set Rs. 30 crore as materiality based on 1% of Turnover. For selecting the samples,
Mr. Santosh – Audit Executive, used the below mentioned formula:-
Ledger Balance * 100
Materialit y * 365 days
Mr. Santosh selected 30 samples for the verification of above mentioned “Review Control”. It was observed
that out of 30 samples, 20 samples had irregularities in invoices which was clearly due to improper functioning
of review control. The amount of irregularity in 20 invoices amounted to Rs. 4 crore. The auditor still issued
the clean audit report and took the written representation letter from the management for efficient
implementation of Internal Financial Controls.
On the basis of the abovementioned facts, you are required to answer the following MCQs:
Multiple Choice Questions (5 questions of 2 Marks each):
11. Is the Control “Designed” appropriately?
(a) No, because there are irregularities amounting to Rs. 4 crore in the samples selected by Auditor s.
(b) No, because Mr. Darshan’s performance bonus is linked with number of invoices authorised.
(c) No, because there are no leave taken by Mr. Darshan during the entire year.
(d) Yes. ‘Review Control’ is designed appropriately.
12. Is the Control “Implemented” effectively?
(a) No, because Mr. Darshan delegates his work of review to other executives who are senior in
experience.
(b) No, because Mr. Darshan is heavily burdened with excessive work.

© The Institute of Chartered Accountants of India


(c) No, because Mr. Darshan is authorized to finalize sales commission and sales discount.
(d) Yes. ‘Review Control’ is implemented appropriately.
13. In the above case, to whom should M/s Bright Moon LLP report first?
(a) To ROC.
(b) To Central Government u/s 143(12) of Companies Act, 2013 as the im pact in above case is more
than Rs. 1 crore.
(c) To Those Charged with Governance.
(d) To Shareholders as they are the appointing authority.
14. How samples are to be selected for the purpose of verification of Internal Financial Control?
(a) Monetary Unit Sampling.
(b) Sampling based on Materiality.
(c) Sampling based on probability of default of the control.
(d) Sampling based on frequency of functioning of control.
15. In the current scenario, how should M/s Bright Moon LLP report?
(a) Clean Report with Other Matter paragraph.
(b) Clean Report with Emphasis of Matter paragraph.
(c) Clean Report with reporting in Key Audit Matter.
(d) Qualified Report (both main report and ICFR report).
MCQ 16. -20.
Integrated Case Scenario 2
Andy & Co; a reputed Chartered Accountants firm is appointed as a statutory auditor of Manava Swaroopam
Limited. The Company is into manufacturing of copper products. The company has advanced in all its
endeavours by supplying million Copper units. The company has incorpor ated another company “Daiva
Swaroopam Private Limited” by investing 45% in the share capital of the company and at the same time
having 100% control over the Board of Directors as per the agreement with the majority shareholder. The
company is listed in the US Stock Exchange but in the process of listing in the Indian Stock Exchanges,
having a net worth of INR 245 crore. The product is promoted by Ali Baba, as its product Brand Ambassador.
You are the audit manager in-charge of the audit team this year and your 1 st year trainee asks you the
following questions listed down. He has also noted down some of the questions for you to answer to discuss
the impact on the planning stage after understanding the entity and its environment:
• The company is required to appoint the Internal Auditor as per provisions of the Companies Act, 2013
and the company complied with the same by delegating the duties to an employee, who joined the
company as 1st year Architect. The audit team is planning to use the work performed by the Internal
Audit function as the reports given by him are designed in a marvellous fashion. Even the Board of
Directors are astonished by the design of the Internal Audit report.
• The company is planning to use the working papers of the previous auditor by demanding the audit
working papers from him citing the confidentiality clause. The auditor also plans to use the same for
testing the opening balances during the year. The previous year auditor having been appointed as the
auditor of subsidiary; the company plans to use his work for verifying the investment balance during the
year.

© The Institute of Chartered Accountants of India


On the basis of the abovementioned facts, you are required to answer the following MCQs:
Multiple Choice Questions (5 questions of 2 Marks each):
16. The engagement partner has requested you to comment upon the usage of work of Internal auditor by
the engagement team in accordance with relevant Standard on Auditing:
(a) As the work done by the internal auditor is marvellously designed and presented the same can be
considered to the extent the statutory auditor can use it. As the work is highly appreciated even by
the Board of Directors, the same should be definitely used by Andy & Co.
(b) The work done by the Internal Auditor need to be assessed for the sufficiency and should be used
to avoid the double work. The audit team of Andy & Co need to reduce the unnecessary work as
the same has been performed by the other auditor.
(c) The auditor is required to assess the competence and professional care of the work performed by
the Internal Auditor. Thus, the auditor Andy & Co needs to reconsider the audit strategy and cannot
use the work of the Internal Auditor.
(d) The work performed by the internal auditor can be used by the External Auditor in this case if the
architect is not an employee of the company but is in private practice.
17. The Trainee asked whether the audit team is to perform any procedures over the investment in Daiva
Swaroopam Private Limited:
(a) The company need to prepare the consolidated financial statements and the same need to be
audited by the auditor and the auditor needs to consider the financial information and also assess
regarding the need to use of the work of the component auditor.
(b) The auditor needs to perform audit procedures over the balances in investments and transactions
with its related party.
(c) The auditor need not perform any procedures as the investment in Daiva Swaroopam Private
Limited has already been made in the previous year.
(d) Both (a) & (b).
18 The trainee asked about role of auditor in case the investment in Daiva Swaroopam Privat e Limited is
increased to 60% in the next year:
(a) The auditor need not do any additional procedures compared to this year except for audit
procedures over the increase in Investment value and its disclosures in the Financial Statements.
(b) The auditor should also audit the group consolidated financial statements as the consolidation
becomes applicable for the company being the investment is raised from 45% to 60%.
(c) The auditor needs to audit the subsidiary’s books of accounts to get comfort over th e balances in
the material subsidiary. Thus, the audit strategy will change for verifying the investment.
(d) The auditor can either on its own, audit the subsidiary or use the work of another auditor to get
comfort over the balances in the subsidiary from the next year.
19. The company has requested its previous auditor to give back its audit documentation (“working papers”)
and warned the previous auditor with legal notice to submit them back to the company showing the
confidentiality clause:
(a) The previous auditor is bound to return the workpapers as the company has raised the
confidentiality clause over the audit firm. Thus, the SA – 230 is not applicable in such scenario as
the original owner itself is requesting to return the working papers.
(b) The auditor has a right over its working paper, and he is the owner of the workpapers but he cannot
give the workpapers to any person even at the request of the company.

© The Institute of Chartered Accountants of India


(c) The auditor has a right over its working paper, and he is the owner of the workp apers and he may
give at his discretion make available the workpapers to the company.
(d) The auditor has a right over its working papers but the owner of them is the company. He should
make available the workpapers to the company at its request and SQC-1 mandates the auditor to
make copies made available to its clients.
20. The trainee asked you whether the IND AS is applicable to the group or not?
(a) Yes, but only Manava Swaroopam Limited need to prepare its financial statements as per the
Companies (Indian Accounting Standards Rules), 2015.
(b) Yes, the Company Manava Swaroopam Limited and its subsidiaries (including associates) need to
prepare its financial statements as per the Companies (Indian Accounting Standards Rules), 2015.
(c) The Company is not required to prepare financial statements as per Companies (Indian Accounting
Standards Rules), 2015 as the company’s net worth is below 250 crore and is not listed in any
recognised stock exchange in India.
(d) The Company is required to prepare books of accounts as per US GAAP as it is listed in US Stock
Exchange and get the books audited by the CPA but not the Indian Chartered Accountant.
(10 x 2 = 20 Marks)
Division B- Descriptive Questions-70 Marks
Question No. 1 is compulsory.
Attempt any four questions from the Rest.
1. Comment on the following:
(a) ANUSHA Associates, Chartered Accountants, conducting the audit of Rishabh Ltd., a listed
company for the year ended 31 st March 2021 is concerned with the auditor's responsibilities relating
to other information, both financial and non-financial, included in the Company’s annual report.
While reading other information, ANUSHA Associates considers whether there is a material
inconsistency between other information and the financial statements. As a basis for the
consideration the auditor shall evaluate their consistency, compare selected amounts or other
items in the other information with such amounts or other items in the financial statements. Guide
ANUSHA Associates with examples of "Amounts" or "other items" that may be included in the "other
information" with reference to SA 720. (5 Marks)
(b) AMRO Limited is availing the services of ABN Private Limited for its payroll operations. Payroll
cost accounts for 67% of total cost for AMRO Limited. ABN Limited has provided the type 2 report
as specified under SA 402 for its description, design and operating effectiveness of control.
ABN Private Limited has also outsourced a material part of payroll operation M/s RST & Associates
in such a way that M/s RST & Associates is sub-service organization to AMRO Limited. The Type
2 report which was provided by ABN Private Limited was based on carve-out method as specified
under SA 402.
CA Param while reviewing the unmodified audit report drafted by his assis tant found that, a
reference has been made to the work done by the service auditor. CA Param hence asked his
assistant to remove such reference and modify report accordingly.
Comment whether CA Param is correct in removing the reference of the work done by service
auditor? (5 Marks)
(c) While verifying the employee records in a company, it was found that a major portion of the labour
employed was child labour. On questioning the management, the auditor was told that it was
outside his scope of the financial audit to look into the compliance with other laws. Comment in
accordance with relevant Standards on Auditing. (4 Marks)
7

© The Institute of Chartered Accountants of India


2. (a) BN Limited is a listed entity having two subsidiaries namely BUS Limited and ROBUS Limited. Both
the subsidiaries are unlisted and are incorporated in Australia. The consolidated net worth of BN
Limited and its subsidiaries is Rs. 350 crore (including net worth of BUS Limited and ROBUS
Limited Rs. 36 crore & Rs. 80 crore respectively) in the immediately preceding year 2019-20. On
observing this fact, your senior manager advises you to inform the management of BN limited to
make certain changes in the board of directors of both the subsidiaries, in accordance with the
LODR regulation 24(1). Comment. (5 Marks)
(b) The Property, Plant and Equipment of Peer Ltd. included Rs.39.61 crore of earth removing
machines of outdated technology which had been retired from active use and had been kept for
disposal after knock down. These assets appeared at residual value and had been last inspected
seven years back. As an Auditor, what may be your reporting concern in view of CARO, 2016 on
matters specified above? (4 Marks)
(c) Mr. Sudhir, a Chartered Accountant in practice, delivered a speech in the national conference
organized by the Ministry of Textiles. While delivering the speech, he told to the audience that he
is a management expert and his firm provides services of taxation and audit at reasonable rates.
He also requested the audience to approach his firm of chartered accountants for these services
and at the request of audience he also distributed his business cards and telephone number of his
firm to those in the audience. Comment with reference to the Chartered Accountants Act, 1949,
and Schedules thereto. (5 Marks)
3. (a) You have been appointed to carry out the audit of Blue Insurance Company Ltd. for the year
2020-21. In the course of your audit, you observed that the commission paid to agents constituted
a major expense in operating expenses of the Company. Enumerate the audit concerns that
address to the assertions required for the Auditor to ensure the continued existence of internal
control as well as fairness of the amounts in accounting of commission paid to agents. (4 Marks)
(b) (i) Mr. Shantinath was appointed as an auditor of RST Co. During the course of audit he finds
that the company has availed an excess ITC of Rs. 40 lakh. Accordingly, he made
recommendations in Part V of GSTR-9C. The management of the company refused to pay
the excess ITC and argued that auditors recommendations are not binding. Comment.
(ii) Mr. Bahubali, made an outward supply of Rs. 4.00 lakh to M/s. S.S. Enterprises on 30 th April,
2020 on a credit period of 15 days. However, M/s. S.S. Enterprises made the payment to
Mr. Bahubali after 45 days along with interest for 30 days delayed payment @ 12%. As such,
Mr. Bahubali received total payment of Rs. 4,04,000/- along with interest. However, while
filing Form GSTR-3B/ Form GSTR-1, Mr. Bahubali declared his outward supplies at Rs. 4.00
lakh. Even while filing Form GSTR-9, Mr. Bahubali did not discharge his tax liability. As a GST
auditor in Form GSTR-9C, what action is recommended by the auditors. Comment.
(6 Marks)
(c) Mr. Dhawal, a practicing CA, is appointed as a Director Simplicitor in Gautam Pvt. Ltd. After three
year of appointment, Mr. Dhawal resigned as the Director and accepted the Statutory Auditor
position of the Company. Is Mr. Dhawal right in accepting the auditor position? Comment with
reference to the Chartered Accountants Act, 1949, and Schedules thereto. (4 Marks)
4. (a) Mr. Shripal has been appointed as an auditor of ASG Ltd., a NBFC company registered with RBI.
Mr. Shripal is concerned about whether the format of financial statements prepared by ASG Ltd. is
as per notification issued by the Ministry of Corporate Affairs (MCA) dated October 11, 2018. The
notification prescribed the· format in Division III under Schedule III of the Companies Act, 2013
applicable to NBFCs complying with Ind-AS. Mr. Shripal wants to know the differences in the
presentation requirements between Division II and Division III of Schedule III of the Companies
Act, 2013. Help Mr. Shripal. (6 Marks)

© The Institute of Chartered Accountants of India


(b) Mr. Sheetal is appointed as a statutory auditor of Mahi Ltd. Mahi Ltd is required to appoint an
internal auditor as per statutory provisions given in the Companies Act, 2013 and appointed
Mr. Kunthu as its internal auditor. The external auditor Mr. Sheetal asked internal auditor to provide
direct assistance to him regarding evaluating the sufficiency of tests performed and the adequacy
of disclosures in the financial statements and other matters affecting the auditor’s report. Discuss
whether Mr. Sheetal, statutory auditor, can ask direct assistance from Mr. Kunthu, internal auditor
as stated above in view of relevant Standard on Auditing. (4 Marks)
(c) CA. Rani is practicing since 2007 in the field of tax audit. Due to her good practical knowledge, she
was offered editorship of a ‘Tax Audit’ Journal which she accepted. However, she did not take any
permission from the Council regarding such editorship. Comment with reference to the Chartered
Accountants Act, 1949, and Schedules thereto. (4 Marks)
5. (a) SSP Bank was engaged in the business of providing Portfolio Management Services to its
customers, for which it took prior approval from RBI. Your firm has been appointed as the statutory
auditors of the Bank’s financial statements for the year 2019-20. Your senior has instructed you to
verify the transactions of Portfolio Management Services (PMS). While verifying the transactions
you noticed that the bank has not maintained separate record for PMS transactions from the Bank’s
own investments. As a statutory auditor what methodology will be adopted by you for verification
of PMS transactions? (4 Marks)
(b) During the process of extracting the exception reports, the auditors noted numerous purchase
entries without valid purchase orders. In terms of percentage, about 40% of purchases were made
without valid purchase orders whereas few purchase orders were validated after the actual
purchase. Also, there was no reconciliation between the goods received and the goods ordered.
You are required to briefly explain the audit procedures to address the validity of account balance
level. (4 Marks)
(c) RAJUL & Co.” is an Audit Firm having partners “Mr. R”, “Mr. A”, “Mr. J”, “Mr. U” and “Mr. L”,
Chartered Accountants. “Mr. R”, “Mr. A”, “Mr. J”, “Mr. U” and “Mr. L” are holding appointment as an
Auditor in 4, 5, 6, 10 and 15 Companies respectively.
(i) Provide the maximum number of Audits remaining in the name of “RAJUL & Co.”
(ii) Provide the maximum number of Audits remaining in the name of individual partner i.e.
“Mr. R”, “Mr. A”, “Mr. J”, “Mr. U” and “Mr. L”.
(iii) Can RAJUL & Co. accept the appointment as an auditor in 80 private companies having paid-
up share capital less than Rs. 100 crore which has not committed default in filing its financial
statements under section 137 or annual return under section 92 of the Companies Act with
the Registrar, 2 small companies and 1 dormant company?
(iv) Would your answer be different, if out of those 80 private companies, 65 companies are having
paid-up share capital of Rs. 115 crore each? (6 Marks)
6. (a) Sunshine Limited is a public sector undertaking engaged in production of electricity from solar
power. It had commissioned a new project near Puducherry with a new technology for a cost of Rs.
4,926 crore. The project had seen delay in commencement and cost overrun. State the matters
that a Comprehensive Audit by C&AG may cover in reporting on the performance and efficiency of
this project. (5 Marks)

© The Institute of Chartered Accountants of India


(b) Atishaya Ltd. holds the ownership of 10% of voting power and control over the composition of
Board of Directors of Neenu Ltd. While planning the statutory audit of Atishaya Ltd., what factors
would be considered by you as the statutory auditors of Atishaya Ltd for the audit of its consolidated
financial statements prepared under Ind AS? (5 Marks)
(c) While auditing REAL Ltd., you observe certain material financial statement assertions have been
based on estimates made by the management. As the auditor how do you minimize the risk of
material misstatements? (4 Marks)
OR
Briefly explain the difference between Peer Review and Quality Review. (4 Marks)

10

© The Institute of Chartered Accountants of India


Test Series: April, 2021
MOCK TEST PAPER - 2
FINAL (NEW) COURSE: GROUP – I
PAPER – 3: ADVANCED AUDITING AND PROFESSIONAL ETHICS
SUGGESTED ANSWERS/HINTS
DIVISION A - MCQs (30 Marks)
Questions no. (1-10) carry 1 Mark each and Questions no. 11-20 carry 2 Marks each.
1. (c)
2. (b)
3. (d)
4. (b)
5. (c)
6. (b)
7. (b)
8. (d)
9. (c)
10. (c)
Questions (11-20) carry 2 Marks each
11. (b)
12. (a)
13. (c)
14. (d)
15. (d)
16. (c)
17. (d)
18. (a)
19. (c)
20. (b)
DIVISION B - DESCRIPTIVE QUESTIONS (70 Marks)
1. (a) Examples of Amounts or Other Items that May Be Included in the Other Information : As per
SA 720 “The Auditor’s Responsibility in Relation to Other Information”, the following are
examples of amounts and other items that may be included in other information. This list is not
intended to be exhaustive.
Amounts
(i) Items in a summary of key financial results, such as net income, earnings per share,
dividends, sales and other operating revenues, and purchases and operating expenses.
(ii) Selected operating data, such as income from continuing operations by major operating
area, or sales by geographical segment or product line.

1
(iii) Special items, such as asset dispositions, litigation provisions, asset impairments, tax
adjustments, environmental remediation provisions, and restructuring and reorganization
expenses.
(iv) Liquidity and capital resource information, such as cash, cash equivalents and marketable
securities; dividends; and debt, capital lease and minority interest obligations.
(v) Capital expenditures by segment or division.
(vi) Amounts involved in, and related financial effects of, off-balance sheet arrangements.
(vii) Amounts involved in guarantees, contractual obligations, legal or environmental claims, and
other contingencies.
(viii) Financial measures or ratios, such as gross margin, return on average capital employed,
return on average shareholders’ equity, current ratio, interest coverage ratio and debt ratio.
Some of these may be directly reconcilable to the financial statements.
Other Items
(i) Explanations of critical accounting estimates and related assumptions.
(ii) Identification of related parties and descriptions of transactions with them.
(iii) Articulation of the entity’s policies or approach to manage commodity, foreign exchange or
interest rate risks, such as through the use of forward contracts, interest rate swaps, or
other financial instruments.
(iv) Descriptions of the nature of off-balance sheet arrangements.
(v) Descriptions of guarantees, indemnifications, contractual obligations, litigation or
environmental liability cases, and other contingencies, including management’s qualitative
assessments of the entity’s related exposures.
(vi) Descriptions of changes in legal or regulatory requirements, such as new tax or
environmental regulations, that have materially impacted the entity’s operations or fiscal
position, or will have a material impact on the entity’s future financial prospects.
(vii) Management’s qualitative assessments of the impacts of new financial reporting standards
that have come into effect during the period, or will come into effect in the following period,
on the entity’s financial results, financial position and cash flows.
(viii) General descriptions of the business environment and outlook.
(ix) Overview of strategy.
(x) Descriptions of trends in market prices of key commodities or raw materials.
(xi) Contrasts of supply, demand and regulatory circumstances between geographic regions.
(xii) Explanations of specific factors influencing the entity’s profitability in specific segments.
(b) Reporting by the User Auditor: As per SA 402, “Audit Considerations Relating to an Entity
Using a Service Organisation”, the user auditor shall modify the opinion in the user auditor’s
report in accordance with SA 705, “Modifications to the Opinion in the Independent Auditor’s
Report”, if the user auditor is unable to obtain sufficient appropriate audit evidence regarding the
services provided by the service organisation relevant to the audit of the user entity’s financial
statements.
The user auditor shall not refer to the work of a service auditor in the user auditor’s report
containing an unmodified opinion unless required by law or regulation to do so. If such reference
is required by law or regulation, the user auditor’s report shall indicate that the reference does
not diminish the user auditor’s responsibility for the audit opinion.

2
Thus, in view of above, contention of CA. Param in removing reference of the work done by
service auditor is in order as in case of unmodified audit report, user auditor cannot refer to the
work done by service auditor.
(c) Compliance with Other Laws: As per SA 250, “Consideration of Laws and Regulations in an
Audit of Financial Statements”, the auditor shall obtain sufficient appropriate audit evidence
regarding compliance with the provisions of those laws and regulations generally recognised to
have a direct effect on the determination of material amounts and disclosures in the financial
statements including tax and labour laws.
Further, non-compliance with other laws and regulations may result in fines, litigation or other
consequences for the entity, the costs of which may need to be provided for in the financial
statements, but are not considered to have a direct effect on the financial statements.
In the instant case, major portion of the labour employed in the company was child labour. While
questioning by auditor, reply of the management that it was outside his scope of financial audit to
look into the compliance with other laws is not acceptable as it may have a material effect on
financial statements.
Thus, auditor should ensure the disclosure of above fact and provision for the cos t of fines,
litigation or other consequences for the entity. In case if the auditor concludes that non-
compliance has a material effect on the financial statements and has not been adequately
reflected in the financial statements, the auditor shall express a qualified or adverse opinion on
the financial statement as per SA 705 “Modifications to the Opinion in the Independent Auditor’s
Report”.
2. (a) As per SEBI - LODR Regulation 24(1), at least one independent director on the board of directors
of the listed entity shall be a director on the board of directors of the unlisted material subsidiary,
whether incorporated in India or not.
For the purpose of Regulation 24(1), notwithstanding anything to the contrary contained in
Regulation 16, the term ‘material subsidiary’ means a subsidiary, whose income or net worth
exceeds 20% of the consolidated income or net worth respectively, of the listed entity and its
subsidiaries in the immediately preceding accounting year.
In the given case, the fact that both the subsidiaries are unlisted and incorporated outside India is
irrelevant. From the above case we have the following details:
 20% of consolidated net worth of BN Limited = 350*20% = Rs. 70 crore.
 Net worth of BUS Limited = Rs. 36 crore.
 Net worth of ROBUS Limited = Rs. 80 crore.
Accordingly, it is clear that out of the two subsidiaries, the net worth of only one subsidiary (ie.
ROBUS Limited) exceeds 20% of the consolidated net worth of BN Limited and all its
subsidiaries. Therefore, the change in the composition of board of directors needs to be made
only for ROBUS Limited and not both the subsidiaries.
Thus, contention of senior manager regarding change in composition of board of directors in BUS
Limited is not in order as per Regulation 24(1). However, change in composition of board of
director is required for ROBUS Limited.
Further, as per Regulation 24 (1) of LODR Regulation, 2015 one of the independent directors
present in the board of director of BN Limited should also be made as a director in the board of
directors of ROBUS Limited.

3
(b) Disclosure in Audit Report: The auditor is required to specifically include certain matters as per
CARO, 2016 under section 143 of the Companies Act, 2013.
According to clause (i) (a) of CARO, 2016 the auditor has to comment whether the company is
maintaining proper records showing full particulars, including quantitative details and situation of
fixed assets; and as per clause (i) (b) whether these fixed assets have been physically verifi ed by
the management at reasonable intervals; whether any material discrepancies were noticed on
such verification and if so, whether the same have been properly dealt with in the books of
account;
In the given case, Peer Ltd. has intention to sale its earth removing machines of outdated
technology which had been retired from active use and had been kept for disposal after knock
down and these assets are appearing at residual value. Further, inspection of such machines
(though it is a retired machine, however value is 39.61 crore which is material amount) was done
seven years back, is not in compliance with CARO, 2016.
Hence, this fact needs to be disclosed in the Audit Report as per clause (i) (a) and (b) of
Paragraph 3 of CARO 2016.
(c) Using Designation Other Than a CA and Providing Details of Services Offered:
Clause (6) of Part I of the First Schedule to the Chartered Accountants Act, 1949 states that a
Chartered Accountant in practice shall be deemed to be guilty of misconduct if he solicits client s
or professional work either directly or indirectly by a circular, advertisement, personal
communication or interview or by any other means. Such a restraint has been put so that the
members maintain their independence of judgment and may be able to command respect from
their prospective clients.
Section 7 of the Chartered Accountants Act, 1949 read with Clause (7) of Part I of the First
Schedule to the said Act prohibits advertising of professional attainments or services of a
member. It also restrains a member from using any designation or expression other than that of a
chartered accountant in documents through which the professional attainments of the member
would come to the notice of the public. Under the clause, use of any designation or expression
other than chartered accountant for a chartered accountant in practice, on professional
documents, visiting cards, etc. amounts to a misconduct unless it be a degree of a university or a
title indicating membership of any other professional body recognised by the Central Government
or the Council.
Member may appear on television and films and agree to broadcast in the Radio or give lectures
at forums and may give their names and describe themselves as Chartered Accountants. Special
qualifications or specialized knowledge directly relevant to the subject matter of the programme
may also be given but no reference should be made, in the case of practicing member to the
name and address or services of his firm. What he may say or write must not be promotional of
his or his firm but must be an objective professional view of the topic under consideration.
Thus, it is improper to use designation "Management Expert" since neither it is a degree of a
University established by law in India or recognised by the Central Government nor it is a
recognised professional membership by the Central Government or the Council. Therefore, CA.
Sudhir is deemed to be guilty of professional misconduct under both Clause (6) and Clause (7)
as he has used the designation “Management Expert” in his speech and also he has made
reference to the services provided by his firm of Chartered Accountants at reasonable rates.
Distribution of cards to audience is also a misconduct in terms of Clause (6).
3. (a) Commission: The commission is the consideration payable for getting the insurance business.
The term ‘commission’ is used for the payment of consideration to get Direct business.
Commission received on amount of premium paid to a re-insurer is termed ‘Commission on
reinsurance accepted’ and is reduced from the amount of commission expenditure. The internal

4
control with regard to commission is aimed at ensuring that commission is paid in accordance
with the rules and regulations of the company and in accordance with the agreement with th e
agent, commission is paid to the agent who brought the business and the legal compliances, for
example, tax deduction at sources, GST on reverse charge mechanism and provisions of the
Insurance Act, 1938 have been complied with.
Role of Auditor: The auditor should, inter alia, do the following for verification of commission:
 Ensure that commission/brokerage is not paid in excess of the limits specified by IRDAI
 Ensure that commission/brokerage is paid as per rates with the agent and rates filed with
IRDAI
 Ensure that commission/brokerage is paid to the agent/broker who has solicited the
business
 Ensure that the agent/broker is not blacklisted by IRDAI and is not terminated for fraud etc.
 Vouch disbursement entries with reference to the disbursement vouchers with copies of
commission bills and commission statements.
 Check whether the vouchers are authorised by the officers-in–charge as per rules in force
and income tax is deducted at source, as applicable.
 Test check correctness of amounts of commission allowed.
 Scrutinise agents’ ledger and the balances, examine accounts having debit balances, if any,
and obtain information on the same. Necessary rectification of accounts and other remedial
actions have to be considered.
 Check whether commission outgo for the period under audit been duly accounted.
(b) (i) The management contention is correct. The auditor has only a recommendatory power, for
recommendations given by the auditor may or may not be acceptable to the registered
person. The registered person has the option to accept, reject or partially accept the
recommended additional tax liability. In line with such recommendations, though not
explicitly stated anywhere in the relevant Form or GST law –
 the registered person can choose to make the payment of the additional tax liability in
full or in part;
 the registered person can even choose to reject the complete recommendations of the
auditor and not make the payment at all.
(ii) In terms of section 15(2)(d) while computing value of taxable supply the transaction value
shall include interest or late fee or penalty for delayed payment of any consideration for any
supply. Since, Mr. Bahubali did not pay tax on interest component, he made violation of
valuation provisions. Mr. Bahubali was having option to discharge such liability at the time of
filing of Form GSTR-9, which he did not avail. Therefore, the GST auditor may recommend
him to discharge such liability at the time of making reconciliation statement in Form GSTR-
9C.
(c) As per Clause (4) of Part I of the Second Schedule of the Chartered Accountants Act, 1949, a
Chartered Accountant in practice is deemed to be guilty of professional misconduct if he
expresses his opinion on financial statements of any business or enterprise in which he, his firm,
or a partner in his firm has a substantial interest.
Section 141 of the Companies Act, 2013 specifically prohibits a member from auditing the
accounts of a company in which he is an officer or employee. Although the provisions of the
aforesaid section are not specifically applicable in the context of audits performed under other

5
statutes, e.g. tax audit, yet the underlying principle of independence of mind is equally applicable
in those situations also. Therefore, the Council’s views are clarified in the following situations.
As per the clarifications issued by the Council, a member shall not accept the assignment of audit
of a Company for a period of two years from the date of completion of his tenure as Director, or
resignation as Director of the said Company.
In the instant case, Mr. Dhawal, a practicing CA, is appointed as a Director Simplicitor in Gautam
Pvt. Ltd. After three year of appointment, Mr. Dhawal resigned as the Director and accepted the
Statutory Auditor position of the Company. In view of above provisions Mr. Dhawal can accept
the Directorship of the company as tenure of two years after his resignation is completed.
Thus, CA, Dhawal would not be held guilty of professional misconduct under clause 4 of Part 1 of
Second Schedule of the Chartered Accountants Act, 1949.
4. (a) Differences between Division II (Ind- AS- Other than NBFCs) and Division III (Ind- AS- NBFCs) of
Schedule III: The presentation requirements under Division III for NBFCs are similar to Division II
(Non NBFC) to a large extent except for the following:
(i) NBFCs have been allowed to present the items of the balance sheet in order of their
liquidity which is not allowed to companies required to follow Division II. Additionally, NBFCs
are required to classify items of the balance sheet into financial and non-financial whereas
other companies are required to classify the items into current and non-current.
(ii) An NBFC is required to separately disclose by way of a note any item of ‘other income’ or
‘other expenditure’ which exceeds 1 per cent of the total income. Division II, on the other
hand, requires disclosure for any item of income or expenditure which exceeds 1 per cent of
the revenue from operations or Rs.10 lakhs, whichever is higher.
(iii) NBFCs are required to separately disclose under ‘receivables’, the debts due from any
Limited Liability Partnership (LLP) in which its director is a partner or member.
(iv) NBFCs are also required to disclose items comprising ‘revenue from operations’ and ‘other
comprehensive income’ on the face of the Statement of profit and loss instead of showing
those only as part of the notes.
(v) Separate disclosure of trade receivable which have significant increase in credit risk & credit
impaired
(vi) The conditions or restrictions for distribution attached to statutory reserves have to be
separately disclose in the notes as stipulated by the relevant statute.
(b) Direct Assistance from Internal Auditor: As per SA 610 “Using the Work of Internal Auditor”,
the external auditor shall not use internal auditors to provide direct assistance to perform
procedures that Involve making significant judgments in the audit.
Since the external auditor has sole responsibility for the audit opinion expressed, the external
auditor needs to make the significant judgments in the audit engagement.
Significant judgments include the following:
 Assessing the risks of material misstatement;
 Evaluating the sufficiency of tests performed;
 Evaluating the appropriateness of management’s use of the going concern assumption;
 Evaluating significant accounting estimates; and
 Evaluating the adequacy of disclosures in the financial statements, and other matters
affecting the auditor’s report.

6
In view of above, Mr. Sheetal cannot ask direct assistance from internal auditors regarding
evaluating the sufficiency of tests performed and the adequacy of disclosures in the financial
statements and other matters affecting the auditor’s report.
(c) Permission from the Council: As per Clause (11) of Part I of First Schedule to the Chartered
Accountants Act, 1949, a Chartered Accountant in practice will be deemed to be guilty of
professional misconduct if he engages in any business or occupation other than the profession of
Chartered Accountant unless permitted by the Council so to engage.
However, the Council has granted general permission to the members to engage in certain
specific occupation. In respect of all other occupations specific permission of the Institute is
necessary.
In the instant case, CA. Rani accepted editorship of a journal for which she did not take any
permission from the Council. In this context, it may be noted that the editorship of professional
journals is covered under the general permission and specific permission is not required.
Therefore, CA. Rani shall not be held guilty of professional misconduct in terms of Clause (11) of
Part I of First Schedule to the Chartered Accountants Act, 1949.
5. (a) Separation of Investment Functions: The auditor needs to examine whether the bank, as
required by the RBI, is maintaining separate accounts for the investments made by it on their own
Investment Account, PMS clients’ account, and on behalf of other Constituents (including
brokers). As per the RBI guidelines, banks are required to get their investments under PMS
separately audited by external auditors.
Thus, in the instant case, SSP Bank is required to prepare separate records for PMS and as per
RBI guidelines PMS investments need to be audited separately by the external auditors and the
auditors are required to give a certificate separately for the same. So, in the above case the
auditor should not verify the PMS transactions and advise the bank to segregate the PMS
transactions from its own investments and provide the certificate of external auditor as described
above. In case SSP Bank does not provide the same the auditor may report accordingly.
(b) In the given scenario, the auditors noted numerous purchase entries without valid purchase
orders during the process of extracting the exception reports. Further, in terms of percentage,
about 40% of purchases were made without valid purchase orders and also few purchase orders
were validated after the actual purchase. Also there was no reconciliation between the goods
received and the goods ordered.
Audit Procedures: The following procedures may address the validity of the account balance:
 Make a selection of the purchases, review correspondence with the vendors, purchase
requisitions (internal document) and reconciliations of their accounts.
 Review Vendor listing along with the ageing details. Follow up the material amounts paid
before the normal credit period and analyse the reasons for exceptions.
 Meet with the company's Purchase officer and obtain responses to our inquiries regarding
the purchases made without purchase orders.
 Discuss the summary of such issues with the client.
(c) In the instant case, Mr. R is holding appointment in 4 companies, Mr. A is holding appointment in
5 companies, Mr. J is holding appointment in 6 companies, whereas Mr. U is having appointment
in 10 Companies and Mr. L is having appointment in 15 Companies. In aggregate all five partners
are having 40 audits.
Provisions and Explanations: As per section 141(3)(g) of the Companies Act, 2013, a person
shall not be eligible for appointment as an auditor if he is in full time employment elsewhere or a
person or a partner of a firm holding appointment as its auditor, if such person or partn er is at the

7
date of such appointment or reappointment holding appointment as auditor of more than twenty
companies other than one person companies, dormant companies, small companies and private
companies having paid-up share capital less than Rs. 100 crore (private company which has not
committed a default in filing its financial statements under section 137 of the said Act or annual
return under section 92 of the said Act with the Registrar).
As per section 141(3)(g), this limit of 20 company audits is per person. In the case of an audit
firm having 5 partners, the overall ceiling will be 5 × 20 = 100 company audits. Some times, a
Chartered Accountant is a partner in a number of auditing firms. In such a case, all the firms in
which he is partner or proprietor will be together entitled to 20 company audits on his account.
Conclusion:
(i) Therefore, RAJUL & Co. can hold appointment as an auditor of 60 more companies:
Total Number of Audits available to the Firm = 20*5 = 100
Number of Audits already taken by all the partners
In their individual capacity = 4+5+6+10+15 = 40
Remaining number of Audits available to the Firm = 60
(ii) With reference to above provisions, an auditor can hold more appointment as auditor =
ceiling limit as per section 141(3)(g)- already holding appointments as an auditor. Hence (1)
Mr. R can hold: 20 - 4 = 16 more audits. (2) Mr. A can hold: 20 - 5 = 15 more audits. (3) Mr.
J can hold: 20 - 6 = 14 more audits. (4) Mr. U can hold 20-10 = 10 more audits and (5) Mr. L
can hold 20-15 = 5 more audits.
(iii) In view of above discussed provisions, RAJUL & Co. can hold appointment as an auditor in
all the 80 private companies having paid-up share capital less than Rs. 100 crore (private
company which has not committed a default in filing its financial statements under section
137 of the said Act or annual return under section 92 of the said Act with the Registrar), 2
small companies and 1 dormant company as these are excluded from the ceiling limit of
company audits given under section 141(3)(g) of the Companies Act, 2013.
(iv) As per fact of the case, RAJUL & Co. is already having 40 company audits and they can
accept only 60 more company audits. In addition, they can also conduct the audit of one
person companies, small companies, dormant companies and private companies having
paid up share capital less than Rs. 100 crores (private company which has not committed a
default in filing its financial statements under section 137 of the said Act or annual ret urn
under section 92 of the said Act with the Registrar). In the given case, out of the 80 private
companies RAJUL & Co. is being offered, 65 companies have paid-up share capital of Rs.
115 crore each.
Therefore, RAJUL & Co. can accept the appointment as an auditor for 2 small companies, 1
dormant company, 15 private companies having paid-up share capital less than
Rs. 100 crore (private company which has not committed a default in filing its financial
statements under section 137 of the said Act or annual return under section 92 of the said
Act with the Registrar.”) and 60 private companies having paid-up share capital of Rs. 115
crore each in addition to above 40 company audits already held.
6. (a) Matters covered in Reporting in case of Comprehensive Audit are: To facilitate a proper
consideration, the reports of the C&AG on the audit of PSUs are presented to the Parliament in
several parts consisting of results of comprehensive appraisals of selected undertakings
conducted by the Audit Board etc. Some of the issues examined in comprehensive audit are:
(i) How does the overall capital cost of the project compare with the approved planned costs?
Were there any substantial increases and, if so, what are these and whether there is
evidence of extravagance or unnecessary expenditure?
8
(ii) Have the accepted production or operational outputs been achieved? Has there been under -
utilization of installed capacity or shortfall in performance and, if so, what ha s caused it?
(iii) Has the planned rate of return been achieved?
(iv) Are the systems of project formulation and execution sound? Are there inadequacies? What
has been the effect on the gestation period and capital cost?
(v) Are cost control measures adequate and are there inefficiencies, wastages in raw materials
consumption, etc.?
(vi) Are the purchase policies adequate? Or have they led to piling up of inventory resulting in
redundancy in stores and spares?
(vii) Does the enterprise have research and development programmes ? What has been the
performance in adopting new processes, technologies, improving profits and in reducing
costs through technological progress?
(viii) If the enterprise has an adequate system of repairs and maintenance?
(xi) Are procedures effective and economical?
(x) Is there any poor or insufficient or inefficient project planning?
(b) 10% Voting Power and Control over the composition of Board of Directors: In this case,
Atishaya Ltd. holds only 10 percent of the voting power but has control over the composition of
the Board of Directors of Neenu Ltd.
In such a case, Atishaya Ltd shall be considered as a parent of Neenu Ltd and, therefore, it
would consolidate Neenu Ltd in its consolidated financial statements as a subsidiary.
The auditor should verify Atishaya Ltd’s management’s assessment of having control in Neenu
Ltd despite having only 10% voting power as per the requirements of Ind AS 110. Auditor would
need to verify as to how Atishaya Ltd controls the composition of the Board of Directors or
corresponding governing body of Neenu Ltd.
There can be various means by which such kind of control can be established. In this regard, the
auditor may verify the minutes of Board meetings, shareholder agreement entered into by the
parent, agreements with Neenu Ltd to which the parent might have provided any technology or
know how, enforcement of statute, etc.
Further, the auditor should verify that the adjustments warranted by Ind AS 110 have b een made
wherever required and have been properly authorised by the management of the parent. The
preparation of consolidated financial statements gives rise to permanent consolidation
adjustments and current period consolidation adjustments. The auditor should make plan, among
other things, for the understanding of accounting policies of the Atishaya Ltd and Neenu Ltd and
determining and programming the nature, timing, and extent of the audit procedures to be
performed etc.
Further, the duties of an auditor with regard to reporting of transactions with any other related
parties are given in SA 550 on Related Parties. As per SA 550 on, “Related Parties”, the auditor
should review information provided by the management of the entity identifying the names of all
known related parties. A person or other entity that has control or significant influence, directly or
indirectly through one or more intermediaries, over the reporting entity are considered as Related
Party.
In forming an opinion on the financial statements, the auditor shall evaluate whether the identified
related party relationships and transactions have been appropriately accounted for and disclosed
in accordance with Ind AS 110 and Schedule III and whether the effects of the related party
relationships and transactions prevent the financial statements from achieving true and fair

9
presentation (for fair presentation frameworks) or cause the financial statements to be misleading
(for compliance frameworks).
(c) As per SA 540 “Auditing Accounting Estimates, Including Fair Value Accounting Estimates,
and Related Disclosures”, the auditor shall obtain an understanding of the following in order to
provide a basis for the identification and assessment of the risks of material misstatements for
accounting estimates:
(i) The requirements of the applicable financial reporting framework relevant to the accounting
estimates, including related disclosures.
(ii) How Management identifies those transactions, events and conditions that may give rise to
the need for accounting estimates to be recognised or disclosed, in the financial statements.
In obtaining this understanding, the auditor shall make inquiries of management about
changes in circumstances that may give rise to new, or the need to revise existing,
accounting estimates.
(iii) The estimation making process adopted by the management including-
(1) The method, including where applicable the model, used in making the accounting
estimates.
(2) Relevant controls.
(3) Whether management has used an expert?
(4) The assumption underlying the accounting estimates.
(5) Whether there has been or ought to have been a change from the prior period in the
methods for making the accounting estimates, and if so, why; and
(6) Whether and, if so, how the management has assessed the effect of estimation
uncertainty.
Or
Difference Between Peer Review and Quality Review: Peer review is a review of the systems
and procedures of an audit firm. Although sample audit files are inspected by the peer reviewer,
it is done for the purpose of testing the effectiveness of the systems and procedures. The
intention is to not to find faults but to help the firm develop effective systems. It is a kind of
mentoring process. Peer review is a part of the activities of ICAI aimed at improving t he quality of
service.
In contrast, a quality review is supposed to act as a deterrent. Quality Review Board (QRB) is
constituted by the Central Government and is independent of ICAI. As per Section 28A of the
Chartered Accountant’s Act, the Central Government has the authority to constitute a Quality
Review Board. QRB carries out supervisory and disciplinary functions. A quality review normally
pertains to one particular audit conducted by an audit firm. The main objective quality review is to
find errors or inadequacies, if any, committed by the auditor while conducting the audit. Serious
errors detected in quality review lead to disciplinary action against the member.

10

You might also like